Twerski Tort Fall 2014
Twerski Tort Fall 2014
o
o
This doctrine applies to any of the five intent torts, and will suffice to make out the intent of the defendant.
If originally used and intended to use reasonable force, would be no battery and no transfer
B. Battery
1. Brzoska v. Olsen p16
Facts: Plaintiffs are ex-patients of , a deceased dentist who was infected with HIV; practiced with HIV for 2
years knowing of his infection, and stopped a few weeks before his hospitalization and death; Delaware
Division of Health told his patients of the issue after he died; none of the 630 patients were found positive;
Division deemed his sterilization practices were better than average; s sued over mental anguish, not
physical ailments.
Rule: Battery
o Intentional contact upon the person of another which is harmful or offensive;
o Contact, not harm; the result also must offend a reasonable persons integrity; Law does not permit
recovery for the extremely sensitive; must offend a reasonable sense of personal dignity
Issue 1: Can the performance of dental procedures by an HIV-infected dentist, standing alone, constitute
offensive bodily contact for purposes of battery, i.e., would such touching offend a reasonable sense of
personal dignity?
Issue 2: Can a patient recover damages for treatment by a health care provider afflicted with AIDS absent a
showing of a resultant physical injury or exposure to disease?
Holding: The incidental touching of a patient by an HIV-infected dentist while performing ordinary, consentedto dental procedures is insufficient to sustain a battery claim in the absence of a channel for HIV infection.
Such contact is offensive only if it results in actual exposure to the HIV virus.
***Footnote*** generally reasonableness is decided by a jury. Bottom p18 Footnote 9
(other case sent dead body to embalmer & didnt tell him it was AIDs infected body.)
Unlike assault where you need to be aware of the apprehension, battery you do not need to be. (girl kissed
while sleeping, when wakes up told she was kissed has a case Poisoned chocolate hypo bottom of page
20: no reasonable person would subject themselves to that kind of risk.)
2. Fisher v. Carrousel Motor Hotel
Facts: Occurs at a hotel restaurant in the 1960s; As was about to get food, Flynn (employee of hotel)
snatched his plate and told him a negro wouldnt be served there; testified that he was not touched, did not
testify to suffering fear, but highly embarrassed and hurt by .
Rule: Physical contact is not necessary to constitute a battery, so long as there is contact with clothing or an
object closely identified with the body.
Issue 1: Whether a battery exists when the physical contact is an act such as grabbing an item from another,
such as a plate in this case.
Holding: The forceful dispossession of plaintiff Fishers plate in an offensive manner was sufficient to
constitute a battery.
o Tort because the plate was snatched out of his hand, an undesirable touching.
Issue 2: Whether an employer is liable for the conduct of his employee manager.
Holding: The act is authorized by the employer if the agent was employed in a managerial capacity and was
in the scope of his employment, thus making the employer liable.
(Class discussion- w/o touching not a battery & not sure could make out intentional infliction of emotional
distress b/c not sure he suffered severe emotional distress w/o it dont have cause of action) (go after battery
if you have a choice b/c youre entitled to ancillary damages for emotional distress whereas to make out
severe emotional distress under intentional infliction of emotional distress is difficult). Court imposed punitive
damages on employer based on the fact the agent was employed in managerial capacity acting in scope of
employment and therefore punitive damages could stick. Huge debate past 10 years about punitive damages
& their size some awards that have been made are huge. 15 years ago Supreme Court got into business of
reviewing punitive damages as violations of due process when huge cases pending; court has cut down
punitive damages very substantially. Open ? in many cases the court has indicated that punitive damages
have to have a ratio between compensatory damages. Said ratio of anything over 9 to 1 might be
unconstitutional.
C. Assault
1. General Understanding of the Law
Assault affects the mind of the plaintiff to lead the plaintiff to believe that she is in immediate harm.
Battery does not require an assault
o Think of the example of kissing a girl in her sleep she does not know that it was happening.
Result of assault is someone being in apprehension
2. Western Union Telegraph Co. v. Hill
Facts: D is an employer and employee who verbally harassed a customer, the P; employee of defendants
shop (Sapp) invited plaintiffs wife behind counter in defendants office, and said that if she would let him pet
her, he would fix her clock; he told her he would fix it if she would let him love and pet her; he extended his
arm but did not touch her; she jumped back, thinking she was in danger of rape;
Rule: To constitute an assault there must be an intentional, unlawful, offer to touch the person of another in a
rude or angry manner under such circumstances as to create in the mind of the party alleging the assault a
well-founded fear of an imminent battery, coupled with the apparent present ability to make the attempt, if not
prevented.
Holding: Since the assault in this case was clearly from a motive or purpose solely and alone to satisfy the
sensuous desires of Sapp, and not in the furtherance of the business of the defendant, the liability rests with
the agent (Sapp) and not the employer; thus, the employer is not liable in this suit; Sapp was not sued
because he had no assets.
Possible Defenses: Since assault is an intentional contact of the mind tort, he could say had no intention of
rape; maybe they could say that he was negligent and that his conduct was unreasonable, but not that he was
tortious for wanting to rape the woman; negligent infliction of emotional distress would be a reasonable
charge, not assault. had no intent!
Why turn to law to consent if I had no intent of causing apprehension/harm I thought it was ok?
Court discussion of width of desk ; look @ drunk, joking or serious, etc.
*Assault is intentional tort; have to intend to cause apprehension and has to be suffered on behalf of Plaintiff
D. False Imprisonment
1. General Understanding of the Law
An actor is liable for the tort of false imprisonment when he intends to confine another within fixed boundaries
and the other person is conscious of the confinement.
o The most frequent litigated cases arise from customers or employees who are detained for questioning
because they are suspected of shoplifting.
False imprisonment is a lot like assault and not like battery.
o Assault is an intention of affecting ones mind; creates the apprehension of harm.
o Mind-to-mind tort, not physical
2. Grant v. Stop-N-Go Market of Texas, Inc.
Facts: P was accused of shoplifting from a grocery store; upon leaving the store, was grabbed by the
manager and accused of stealing a pack of cigarettes; grabbed his arm; police were called to the scene and
arrived in 15-20 minutes; P did not leave because he was scared that leaving would indicate guilt, though he
didnt steal the cigarettes; Part of P testimony is that D said he couldnt leave. Top p31 D said he didnt say P
couldnt leave. Disputed fact, go to jury dont get summary judgment. Trial court granted it; appeals court says
this is not summary judgment.
Rule: Elements of false imprisonment are (1) a willful detention, (2) without consent, and (3) without authority
of law. D has to intend to confine and P has to in fact be confined.
Issue: Is the false imprisonment claim applicable when the plaintiff feels as though he is not free to leave?
Holding: For P; although physical restraint is one way to establish a willful detention, it may also be
accomplished by violence, threats, or any other means that restrains a person from moving from one place to
another in this case, it is the grabbing of Ps arm and the fear of the cops coming; Ds claims for the
Shopkeepers Privilege are not valid because they do not satisfy the element of detention in a reasonable
time/manner: relies on Resendez for the reasonable time, but court rules that unlike in Resendez in which 1015 minutes were deemed reasonable, P was detained for 15-20 minutes in the store, and another hour in the
precinct, thus not making this applicable.
3. Shopkeepers Privilege
(1) A reasonable belief a person has stolen or is attempting to steal;
(2) Detention for a reasonable time (to investigate possession of property);
(3) Detention in a reasonable manner.
(Twerski says without explicitly telling him youre free to leave the situation is so coercive he feels confined.)
(the law says a lot depends on the facts did the person feel free to leave? First ? youll have to deal with. Now we get to
did the D intend (act w/ the purpose to or knowledge to substantial certainty) to confine the P? )
(Example Ranson- may have reasonably believed he had the cigarettes, shot @ a wolf and got a dog) because of that
most states have shoplifters statutes and prong 1 rids of ranson.
NYS Statute- reasonable time to make or refuse to make a statement Jacques vs Sears&Roebuck 285 NE 2nd 271?
Twerski doesnt like the time for the cops to come
E. Intentional Infliction of Emotional Distress
1. Harris v. Jones
Facts: P is a 26-year-old worker in a factory who has a stutter; D is his manager who has made fun of him for
his speech problem about 30 times; P was under the care of a physician for 6 years prior, prescribed pills for
anxiety; teasing and roughhousing were common in their place of business; P was deeply affected by this, led
to abusing his wife and a 2 week separation.
Issue: Can a person recover for a tort of intentional infliction of emotional distress when he is verbally teased
in his work place because of a speech impediment?
Rule: (1) Conduct must be intentional or reckless; (2) Conduct must be extreme and outrageous; (3) There
must be a causal connection between the wrongful conduct and the emotional distress; (4) The emotional
distress must be severe. (Restatement 46)
Holding: The humiliation was not so intense as to constitute severe
o The court did not rule on whether the abuse was extreme and outrageous. This case cannot be cited now
in a case where the abuse is in question. This is because ruling on such an issue would give a pretty
broad ruling as to what constitutes this sort of abuse.
o Lower ct said was extreme & outrageous but P didnt suffer severe emotional distress
o On Appeal MD Supreme Court they do not affirm the extreme and outrageous, they say we dont have to
reach that ? as a matter of law (because thats a ? of fact for a jury/sidestepping the ?) we are simply
going to decide that the P did not suffer severe enough emotional distress. When can decide on narrow
grounds that cant be repeated.
o First time severity becomes an element of the action if you dont have severe injury no matter how
outrageous conduct was you dont have cause of action. Courts have been very tough on what is extreme
and outrageous conduct. (they do not send them routinely to juries they step in)
o Twerski says emot distress is made of 2 parts- 1. Part that isnt extreme and 2. Part that is. The part
thats below the threshold causes x amount of emotional distress and if you were to be fair youd almost
have to discount for the part thats ok. You stink discount
F. Trespass to Land
1. General Understanding
A possessor of real property has a right to exclusive possession.
o One who intentionally enters land in the possession of another has committed the tort of trespass to land.
o One who mistakenly steps on anothers property, believing it to be his own, commits the tort too.
This is because it is still done intentionally. You intentionally step there thinking it is yours and the fact
that it was a mistake doesnt matter. Ranson helps this out.
2. Rogers v. Kent Board of County Road Commissioners
Facts: Decedent was mowing his property when he ran over a pole left in the ground by ; for two winter
seasons, obtained a license to put a fence and anchor posts parallel to decedents farm, on decedents
property; in the third winter, neglected to remove a steel anchor post, which was 6-8 inches above ground in
a meadow that had very tall grass, disguising it.
Procedural Posture- lower court found no cause of action for negligence b/c of government immunity but no
such immunity for trespass.
Rule: 160: Failure to Remove a Thing Placed on the Land Pursuant to a License or Other Privilege a
trespass may be committed by the continued presence on the land of a structure, chattel or other thing which
the actor has placed thereon.
Issue: Is leaving something on anothers property a trespass, once the contract has expired?
Holding: Failure to remove the anchor stake upon expiration of the license to have it on defendants land was
a continuing trespass and is alleged by plaintiff to have been a proximate cause of the damage, which she
seeks to recover. Reversed and remanded found not immune from negligence.
Twerski questions the restatement. Sees breach of K but not trespass. Questioning intent; negligent/forgot
but not intentional argument. Raises the issue not sure that theyre right.
G. Trespass to Chattels and Conversion
1. General Information
Twin torts of trespass to chattels and conversion deal with the intentional interferences with the personal
property of others. The difference depends on the seriousness of the interference.
o Trespass to Chattel
Minor interference
Defendant pays only the value of the harm caused to the chattel
o * Without damage to the chattel there is no cause of action for trespass to chattels
Requires proof of harm to chattel- included in cause of action (the severity of harm) similar to
intentional infliction of emotional distress. Why? If it wasnt it would be an administrative nightmare.
o Conversion
Serious interference
Law gives plaintiff the option of retaining the chattel and recovering the value of the harm from the
defendant or relinquishing the chattel to the defendant and recovering its fair market value
Twerski doesnt agree w/ Restatements dominion of control; Twerski believes intend to steal in bad
faith turns into converter or trespass to chattel gone bad.
A. Consent
1. General Concept
One is privileged to act in a certain way when one owes no legal duty to refrain from so acting.
o Term refers to conduct that would normally be prohibited, but, under the circumstances, is allowed.
The defendant bears the burdens of both producing proof to support the privilege/defense and persuading the
trier of fact of its validity.
One consents to the acts of another if one is subjectively willing for that conduct to occur.
o Consent can be communicated, but it can also be proved that the willing state of mind exists, and then
conduct may be legally privileged.
892. Meaning of Consent
o Consent is willingness in fact for conduct to occur. It may be manifested by action or inaction and need
not be communicated to the actor.
o If words or conduct are reasonably understood by another to be intended as consent, they constitute
apparent consent and are as effective as consent in fact.
2. OBrien v. Cunard Steamship Co.
Facts: Steamship on its way from Ireland to Boston had signs indicating that vaccinations would be given as
a convenience to its passengers to hasten their customs proceedings; P sues D because a surgeon
administers vaccination while she is waiting on line with other women who are getting vaccinated; when it
became Ps turn on line, she did not communicate clearly with the doctor that she had already had a vaccine,
and just needed the card indicating that she was vaccinated; the doctor gave her the shot because of this,
and as a result she suffered skin eruptions; P claims D is liable for battery.
Issue: Whether consent exists when the plaintiff does not verbally communicate that she would like to be
administered a shot; Did the doctor use force upon the plaintiff against her will?
Holding: Judgment for the D; Those in line with the P indicated by their conduct that they desired to avail
themselves to the provisions made for their benefit. There was nothing in the conduct of P to indicate to the
surgeon that she did not wish to obtain a card that would save her from detention at quarantine, and to be
vaccinated, if necessary, for that purpose. (court says overt acts) Surgeons conduct in light of the surrounding
circumstances was lawful; if her behavior indicated that she was consenting to the act, then his behavior is
guided.
Twerski questions whether she has a cause of action for battery. People treated as steerage, case is a
sign of how much info is a reasonable amount of info before you have informed consent- you have to look at
surrounding circumstances and social attitudes towards that activity.
How free and easy do we want underlying conduct to be? Set level of communication accordingly.
Question here is how much communication is reasonable? Court in 1880 viewed these people as steerage.
Twerski poses ? of whether this was a fancy lady would the attitude have been the same? Twerski thinks 17
yr old girl in line immigrant speaking up is a big communication. She got very sick (blisters, etc)
Twerski wants to see that whether or not consent is communicated, did the person have reasonable
grounds to believe it was ok to give the shot. Depends on social milieu of whats going on.
Bottom 60 HYPO. No consent here, back turned and no communication on her part. He did not intend an
offensive conduct. Unlike Ranson where you cant converse with the wolf, here put burden on D to get consent
and mistaken consent will run against D.
3. Hackbart(defensive end) v. Cincinnati Bengals, Inc. (Charles Booby Clark) (1979)
Facts: P is a football player who was intentionally hit by a player on the opposing team, D; D tackles P but
after he is down ran past him and struck a blow to his head and neck with sufficient force to cause both
players to fall forward on the ground; it was discovered by doctors that he had a serious neck fracture injury;
the actions of D were in clear violation of the rules set forth by the NFL (p63) , but the incident occurred after
the play.
Issue: Whether consent to play in a sport, such as football that is violent, negates liability from violent acts
that take place on the field of play.
Holding: For D; pervasive culture of violence in professional football makes it very difficult for court to
differentiate between on-the-field incidents that are fair play and those that merit legal sanctions; we take the
view that this was not a proper issue for determination and that D was entitled to have the case tried on an
assessment of his rights and whether they had been violated.
Discussion: According to the trial judge, when a player steps onto the field he consents as a matter of law to
all violent contact that could be construed as part of the physical struggle commonly referred to as
professional football; it could be argued that if the hit had been made before the whistle blew as opposed to 2
seconds after, that it simply would have been a penalty within the game; a court should not have to distinguish
between these fine points of the game it should be established within the rules of the game of football
beforehand.
Twerski says let them contract to how to handle these issues or have the NFL sanction. Doesnt
understand why this is a court business. These are things that take place in games all the time. This case
raises for him why the court system should be made to decide this rotten ?. if it had been before the whistle it
isnt a case.. dangerous sport. Are the emotions that were ok 3 seconds before not ok 3 seconds after??
Doesnt think we should be figuring this out think about Alternative Dispute Resolution.
If one believes the coach would testify to degree of anger theyre put to before the game seems to
Twerski if it occurred 2 seconds earlier wouldve been a foul, and ? is now is the fact that it takes place 2
seconds later significant enough. He has a problem with that and tends to agree w/ lower court. More of a
process ? though why the heck is this in court? High price f-ball players with very sophisticated unions not
contracting on this issue.
4. Implied-in-Fact Consent
The consent tacitly given by one individual to another, based on their shared history, is implied-in-fact
consent.
o Example: If friends engage in patterns of roughhousing or practical jokes, a court might find that they
have established a factually based implication of consent to such behavior.
o Implied-in-law consent has an off the rack quality think of a Toshiba computer, whereas implied-in-fact
consent is tailor-made think of a Dell to each unique set of factual circumstances.
o
Christman v Davis 889 A.2d 746 (Vt 2005) p66
PROCEDURAL POSTURE: Plaintiff patient sued defendant dentist for dental malpractice, lack of informed consent, and battery.
The patient dismissed all but the battery claim b/c never consented to flap procedure. The dentist filed a motion for summary
judgment. The Chittenden Superior Court (Vermont) granted the dentist's motion. The patient appealed.
OVERVIEW: The patient consented to have the dentist perform a tissue graft to obtain root coverage. After beginning the procedure,
the dentist determined that he would perform a flap procedure instead. The patient was surprised that he did not receive a full graft and
later learned that he would need to undergo a tissue graft because the flap procedure did not achieve full results. On appeal, the patient
argued that he did not give consent to perform the flap procedure and therefore the dentist committed a battery. The dentist indicated
that the flap procedure was a less invasive procedure done to determine whether there was sufficient tissue of adequate quality to
perform the graft.
The supreme court concluded that the procedure done was consented to and thus, did not support the battery claim because battery is
only when you exceed the consent. To the extent that the patient had an actionable claim, it fell within the lack-of-informed-consent
line of cases. Because the patient conceded that the flap procedure was necessary to the tissue graft, there was no material issue of fact
as to whether the consent covered the flap procedure.
OUTCOME: The judgment of the trial court was affirmed. Say battery is only if you exceed consent, if you havent and other things
go wrong (malpractice/informed consent) it is not battery. Court sees informed consent as different than battery.
Twerski says if it was very clear that she agreed only to graft procedure the dentist made the choice and if he believes her testimony
he thinks that theres a strong case for battery. No one is saying youre negligent you exceeded your consent and therefore thats a
battery argued by Twerski.
P71 Bundrick v Stewart consent forms (too broad) here where someone says to doctor I dont want a resident to touch
me the doctor agrees or doesnt.
5. Kennedy v. Parrott
Facts: goes in for surgery, doctor diagnoses appendicitis; opens her up and sees a large cyst on her
ovary that requires operation; while puncturing cyst, he cut a blood vessel and injured her leg by causing
phlebitis; sues for battery, claiming she did not consent to operation of the cyst on ovary.
Issue: Whether a doctor can perform an operation that has not been consented to when it arises out of an
emergency rendering the patient incapable of consenting.
Holding: Court holds that it is unreasonable for to finish operation and inform the of the cysts, and then
after receiving consent, open her back up and operate; where an internal operation is indicated, a surgeon
may lawfully perform, and it is his duty to perform, such operation as good surgery demands, even when it
means an extension of the operation further than was originally contemplated, and for so doing he is not to be
held in damages for an unauthorized operation.
6. DeMay v. Roberts (here were talking about D has to tell P; obligation to disgorge himself to the P up until now
weve discussed P tell D)
Facts: D is a doctor making a house call to P in 1881; the weather was bad out, so D brought along an
assistant who had no medical knowledge to help him carry his supplies; P allowed both men into the home,
but there was no mention by D of who the assistant was; during labor, wife kicks her maid, who was holding
her hand, in the stomach; D tells assistant, who was not watching or near the process, to step in and hold her
hand while maid is recovering; he does this for a few seconds, and then is relieved by the maid.
Issue: Whether there was implied-in-fact consent when P allowed both men to enter, under the presumption
that they were both physicians.
Holding: D was guilty of deceit because at the time, she consented to the presence of Scattergood
supposing him to be a physician does not preclude P from maintaining an action and recovering substantial
damages upon afterwards ascertaining his true character; D has the obligation to present the proper
information to P;
Twerski ?s why isnt the obligation on P to ask who he is. She isnt in labor yet. Assuming this is a first year
med student and this is his first case, is that ok? Assuming he never touched her, what tort would we have?
Trespass to land. Battery will not work b/c had a nonconsensual privilege to act since she was in danger.
7. Informed Consent
Informed consent is the understanding of a decision based on adequate information about the treatment, the
available alternatives, and the collateral risks.
As essential as a physicians care and skill in the performance of the therapy.
If a physician breaches this duty, patients consent is defective, and physician is responsible for the
consequences.
o Battery exists when treatment is unauthorized and performed without any consent at all.
o If the physician obtains a patients consent but has breached his duty to inform, the patient has a cause of
action sounding in negligence for failure to inform the patient of his options, regardless of the due care
exercised at treatment, assuming there is injury.
The scope of a physicians communications must be measured by his patients need to know enough to
enable him to make an intelligent choice.
o Full disclosure of all material risks incident to treatment must be made material meaning it would likely
affect patients decision.
Exceptions to Informing
o No need to disclose risks that either ought to be known by everyone or are already known to the patient;
o When full disclosure would alarm an emotionally upset patient (therapeutic privilege);
o Almost never used would undermine informed consent
o An emergency situation in which the patient is in no condition to determine for himself whether treatment
should be administered
Rule: No consent = Battery; No informed consent = Negligence
8. Scott v. Bradford
Not a word in this case about negligence didnt inform her of the risk of incontinence that comes along with
hysterectomy so even non-negligently performed surgery may be a risk for incontinence. Subsequent surgeries to fix
this, and sues for his lack of information. Jury finds for D and argument is jury instructions werent adequate. Court
finds jury instructions were pretty ok but this opens up the court to discussion to what makes out a cause of action for
informed consent for failure to give patient adequate information. The elements of this tort.
Facts: performed surgery on , which was consented; post-op, experienced problems with incontinence
unable to control the bladder; consult with another physician who discovered she had a condition that
permitted urine to leak from the bladder to the vagina; she was then referred to a urologist who required three
surgeries to remedy the problem; claims that failed to advise her of the risks involved or of available
alternatives to surgery and if she had been properly informed, she would have refused the surgery.
Rule: Lack of informed consent is divided into three elements:
o The duty to inform
Defendant physician failed to inform adequately of a material risk and alternatives before securing
his consent to the proposed treatment;
Amount that this reasonable patient would want to have.
Two tests for materiality; how much information required to give
As opposed to the reasonable doctor test
Amount of information doctor must disgorge himself of is twofold:
Court acknowledges there are two tests to determine whether or not doctor gave adequate
information: 1. Canterbury rejected but test was reasonable doctor test that applies to malpractice
cases ? placed to jury would be did the doctor give the info that a reasonable doctor would give? 2.
What would a reasonable patient want to know? In the latter, do not need a medical expert to testify
that someone violated medical custom. You might still need a doctor to get on the stand to tell you
about what the risks are. Question of what a reasonable patient wants to know thrown to jury. This is
hopelessly open-ended according to Twerski.
Assuming you got past either of these tests had the patient been given the information, what would
they have done with it.. is there a causal relationship betw failure to communicate and the choice that
was made?
o Causation
Requires would have chosen no treatment or a different course of treatment had the alternatives
and material risks of each been made known to him;
If the patient would have elected to proceed with treatment had he been duly informed of its risks,
then the element of causation is missing;
Test for causation
Would a reasonable person have chosen against the surgery? (use Canterbury reasonable
patient test) Problem with this is that its not what I wouldve done and worse than that,
Twerski says think about this- the doctor is not guilty of malpractice and not negligent for
recommending hysterectomy, not negligent in performing it, why did the doctor recommend the
hysterectomy-cause she needed it now that you ask, the risks pale in comparison to what
wouldve been had she not had it. What would a reasonable patient have done? Listen to a
reasonable doctor because by definition the doctor is not negligent. By definition an informed
consent case, not negligent made right decision and now going to ask reasonable patient test
wouldve made. Destroys the right of self determination. I dont have to be reasonable its my body
and I know the answer to the ? if not negligent in recommending the reasonable patient will listen
to reasonable doctor cause if not youd be suing for malpractice. Canterbury test basically undoes
what they intended to do, it puts back reasonable doctor test. Therefore doesnt make any
difference.
A careful petitioner can always protect himself by insuring that he has adequately informed each
patient he treats. If he does not breach this duty, a causation problem will not arise.
Court says the true test should be what this patient would have done. Creates the
autonomy. Would this patient have decided they wouldnt undergo the surgery or taken an
alternative. Patients going to say I would not have done it. Court recognizes that. If they say that,
the defense lawyer will cross examine them to death bleed to death 4 out of 5 times instead of
incontinence. There are cases where plaintiff can realistically say had I been given the alternative
I wouldnt have done this b/c this was a risk I was not willing to take.
o Injury
The adverse consequences that were not made known did in fact occur and he was injured as a
result of submitting to the treatment.
Holding: A new duty of informed consent is being imposed on physicians; the instructions objected to by
did instruct that should have disclosed material risks of the hysterectomy and feasibility of alternatives.
Instructions are sufficient when considered as a whole they present the law applicable to the issues; this is not
a medical decision; it is a human decision.
Is something called therapeutic privilege? Dont have to tell patient things that will so upset the patient hat
will adversely affect their health/state of mind. Problem with it is if you read it broadly youre going to destroy
informed consent. Twerski has yet to see a therapeutic privilege case. Theoretically out there, but he hasnt
seen it practically.
Discussion of Causation: would not have arisen without a battery.
o Causation test
Would the patient have chosen a different alternative?
Big deal because the ball should be in the patients court, not the doctor he should not have the
right to decide what is best for the patient if the patient is available to make the decision.
What would a reasonable patient have decided?
Canterbury used this, but it is wrong because it should be applied to the particular case
o How do you know if the current patient is reasonable? The patient could have some bizarre
defect that it unreasonable, and therefore requires you to decide on THIS patient.
o It can be argued that a reasonable patient would listen to a reasonable doctor, so this
becomes circular.
o What would this patient have decided?
Major holding of Scott as a deviation from Canterbury.
Argument: unfair to doctor because he would have no idea what this patient would have done.
Response: if the doctor had performed his duty and informed the patient, then this would not be
an issue
o Reasonable Doctor Test: court does not like this because it undermines patient autonomy
Advantages
Expert needed to testify (difficult);
Substantively difficult different doctors weight substantive risks differently;
Easier for doctors because it deals with the doctors point of view (how can you know what a
reasonable patient would need?)
Disadvantages
Doctor has no way of measuring what a patient would deem reasonable.
Courts dont want to call this battery b/c dont want to treat doctors that way; put them in negligence rubric and have to
show causation to harm that took place.
How do you assess damages in this case? Question missing from this case Twerski says had you not had this
hysterectomy, you mightve bled to death. The function of damages in tort law is to put you back in status quo. Yes you
have suffered damages of surgery, but you didnt come into the office very sick. Consequences w/o the hysterectomy.
Why dont we compare the risks with each? Are you entitled to full scope of damages for what has now happened to you?
Whats going on here is basically its a dignitary tort we are offended that she did not get the right to make her decision.
We translate the value of the dignitary tort in terms of value of hard damages which actually suffered which differ from
value of dignitary tort. ? here to be grappled with.
Twerski review up until here : in a case based on informed consent, were not dealing with a claim of negligence in
diagnosis by doctor. Not dealing w/ negligent performance of surgery. That kind of negligence is out of the picture, if you
had it you would be suing for malpractice or failure to perform adequately. Not medical malpractice of negligence, its the
medical malpractice arising from failure to inform.
First question in any informed consent case is how much info should the dr have given the patient.. the courts call this
materiality. (if patient asks for more information and seeks it and doctor doesnt give it, the patients entitled to as much info
as they want as doctor is entitled to say I dont want to give it to you go elsewhere) the problem most of the time is the
patient doesnt know enough to ask the ?... and then the ? is what is the minimum amount of info that doctor has to
disgorge him/herself of in giving the information to the patient. Materiality- two tests reasonable doctor test (med
malpractice test) that requires expert testimony as to fact that doctor failed to meet medical custom could not make out
case w/o it. Canterbury case separated from reasonable doctor test b/c it was really a human autonomy question,
changed it from how much info the reasonable doctor should give but instead opted for a reasonable patient test. How
much would a reasonable patient want to know.. much more difficult test for doctors to comply with, but doesnt require
expert testimony tells us what custom is but may need testimony for what the risks are. Canterbury is viewed as more
enlightened view because provides for greater human autonomy once risk info is out, up to jury to decide whether the
info is the kind of info a reasonable patient would want to have before undergoing the procedure.
Next question is assuming doctor didnt provide x amount of info (failed either materiality test) would it have made any
difference? Decision/causation ?... would a reasonable patient have decided differently or would the patient have had the
procedure anyway. 2 tests. 1. Canterbury test was reasonable patient test- what would reasonable patient have done?
Had the therapeutic intervention or not 2. Scot v Bradford test, what would THIS patient have done?
Scot court criticizes reasonable patient test b/c it undoes autonomy if youre supposed to give patient reasonable info a
reasonable patient would ought to have the Canterbury ? misses the boat. Since doctor isnt negligent in performing,
the ? what would the reasonable patient have done is listen to reasonable doctor and defeats autonomy purpose. On
cross the defendant counsel could really work over the plaintiff by saying would you really have risked bleeding to death
just because of risk of incontinence?
Causation ? has to be answered because court is using negligence standard where have to show standard of care and
causation. Courts have made it very clear wont use battery for informed consent offended by using idea of intentional
tort like that against doctors, so put it into context of negligent informed consent. Gets them to ask ? what decision would
you have made if its unconsented to touching, the answer might be relevant to damages she might get but not cause of
action. Courts dont do that- view it as negligence and require breach of standard of care and causation.
Twerski added 3rd element aside from decision causation but injury causation asks the ? as to why it is that after we find
the violation of informed consent and even find that theres decision causation, had the patient decided otherwise now she
suffers incontinence. We give her all the damages for the incontinence, on the other hand, had she not had the
hysterectomy she mightve well have suffered substantial consequences in any event. May have suffered fairly serious
consequences. Why dont we take those into account when establishing consequences? One sick person before given
inadequate info.. failure to have it she mightve had a whole lot worse/complications equally serious. Should those be
taken into account in setting the injury? Twerski thinks they should but courts dont.
[p81 NY statutorily brought in reasonable doctor test & reasonable patient test for causation. If youre a patient in NY you
have worst of both worlds attempt to limit doctors from malpractice and absolutely did it.]
p83 medical provider statistics. ? is if doctor has that info does the hospital have obligation to give you that info?
Twerski argues in fact med provider statistics are in one sense an easier case than the classic issue of whether or not I
shouldve told you alternative risk associated w/ surgery. Ex: scot v Bradford, doctor didnt tell you 1 in 500 risk of
incontinence said dr wasnt negligent b/c gave good advice about hysterectomy, ? is then what would this woman have
done had she had this info? In medical provider statistics- the ? isnt whether she wouldve had the surgery, but whether
shed have it with a doctor with high or low performance rate? Thats a lot easier if disparity is significant.
B. Self-Defense
1. Courvoisier v. Raymond
Facts: heard people trying to break into his jewelry store, which was on the first floor below his apartment
where he was sleeping; he used his gun to expel the intruders from the building; he fired three shots into the
air until they fled; shots attracted police officer/D to the scene with two other officers; D approached, and the
other two stayed back; P fired and hit D, thinking he was one of the gang members who he had chased away.
Issue: Did P act out of necessary self-defense to reasonable fears under the circumstance that he thought his
life was in danger?
Reasoning: this is Talmage (right to get off his property and use reasonable force kids on the shed) the
court says that if had the right to shoot to defend, and it was an inadvertent hitting, then he within reason; if
you act in a way that you have the right to act, then it is self-defense; differs from Ranson because in that
case he did not have to shoot the dog; in this case, he had to defend himself due to an invasion.
What was wrong w/ trial judge instruction? Didnt really explain self defense well said if he
assaulted/battered the guy and got the wrong guy he was liable court said nothing about fact of justification.
Why is this not Ranson? He was defending himself from invasion whereas Ranson didnt have to shoot the
dog. Courvoisier is REACTING whereas Ranson was ACTING. Act of privilege is not tortious has a right to
shoot, fact it ended up badly is tough but hes in reactive mode is treated as privileged and therefore no
liability.
2. Restatements
63. Self-Defense by Force Not Threatening Death or Serious Bodily Harm
o An actor is privileged to use reasonable force, not intended to or likely to cause death or serious bodily
harm, to defend himself against unprivileged harmful or offensive contact that he reasonably believes that
another is about to inflict intentionally upon him.
65. Self-Defense by Force Threatening Death or Serious Bodily Harm
o An actor is privileged to defend himself against another by force intended or likely to cause death or
serious bodily harm, when he reasonably believes that the other is about to inflict upon him an intentional
contact or other bodily harm, or that he it thereby put in peril of death, which can be prevented only by the
immediate use of such force.
70 and 71
o Address the issue of excessive force, limiting the self-defense privilege to that amount of force that the
actor correctly or reasonably believes to be necessary to protect himself, and holding the actor who uses
excessive force liable only for so much of the force as is excessive.
C. Defense of Others
1. General Concept
American courts have extended a privilege to actors who intervene and use force to protect and defend
others from threats by third persons.
Privilege is same as self-defense.
D. Defense of Property
1. Katko v. Briney
Facts: D had house they inherited with valuable jars. Had no trespass signs. D set up a shotgun trap pointed
at the would-be intruders leg, triggered by the opening of a bedroom door; no warning of the guns presence
was posted; P thought the building to be abandoned; Ps tibia was blown away as a result of the shot.
Issue: Is using a spring gun within reasonable force in the protection of ones property when the intruder
poses no threat to the owners personal safety?
Rule: There is no privilege to use any force calculated to cause death or serious bodily injury to repel the
threat to land or chattels, unless there is also such a threat to the defendants personal safety as to justify a
self-defense.
Holding: Judgment for P for $20,000 and 10,000 who was shot upon trespassing because a person owning
property is prohibited from setting out spring guns and like dangerous devices that will likely take life of inflict
great bodily injury, for the purpose of harming trespassers; the fact that the trespasser may be acting in
violation of the law does not change the rule; value life over property.
E.
F.
G.
H.
I.
Court mentions no warning was given. Twerski doubts if you have signs saying this house is laced with
shotguns wouldve gotten them off the hook (may be wrong) but wouldve been a good step. Absolute value of
property was midemeanor, used spring gun to do what they wouldve had a right to do if there but werent
there, unkempt lawn looked abandoned Twerski wonders if it wouldve been diff if it was commercial prop in
a high crime area w/ posted signs about spring guns.
Recovery of Property
1. General Concept
Courts recognize a privilege to use reasonable force to regain a chattel tortiously taken by another so long as
the rightful possessor acted promptly in hot pursuit after dispossession or after timely discovery of it.
o Once the sense of immediacy is lost, the self-help privilege is gone.
o Force likely to cause death or serious bodily harm is never permitted to recapture property.
Necessity
1. Vincent v. Lake Erie Transportation Co.
Facts: A storm caused Ds ship to be docked at Ps wharf overnight until the weather cleared; during this time
no captain would have brought the ship out to sea; the ship was docked in a legal place on the wharf, and
lines were used to keep it from floating to sea; the storm was of such force that it the boat was constantly
being lifted and thrown against the dock (which court says right thing to do b/c without doing so wouldve been
a guided missile unpredictable), resulting in the damage sought by P.
Issue: Does the necessary conduct of the D to keep the ship docked during the storm prevent it from being
held liable for the damages to Ps property?
Reasoning: Judgment for P because those in charge of the vessel deliberately and by their direct efforts held
the ship in such a position that the damage to the dock resulted, and, having thus preserved the ship at the
expense of the dock, it seems to us that her owners are responsible to the dock owners to the extent of the
injury inflicted.
o In other words, the D damaged Ps property for the purpose of preserving its own more valuable property,
and the Ps are therefore entitled to compensation.
Discussion: If the ropes never broke, causing it to be refastened to the dock, or if he had done nothing to put
the boat against this particular dock, it wouldve simply been considered an act of god, and there would be no
liability.
o The situation is one in which the ordinary rules regulating property rights were suspended by forces
beyond human control, and if, without the direct intervention of some act by the one sought to be held
liable, the property of another was injury, such injury must be attributed to the act of God.
The reasoning (above) is the specific reason why in this case the court rules against the ship.
The moment he was on the dock he had a right to stay there; P no right to cut the tie from the dock
(Putnam cited in the case)
Bottomline is have to consider if we do not have the shipowner pay the dock owner, there are things
the dock owners can do to make their docks very inhospitable to rescue. Does dock owner have a
duty to make his dock a safe haven for every ship? No. this court makes it more hospitable to rescue
because the economic burden isnt placed on dock when danger arises.
Legal Authority
1. General Concept
When public officials act within the limits of their predefined roles, exercising the power and authority that
those roles afford them, such officials are not subject to liability for those actions.
Disciplining Children
1. General Concept
The law privileges parents to use reasonable physical force in disciplining their children.
Teachers and other school officials also possess a similar privilege to inflict reasonable corporal punishment
upon students.
o Majority of states now either strongly discourage or completely prohibit corporal punishment.
o However, even those states that prohibit corporal punishment recognize that in some situations, a teacher
much be permitted to contact a student.
An Umbrella Justification Defense
1. Sindle v. New York City Transportation Authority
Facts: Students on bus committed vandalism to the bus; driver (employee of ) inspected damages and
advised students he would be driving directly to the police station without making any more stops due to the
damage they had caused; following the lead of others, attempted to jump out of the window to escape, and
upon doing so got run over by the bus.
Issue: Did the bus driver have the discretion to take reasonable measures to protect the safety of the
property and the students in his custody? Reasonable force for defense of property? Issue for jury whether he
was justified for doing what he did.
Reasoning: Judgment for because although confinement reasonably perceived to be unlawful might invite
escape, the person falsely imprisoned is not relieved of the duty of reasonable care for his own safety in
extricating himself from the unlawful detention. In other words, if youre going to escape from what you think is
false imprisonment, you still must use care, or its on you.
o Restraint and detention, reasonable under the circumstances and in time and manner, imposed for the
purpose of preventing another from inflicting personal injuries or interfering with or damaging real or
personal property in ones lawful possession or custody is not unlawful. Was he acting in their defense
and in defense of his own property?
o Theres a diff between reacting when being threatened and make reasonable mistakes than acting and
making reasonable mistakes.
o Exam ? last semester which had somebody locking students in library who thought someone dangerous
was around would think same logic would apply.
Problem Twerski sees with umbrella justification defense is that it could run counter to what we have been talking about all
along (in Ransen, you shoot, believe youre shooting at wolf and turns out to be dog- is that now a justification defense
because youve acted reasonably under the circumstances) Could undercut the views weve had w/ regard to
intentional torts up until now.
Chapter 3: Negligence
A. Introduction
1. General Concept
A theory of law that judges almost all conduct as to whether it meets the standard of what a reasonable
person would do under similar circumstances provides enormous latitude to injured persons seeking redress
for harm done to them. Sweeping in its scope there is a one line test which is supposed to work for most; did
you act as a reasonable person under the circumstances. Tort law is dominated by negligence; pockets of
strict liability and products liability.
2. Elements of Negligence
Duty betw D and P
o Without a duty to act, there can be no liability.
We are only dealing with cases that people have a duty. start off with belief we have a general duty
of reasonable care
Breach of Duty
o Once a duty has been established, plaintiff must establish that the defendant failed to act reasonably.
o Defendant must violate standard of care (1. What is the standard, 2. Factual ? did you reach the
standard- juries decide, huge amount of discretion and only rarely judge steps in with directed verdicts),
acting as a reasonable person under particular circumstances.
Cause-in-fact
o Plaintiff must prove a connection between the defendants negligent conduct and the harm suffered.
Proximate Cause
o A single negligent act may produce untold and unforeseeable consequences.
o Crazy things happen in tort cases and sometimes risks happen and the court will absolve you from
liability.
o Fact ? if wasnt negligent, would it have mattered anyway. Is the injury in the scope of your negligence?
Limits of liability.
Harm
o You must prove actual injury. Without harm there is no negligence claim
When jury makes finidng of negligence, it does 2 things
o It sets a standard of care
o Found that standard of care has been breached
B. Negligence Balancing
1. Lubitz v. Wells
Facts: left his golf club lying on the ground in his backyard; his son, while playing in the backyard with his
friend (), swung the club at a rock and hit in the face.
Issue: Is someone negligent for leaving in his property an object that he should have known could be
harmful to kids?
Holding: Judgment for because it would hardly be in good sense to hold that this golf club is so obviously
and intrinsically dangerous that it is negligence to leave it lying on the ground in the yard.
Discussion: Dismissed on demurrer because theres no cause of action. Reasonable people differ in how
they would judge something to be intrinsically dangerous.
o No definition given of what makes some intrinsically dangerous.
o Whether under a given set of circumstances conduct is negligent is for the trier of fact.
o Twerski poses ? why demurrer and not summary judgment to develop facts & then decide. Court says
case is gone we arent taken these kind of cases as a matter of law. No fact would make a difference.
This sounds like court is setting legal standard of care/duty no liability for golf clubs left in backyard.
Negligence is like playing a violin have whole symphony going and with all those facts did the D act
reasonably. When says theres no duty this is a common object no case for golf club in backyard, thats
it. Moment you make a rule inherently dangerous people come and say rule doesnt apply to me, I dont
fit that rule.
That continued to be aware of the danger and take exemplary precautions to avoid it until his fatal
accident was illustrated by the care that he and his friend took when they lowered and laid the antenna
next to the fence several days before the accident.
Discussion: Jury finds liable, Appeal reverses, Supreme Court affirms reversal.
Viewing this problem in terms of algebra
o High degree of gravity of loss (L) multiplied by the very small possibility of the accident occurring (P) in
this case, it is clear that the product does not outweigh the burdens or costs of the precautions of
relocating or insulating the power line.
o The burden to the company of taking precautions against all such slight possibilities of harm should be
balanced against the total magnitude of all these risks, including the relatively few losses resulting from
the total of all those insignificant risks. The burden will be a lot greater once move out of this 150 feet but
ultimately the court is deciding/the bottomline is the risk is not worth the burden. Cost of insulating
150/250 miles of wire, there may be a death or two over 20 years, what are we supposed to do. We as a
matter of social policy are directing a verdict will not impose those costs on rate payers of Louisiana.
o P in this case is very low (read above bullet why).
o B was the guys backyard for the case, but B is all wires in general.
o Court is saying that as a matter of safety, the wires current status and protection of being raised at a
certain level above the ground is safe enough.
4. Restatements (Second) of Torts
291. Unreasonableness; How Determined; Magnitude of Risk and Utility of Conduct
o Where an act is one in which a reasonable man would recognize as involving a risk of harm to another,
the risk is unreasonable and the act is negligent if the risk is of such magnitude as to outweigh what the
law regards as the utility of the act or of the particular manner in which it is done.
In other words: Unreasonable if Risk > Utility
292. Factors Considered in Determining Utility of Actors Conduct
o In determining what the law regards as the utility of the actors conduct for the purpose of determining
whether the actor is negligent, the following factors are important:
The social value which the law attaches to the interest which is to be advances or protected by the
conduct;
Extent of the change that this interest will be advanced or protected by the particular course of
conduct;
Extent of the change that such interest can be adequately advances or protected by another and less
dangerous course of conduct.
293. Factors Considered in Determining Magnitude of Risk
o Social value;
o Extent of the change that the actors conduct will cause an invasion of any interest of the other;
o Extent of the harm likely to be caused;
o Number of persons whose interests are likely to be invaded if risk affects them.
5. Liability for Foreseeable Risks
Factors are evaluated from the vantage point of the actor prior to engaging in conduct.
The question becomes whether a reasonable person would have invested more resources to learn about
potential risks before acting.
o Knew or should have known about the risks.
C. Qualities of the Reasonable Person
1. General Concept
The general negligence balancing formula judges the actor by the standard of what a reasonable person
would have done under the same or similar circumstances.
o First, what data was available? This is a subjective analysis.
o Second, should you have known the risk? This is an objective analysis.
o What are the costs of getting better information
Where a defendant holds herself out to have expertise and another relies on such representation, there is no
question that she is held to have the general knowledge and skill of that field of expertise.
Superior knowledge and skills should be taken into account in determining whether an actor has behaved as
a reasonable person.
2. What the Reasonable Person Knows
Delair v. McAdoo
o Facts: tries to pass on the road; s rear left tire blew out and caused s car to swerve into contact
with s car; claims that was driving with defective tires; two witnesses for the state that the tires
o
were worn through to the thread; the repairman who replaced the worn tire said that he could see the
breaker strip on the tire, just under the fabric.
o Rule: Apply the reasonable person test, balancing available data against if actor should have known risk.
o Issue: Is an ordinary man deemed to have the knowledge of what constitutes a dangerous tire?
o Holding: Any ordinary individual, whether a car owner or not, knows that when a tire is worn through to
the fabric, its further use is dangerous and it should be removed.
An owner or operator cannot escape because he says he does not know. He must know. The hazard
is too great to permit cars in this condition to be on the highway.
The law requires drivers to know the condition of those parts that are likely to become dangerous
where the flaws or faults would be disclosed by a reasonable inspection.
3. How the Reasonable Person Responds to Emergencies
Cordas v. Peerless Transportation Co.
o Facts: A mugging occurred on the streets of New York City; the mugger hopped into s cab and at
gunpoint ordered him to drive away from the scene in a hurry; complied, and fearing for his life, jumped
out of the cab while it was still in motion; the mugger then got out as well and fled; the cab continued on
and went onto the sidewalk where it hit s.
Cause of action is the contention that the driver was negligent in abandoning the cab under aforesaid
circumstances.
o Rule: The law in this state does not hold one in an emergency to the exercise of that mature judgment
required of him under circumstances where he has an opportunity for deliberate actions. He is not
required to exercise unerring judgment, which would be expected of him, were he not confronted with an
emergency requiring prompt action.
o Issue: Does an actors actions in an emergency situation make him negligent for any resulting
consequences?
o Holding: Judgment for driver because it would be ridiculous for the law to require an ordinary man to
behave in the same way in an emergency situation, than he would have otherwise acted if the emergency
did not exist.
Negligence has been variously defined but the common legal acceptation is the failure to exercise
that care and caution which a reasonable and prudent person ordinarily would exercise under like
conditions and circumstances.
o Discussion: Applying the reasonable person test to an emergency
How does emergency factor into the formula?
You do not have enough time to gather adequate data insufficient data as opposed to someone
who is driving along normally and would be able to prevent hitting the .
If your data is imperfect, the next question is what is the cost of getting additional data?
o Cost of getting additional data in emergency is extremely high.
o Even if he were thinking of staying in the car to gain additional data, the cost of staying in the
car is the risk of getting shot by the mugger.
Also, he has a very limited time to make a good judgment.
Good reasons to let defendant off the hook.
o Not true in every circumstance.
Even given the data that you have, you can act negligently.
o Seriously affects the data and judgment making capability.
He either does see or does not see the s. His P is uncertain; his B is uncertain
4. Does the Reasonable Person Follow Customary Practice?
General Concept
o Where there is a firmly entrenched custom, the party challenging the custom bears a significant burden in
presenting evidence that the custom is ill conceived. In short, customs are not sacrosanct. On the other
hand they are not to be thrown to the wind.
o If an actor departs from industry custom, she has little hope of prevailing before a jury. The only real way
to win a case like this would be to argue that the custom speaks to a practice that is not directly on point
with the case at bar.
Trimarco v. Klein
o Facts: P, upon exiting his bathtub in his apartment, got injured when the glass enclosure shattered;
expert testified that the practice of using shatterproof glazing materials had come into practice in the early
1950s; bulletins of national safety and consumer reports were issued about this.
o Issue: Is a landlord required to install safer glass doors as per the common practice in similar situations?
What is the reasonableness of the custom of landlords installing these doors?
Holding: Judgment for P because when proof of a customary practice is coupled with a showing that it
was ignored and that this departure was a proximate cause of the accident, it may serve to establish
liability. The jury can decide whether, at the point in time when the accident occurred, the modest cost and
ready availability of safety glass had transformed what once may have been considered a reasonably
safe part of the apartment into one which, in the light of later developments, no longer could be so
regarded.
o Discussion: This case deals with custom practice in similar situations. argues that custom was that
they would replace everything when something was broken, not before something breaks. But, this
doesnt apply to this case because this case regards something that should have been replaced before
the accident, not after. If it had been done before, the resulting injury would have been prevented.
5. Physical and Mental Attributes of the Reasonable Person subjective
Roberts v. State of Louisiana
o Facts: was on his way to the bathroom, without using his cane; he bumped into causing him to fall
and injure his hip; alleges respondeat superior; blind guy was the operator of the concession stand
located in the building.
Respondeat superior: states that an employer is responsible for employee actions performed within
the course of the employment.
o Issue: Is a blind person held to the same level of performance that a physically capable person would be
held to in a similar circumstance?
o Rule: A blind person cannot be required to do the impossible by conforming to physical standards that he
cannot meet. The conduct of the handicapped individual must be reasonable in the light of his knowledge
of his infirmity. He must take the precautions that the ordinary reasonable man would take if he were
blind.
o Holding: Judgment for because it is not uncommon for blind people to rely on other techniques when
moving around in a familiar setting. Also, if the operator is in a relatively busy area, the cane can be more
of a hazard than an asset. admitted that he was not using cane, explaining that he relies on his facial
sense, which he feels is an adequate technique for short trips inside the familiar building.
o Discussion: Burden is to use the cane. Blind people might have to be more careful, but not more
reasonable. Everything is subjective except judgment (did you make a good societal judgment)? Did he
act reasonably under the circumstances (one of the circumstances is that he cannot see)? Court did not
want to rule against , which would cause the precedent that blind people have the burden of having to
use a cane whenever they walk.
Physical Disabilities
o A person may have to be more careful due to his disability, but he is not held to a higher standard of care.
o Negligence is determined by what is reasonable conduct under the circumstances and one fo the
circumstance that must be taken into account is the physical disability.
o In a situation where there is a sudden incapacitation, it depends if the condition is one that the actor could
have foreseen if so, then negligence.
Mental Illness
o The law must make a hard choice as to whether it wishes to hold him liable for failing to meet normal
societal standards. Case law gives no quarter for mental disability.
Law concerning the liability of the mentally ill appears unusually harsh, in not providing an exception
for injuries caused by the sudden onset of a mental illness without prior notice.
The problem is that they perceive risks differently cant act as a reasonable person would!
Contributorily Negligent
o When the defendant was a competent negligent wrongdoer and the mentally ill plaintiff failed to act
reasonably with regard to his own safety, there is good reason to allow him to recover.
Courts are split on the issue.
6. To What Standard of Conduct Is a Child Held?
General Concept
o Child is held to a standard of care based on a test: age, intelligence, experience goes to the data; their
judgment is far below the standard of a reasonable adult.
o Data and judgment are the two elements considered did this child act as a child with like age and
experience would?
Restatement (Third) 10. Children
o When the actor is a child, the actors conduct is negligent if it does not conform to that of a reasonably
careful person of the same age, intelligence, and experience
Does not apply when the child is engaging in a dangerous activity that is characteristically undertaken
by adults.
o
Stevens v. Veenstra
o Facts: was a 14-year-old genius who was enrolled in a drivers education course so he could drive to
college; on his first day ever driving in the course, he made a sharp right hand turn, and after he and his
instructor were unable to steer clear of the , he struck him as he exited his car.
o Issue: Is a drivers education program an adult activity, which would warrant a minor to be treated with
the same standard of care as an adult?
In other words if Drivers Ed. is adult = same standard of care
o Rule: A minor who engages in an adult activity that is dangerous, like driving a care, is charged with the
same standard of conduct as an adult.
o Holding: Judgment for because some activities are so dangerous that the risk must be borne by the
beginner rather than the innocent victims, and lack of competence is no excuse. We believe that driving
an automobile is such an activity, and that anyone driving an automobile, regardless of age, must be held
to the same standard of competence and conduct. Because of the frequency and sometimes catastrophic
results of automobile accidents, it would be unfair to the public to permit a minor operating an automobile
to observe any standard of care other than that expected of all others operating automobiles.
7. Standard of Care for Professionals
Restatement (Second) 299A. Undertaking in Profession of Trade
o Unless he represents that he has a greater or less skill or knowledge, one who undertakes to render
services in the practice of a profession or trade is required to exercise the skill and knowledge normally
possessed by members of that profession or trade in good standing in similar communities.
The should have known is taken for granted. Having held herself out as a professional, she has a clear
obligation to acquire the requisite knowledge.
Did you exercise reasonable judgment, given the risk data that you know what reasonable doctors know?
Problems with professional malpractice cases:
o Juries cannot rely on common experience to guide them as to what is reasonable conduct;
o The judge will not give the jury a simple reasonable person instruction but will instead ask the jury
whether the defendant acted with the skill and knowledge normal to the profession
o Medical profession have developed some very special rules
Boyce v. Brown
o Facts: performed an operation to reduce a fracture in the ankle of ; permanently fixed the bone into
place by means of a metal screw; 7 years later consulted regarding serious pain to the ankle again,
which he treated with a wrap and filed the edge of her arch support, which he had made for her years
ago; 2 years later she returned again with pain; after this visit she consulted another physician, Dr. Kent.
Dr. Kent performed an x-ray, which showed there had been necrosis of the bone (death to cells)
around the screw; he removed the screw, and she had a normal recovery and has full use of ankle
now.
o Issue: Was the negligent for not taking an x-ray in 1934? Can a person be convicted of medical
malpractice without competent expert testimony against his actions?
o Rule: General rules of law governing actions of malpractice:
One who practices is presumed to have the skill and learning that is possessed by the average
member of the profession;
He must have done something in the course of his treatment that the recognized standard of good
medical practice in the country would forbid;
The standard of medical practice must be shown by affirmative evidence, or a jury may not be
permitted to speculate as to what the required standard is;
Negligence must be affirmatively proven, and not based on the mere fact that a treatment was
unsuccessful;
Reason of departure from the proper standard of practice must be established by an expert witness,
unless the negligence is so apparent that a layman would have no difficulty recognizing it;
It must appear that the course of treatment deviated from the methods used by the standard of the
nation testimony of other physicians is not enough, it must be the whole community.
The dr who did the surgery says he wouldve taken an xray in 1934.. court says not enough, needs to
say bad not to/its custom to take xray.
In many cases can make out negligence case but is there any harm?
In medical, custom evidence is binding. Unless you can prove that doctor violated medical custom,
doctor is going to walk. Special rule for medical profession primarily b/c dont want juries second
guessing widespread medical custom.
o Holding: Insufficient evidence in the record to show that was guilty of malpractice. The testimony of Dr.
Kent revealed that the method to unite the bone was a standard one, and that the screw is not normally
removed unless it makes trouble he never said that the methods deviated from the normal course of
treatment.
Courts universally dont recognize clergy malpractice in advice dont even know what standard to go with. No cause of
action with clergy malpractice. Cut this thing off at the duty stage.
0 worst standard of care, 10 best, if conduct is 2 verdict for plaintiff. Clearly inadequate&clearly negligent. Above an 8
court can direct a verdict and say clearly standard of care has been met no ? def is not negligent. Juries find facts and
then impose standard of care. Court can direct a verdict on either of those ?s reasonable jury could not find facts this
way, had to be applying wrong standard couldnt support this/prejudice/bias. Or a court can say the standard is out of
whack D acted ok and agreed set of facts we find the jury negligence is untenable. Between 2-8 goes to jury and jury
sets standard. When courts direct verdicts they do so under facts of case sometimes they speak broadly though.
(railroad get out of car and look example and case came up where he didnt do that said contributory negligence broad
language was bad, courts have to be careful.)
D. Judicially Determined Standards of Care
1. Courts Utilizing Risk-Utility Balancing to Set Standards of Care
Helling v. Carey
o Facts: is suing for medical malpractice; fitted for contact lenses for nearsightedness in 1959; from
1963-69 had complaints about irritation, which reasoned to be a result of the contacts; it was in 1969
that first tested for glaucoma, which is done by a eye pressure and field of vision test; tested positive,
and lost some vision over the time that she wasnt tested for this; didnt test because it is not common in
people under 40, and was 32, so did not consider it a possibility and therefore neglected to perform
the test.
Expert testimony the standards of the profession of that specialty in the same or similar
circumstances do not require routine pressure tests for glaucoma upon patients under 40 years of
age.
However, standards of the profession do require pressure tests if the patients complaints and
symptoms reveal to the physician that glaucoma should be suspected.
o Issue: Whether s compliance with the standard of the profession of ophthalmology, which does not
require the giving of a routine pressure test to persons under 40 years of age, should insulate them from
liability under the facts in this case where the plaintiff has lost a substantial amount of her vision due to
the failure of the s to timely give the pressure test to the .
o Holding: It is the duty of the courts to say what is required to protect patients under 40 from the
damaging results of glaucoma. That one person is entitled to the same protection, as afforded to persons
over 40, essential for timely detection of the evidence of glaucoma where it can be arrested to avoid the
grave and devastating result of this disease. The test is a simple pressure test, relatively inexpensive.
Pretty much says that regardless of how rare the disease is for people this age, you shouldnt be
excluded from tests simply because it is rare and you are entitled to the same treatment as an older
person, especially since it is so cheap and easy to perform.
Discussion: The court did not defer to medical custom as controlling in a case alleging medical malpractice.
It established a rule of law governing malpractice without expert testimony making the courts position the
reasonable one for the medical profession, not the experts.
o Thus, courts can judicially set the standard of care when they conclude that reasonable persons cannot
differ as to what are the appropriate risk-utility tradeoffs.
What happens when P can establish that D violated a statutory standard?
By and large courts will direct verdict on standard of care, not on causation. The whole reason for giving it to a jury from 28 is for a community standard. Administratively simple way of setting standard of care.
It is no defense to a negligence per se case that you acted reasonably. In other words, dont come back and tell me that
you used the violins and that I get off the hook because I acted reasonably b/c if in fact we decide to use statutory
standard (most of the time we will) a judge is directing a verdict; judge says sorry, not listening to violins, directing verdict
on standard of care. Even though statute wasnt written for civil liability, we are (the judge) imports the statute and say
standard of care here, using it, and directing a verict for it. Causation question is separate ?... ultimately have to get to 2 nd
step.
2. Court Utilizing Statutes to Set Standards of Care: Negligence Per Se
General Concept
o The party whose conduct falls below the statutory standard of care is held to be negligent per se. The
issue of what constitutes reasonable conduct is not for the jury to decide.
Courts have a vital role to play in deciding when it is appropriate to utilize the statutory standard as the
tort standard of reasonable care.
Restatement (Third) of Torts
o 14. Statutory Violations as Negligence Per Se
An actor is negligent if, without excuse, the actor violates a statute that is designed to protect against
the type of accident that actors conduct causes, and if the accident victim is within the class of
persons the statute is designed to protect.
o 14 does not say that you are negligent there is no tort law here, simply statutory law.
A judge does not have to rule by the statute.
By standard of care a trial judge, in torts, directs the verdict not the statute.
o Courts should not only regard the actors statutory violation as evidence admissible against the actor, but
also should treat that violation as actually determining the actors negligence.
o Negligence per se: an unexcused violation of the statute
o Illustration
A statute exists that states you are negligent if you do not put a warning sign 100 feet from your truck
if it is obstructing the road; Carl puts the sign right behind his truck; Ann hits the truck and gets hurt
because she didnt see it in time.
Carls violation of the statute is negligence per se.
In the absence of the statute, his failure to place a warning sign at least 100 feet away would
merely raise a jury question as to Carls possible negligence.
Discussion
What if the flare were set up 50 feet up, instead of the 0 feet?
would argue that he used discretion to determine the reasonable distance.
still needs to prove that if the flare were set at 100 they would have been able to stop
Martin v. Herzog (1920)
o Facts: and husband were driving in a buggy around a curve, when they were struck by s vehicle
coming in the opposite direction; husband dies. Negligence charged against because he did not keep to
the right of the center of the highway; charged against for traveling without lights.
o Issue: Does the violation of a statute, creating negligence per se, trump or negate the negligence of the
other party?
o Rule: Highway Law, 329a
Lights are intended for the guidance and protection of other travelers on the highway.
Violation of this is negligence.
o Holding: Judgment for because the evidence of a collision occurring more than an hour after sundown
between a car and an unseen buggy, proceeding without lights, is evidence from what a causal
connection may be inferred between the collision and the lack of signals. If nothing else is shown to break
the connection, we have a cause of negligence contributing to the result.
A statute designed for the protection of human life is not to be brushed aside.
o Discussion: Cardozo was mad that the trial court charged the jury to consider that there was a statute
saying that s lights should have been on that was negligence in itself. Burden of rebutting evidence
shifts to plaintiff. Lower court judge said driving without a light is not negligence itself but just consider it.
Cardozo says it IS negligence in itself to not have the light on plaintiff cant rebut it. The statute is clear
about standard of care. Unless theres something called justified excuse for violating statute (driving over
speed limit to get heart attack to hospital) we will direct a verdict. Even if direct verdict, then issue of
causation. Generally goes to jury but here so clear. Therefore, nothing to send back to trial contributory
negligence as matter of law.
You are required to direct a verdict on standard of care there is no question of fact!
A jury should not have been given the case to decide.
Even if there is a violation of statute, causation must be proven Cardozo says this is a giveaway
because was contributorily negligent.
Violation of a Statute as Negligence Per Se
o The strong majority view follows Martin, that violation of a statute is negligence per se.
o Judge decides whether the statute is applicable and then directs a verdict.
o The only role for the jury is to decide whether the actor did, in fact, violate the statute and whether the
negligence had a causal relation to the harm suffered.
Violation of a Statute as Evidence of Negligence
o A fair number of states take the position that violation of statute is some evidence of negligence.
o The jury hears evidence of the allegedly negligent conduct and takes the statute into account.
o
Licensing statutes create a problem. Move from Penn to NY and get into accident sued for not having NY license.
Shouldnt use licensing statute because it doesnt set a standard of care. P172
Brown v Shyne 151 NE 197 p172 court reversed judg for plaintiff holding proper formulation of general standards of
preliminary education and proper examination of the particular applicant should serve to raise the standards of skill and
care generally possessed by members of the profession in this State but the license to practice medicine confers no
additional skill upon the practitioner. Unless you can prove failure to have knowledge caused harm this didnt sit well
with legislature who came up with statute p173- practicing without license shall be pf evidence of negligence. If you are
practicing as a doctor in NY without the license we will shift burden onto you to produce sufficient evidence of nonnegligence to put presumption in doubt once presumption is put in doubt then plaintiff has to prove the case.
If you comply with a statute, it becomes the minimum standard of care. Relevant but not binding. Sometimes fed statutes
are preemptions (dictate that this is the standard of care and a tort action cannot be brought on common law standard of
care)
There are certain cases of which it may be said res ipsa loquitur and this seems one of them. In some
cases the court has held that the mere fact of the accident having occurred is evidence of negligence.
Requirements
o There must be reasonable evidence of negligence. But where the thing is shown to be under the
management of the defendant or his servants, and the accident is such as in the ordinary course of things
does not happen if those who have the management use proper care, it affords reasonable evidence, in
the absence of explanation by the defendants, that the accident arose from want of care.
In res ipsa cases we have nothing more than a generalization. We admit we dont know what went wrong, but
are willing to conclude that whatever went wrong was more likely than not the result of negligent conduct.
Plane crashes dont know what went wrong, but more of explanations are negligent related. Courts are
uncomfortable and rightfully so with these cases. Talking about generalization.
Issue
o Whether there is sufficient evidence to support the proposition that the negligence that caused the harm
points to the defendant. Where the defendant can point to an alternative cause that is equally probable to
have been the cause of the harm, the judge cannot submit the res ipsa case to the jury.
o
Holding: Judgment for plaintiff. Factor one is satisfied because the unexplained departure of a car from the
roadway ordinarily bespeaks negligence; Factor two is satisfied because once the jury found that had been
the driver, it could logically have found that she had been in exclusive control of the car; Factor three is
satisfied because it is deemed irrelevant because of factor two.
Defense from Res Ipsa: We issue res ipsa when there is no evidence, and 9/10 times youre going to win.
The key is preventing the case from going to the jury based on the fact that nobody proved anything. As a
defendant in cases like this, how do you beat it?
o Introduce another idea that could be a probable explanation.
An alternative cause for which there is some evidence.
o You dont have to prove your case, but muddy the plaintiffs case by creating doubt that a generalization
was at work.
o Beating it requires getting the judge to not give the case to the jury you want him to dismiss the matter
because of lack of evidence.
3. Ybarra v. Spangard
Facts: is suing multiple , who are all doctors and nurses that were present during an appendectomy on
him; when he awake the following morning he had pain between his neck and right shoulder that was severe;
the pain persisted for 5 months before spreading down to his lower arm; he finally returned to work, but wore
a splint; an x-ray was taken by another physician and it was determined that the injury was a paralysis of
traumatic origin, not arising from pathological causes, and that the injury resulted in atrophy, loss of use and
restriction of motion of the right arm and shoulder.
o Defendants argue that where there are several defendants, and there is division of responsibility in the
use of an instrumentality causing the injury, and the injury might have resulted from the separate act of
either one of two or more persons, the rule of res ipsa cannot be invoked; and that where there are
several instrumentalities, and no showing is made as to which caused the injury or as to the particular
defendant in control of it, the doctrine cannot apply.
Issue: Whether res ipsa can be applied when there is no one particular defendant that can be accused with
certainty because the injured was suffered when the plaintiff was unconscious.
Holding: Judgment for plaintiff. Where a plaintiff received unusual injuries while unconscious and in the
course of medical treatment, all those defendants who had any control over his body or the instrumentalities
that might have caused the injuries may properly be called upon to meet the inference of negligence by giving
an explanation of their conduct. It should be enough that the plaintiff can show an injury resulting from an
external force applied while he lay unconscious in the hospital; this is a clear case of identification of the
instrumentality as the plaintiff may ever be able to make.
Note: This case goes against the norm, and most courts place the burden of proving liability against each
defendant squarely on the plaintiff. Court said that in this sort of case where the plaintiff is unconscious, we
would change the burden of initial explanation the burden of coming forward with the evidence.
4. Sullivan v. Crabtree
Facts: is a passenger in a motor truck operated by ; when the truck was approaching a curve, another
truck passed it, and just after, s truck suddenly swerved and ran off the left shoulder, overturned down a
steep embankment, killing ; was unable to determine what caused the swerve, claiming it could be loose
gravel, poor pavement, or brake malfunction. Goes to jury Trial court ruled for the defendant.
Issue: Whether res ipsa should be applied when no explanation by the defendant for the cause of the
accident is given.
Holding: Judgment affirmed for defendant. The court found that the application of res ipsa loquitur only
allowed the jury to infer the negligence of the appellee defendant in preference to other permissible or
reasonable inferences. The court further found it was up to the jury to choose the inference they thought most
probable and that there was sufficient evidence for the jury to find in favor of defendant. The court also found
no other assignments of error presented by plaintiffs that could be considered and affirmed the judgment for
defendant.
o won a trial by jury before the appeal, probably because he brought up all kinds of possibilities why it
could have happened to dispel the claims. Also, he is a trucker and knows the proper way to drive,
meaning that the jury probably took into account his skill and ability to control a truck.
Inferences: Normally there is an inference when the case goes to the jury. Ways to treat the inference:
o Presumption shifts the burden of coming forward with the evidence to the defendant (he must either put
up or shut up and put the presumption in doubt).
Defendant must create doubt that the presumption is true.
If he cannot do that, the plaintiff will get a directed verdict.
o Not a presumption, its an Inference goes to the jury. Law in NY is res ipsa is inference instead of
presumption. Overwhelming majority of states treat as inference. Sometimes its case by case; if you find
a toe in coca cola, if im trial judge toes do not belong in coca cola bottles, and unless D shows with proof
that elf put it into the bottle, going to direct a verdict. Some res ipsa cases you cant use inference.
Chapter Four: Actual Causation
A. But-for Causation: Did the Defendants Negligent Conduct Cause the Plaintiffs Harm?
1. General
The section discusses quantity of proof regarding the factual circumstances surrounding the accident. The
issue for this section is
o Whether the negligent aspect of the defendants conduct contributed to causing plaintiffs harm.
2. Doors to pass through to prove liability:
1 Negligence
2 But for
3 Proximate cause
3. Perkins v. Texas and New Orleans Ry.
Facts: was riding in the passenger side of a car; is the train company who hit s car, killing him; the train
and car were approaching the same intersection, but were riding perpendicularly toward it, on a collision
course; crewmen spotted the automobile, but the train could not stop in time, and hit and dragged the car
1250 feet until it stopped; both people in the car were dead; s train was traveling at 37 mph, and the speed
limit was 25 mph in that zone (created by the train company); the speed of s car was unable to be
determined; there was a warehouse that obstructed the view of both the driver of the car and train. Both
parties concede that the driver of the automobile was negligent for being on the tracks; both parties concede
that the train was traveling at 12 mph over the speed limit.
o Plaintiffs argue an Escape theory
The automobile would have cleared the track and evaded the impact had the train been moving at a
proper speed at the time the trainmen observed the automobile emerge from behind the warehouse.
Problem: the speed of the automobile had not been definitely established.
The collision and loss of life might not have occurred if the train was traveling at the required speed.
Issue: Whether the excessive speed of the train was the cause in fact of the fatal collision (if the train were
moving at 25mph would the plaintiff be dead?). If P cant show that @ 25mph same thing wouldnt have
happened, speeding is not the cause.
Rule: It must appear that it is more likely than not that the harm would have been averted but for the
negligence of the defendant. Burden of proving the link is upon . It is required by law for the evidence to
show that it is more probable than not that the harm was caused by the tortious conduct of .
o Negligence is not actionable unless it is a cause in fact of the harm for which recovery is sought.
Negligence is a cause in fact of the harm to another if it was a substantial factor in bringing about that
harm.
Applied: The excessive speed was undoubtedly a substantial factor in bringing about the collision if
the collision would not have occurred without it. On the other hand, if the collision would have
occurred irrespective of such negligence, then it was not a substantial factor.
Holding: Judgment for , as has failed to discharge the burden of proving that the negligence of was a
cause in fact of the tragic death. Escape theory is rejected because of the deficiencies in the evidence that we
have already noted, making it devoid of evidentiary support. The record contains no probative facts from
which the court can draw a reasonable inference of causation under this theory.
Note: When the plaintiffs harm would have occurred even if the defendant had not acted negligently, then the
defendants negligence did not legally cause the plaintiffs harm. It is often said that the defendants negligent
conduct must be a but-for cause of the plaintiffs harm.
o Remember to always determine if the plaintiff can prove a prima facie case against the defendant. It will
make everything much easier when evaluating the steps of proving liability.
o Defendant will always argue that the plaintiff has not made a prima facie case and that negligence is not a
but-for cause of harm.
o Burden of proof is on the plaintiff to show that the defendants negligence made a difference. It is not
enough for you to prove that it might have made a difference.
o You pretty much create a hypothetical in which you make the defendant doing the right thing, and if the
same result would occur regardless, you cannot prove that his negligence was the cause of harm. For
example, if you hypothesize that the train was traveling the speed limit, you then have to ask if the driver
of the car would still have been hit and killed. Would it have happened then too? If you cant prove this,
then the burden of proof is not satisfied.
But for causation is a screen that lets through cases that we might want to kill but need another construct
to kill isnt going to be but for causation.
Article cited on p206 on cause in fact- cause in fact is only partially a hypo but-for scientific question.
Clearly when it can be determined that Ds negligence is not the cause in fact. Its true in general we put
burden of proof on the plaintiff (more probably than not) but says this is not math, there is policy here too
we are trying to figure out whether I have enough causation to impose liability. Trying to figure out whether
there is enough causation to impose liability. In Reynolds, negligent towards very type of people who light is
intended for.
Purpose: To determine whether those experts proposed testimony amounts to scientific knowledge,
constitutes good science, and was derived by the scientific method.
o Findings: That an expert testifies based on research he has conducted independent of the litigation
provides important, objective proof that the research comports with the dictates of good science.
Means that expert testimony was objective and within the standard of being derived by the
scientific method.
o Holding: First prong has been satisfied.
Second Prong
o Purpose: To determine whether the testimony will assist the trier of fact in resolving the factual issue to
which it purports to relate.
o Findings: s must establish not just that their mothers ingestion of Bendectin increased somewhat the
likelihood of birth defects, but that it more than doubled it only then can it be said that Bendectin is more
likely than not the source of their injury.
Thus, s cannot carry their burden of proving that their injuries were the result of the accused cause
and not some independent factor.
o Holding: Second prong has not been satisfied.
7. Herskovits v. Group Health Cooperative of Puget Sound
Facts: This is an appeal arising out of estates claim alleging medical malpractice against ; decedent first
went to for chest pains in early 1974, and cough medicine was prescribed; after the problem persisted for
18 months, went to another doctor who recommended new tests, and it was discovered in July 1975 that
decedent had lung cancer; in March 1977 decedent died of same.
o Plaintiffs expert argues that if diagnosis were made in December 1974, decedents possibility of 5-year
survival was 39 percent; at the time of the diagnosis 6 months later, the possibility of 5- year survival was
reduced to 25 percent. Thus, the delay in diagnosis may have reduced the chance of a 5-year survival by
14 percent; this is a 36 percent reduction!
Issue: Whether a 36 percent reduction in chance of survival is sufficient evidence of causation to allow the
jury to consider the possibility that s failure to timely diagnose the illness was the proximate cause of
decedents death.
Rule: Case Law Hamil v. Bashline (1978)
o Court determined that the original survival chance of 50 percent was sufficient to send the case to the
jury.
o Once has demonstrated that s acts or omissions have increased the risk of harm to another, such
evidence furnishes a basis for the jury to make a determination as to whether such increased risk was in
turn a substantial factor in bringing about the resultant harm.
o Where percentage probabilities and decreased probabilities are submitted into evidence, there is simply
no danger of speculation on the part of the jury.
Holding: Judgment for plaintiff damages should be awarded to the injured party or his family based only on
damages caused directly by premature death (not the whole death, because he would have died soon
anyway). Plaintiff did not need to show that decedent's probability of survival was 51 percent, and plaintiff's
evidence of a reduced chance of survival was sufficient for jury determination of proximate cause.
Concurring: Causation shouldnt be all or nothing proposition. If you can identify what lost chance is and can
figure out what total damages wouldve been if had lived, you give 14%... the idea is to say that causation will
be proportioned based on the lost chance. He was 61 percent dead, and you reduced it, so give him either
the 36 percent reduction in chances, or 14 percent difference. What is 14 percent worth? Youre 100 percent
alive when you have those chances, and youre 0 percent alive when you dont. How can you value that?
What if he had a bad heart as well? How do we incorporate that into the cancer risk? Would we reduce the
recovery even more because of other health problems?
8. The Aftermath of Herskovits
The minority of courts refuses to allow recovery for loss-of-chance unless the plaintiff can establish causation
under the traditional negligence standard. In these cases, the plaintiff must show that the defendants failure
to diagnose or treat the plaintiff did, more likely than not, cause the plaintiffs harm.
A majority of jurisdictions allow loss-of-chance claims to reach the jury even when the plaintiff cannot prove
that the defendant was, more likely than not, the cause of the plaintiffs harm.
o Courts hold that it would be for a jury to decide whether the increase in risk due to the defendants
negligent failure to provide reasonable care was a substantial factor in causing the harm.
Another difference among the courts is the way they conceptualize the harm for which the plaintiffs seek to
recover. Some view the harm as the resulting disability or death, while others view the chance itself as a
valuable interest and allow recovery for its loss.
o
If causation is proven, courts that follow the traditional conception of the death or injury as the harm
typically leave the jury free to determine the recovery amount, allowing either full compensation or a
lesser amount to be awarded at their discretion.
o Other courts that recognize the chance, itself, as the loss provide a percentage-based formula for the jury
to use in calculating damages.
C. When Two (or More) Negligent Actors Concurrently (or Successively) Cause the Plaintiffs Harm
1. Hill v. Edmonds
Facts: appeals from trial courts dismissal; is a tractor truck driver that left the truck parked in the middle
of the road on a stormy night without lights; was a passenger in the car that hit the truck; trial court
concluded that the driver of plaintiff passengers car was negligent as well because it was enough car lengths
ahead that she could have avoided the truck.
Issue: Whether two actors can be tried for the same act that caused plaintiffs harm.
Rule: Where separate acts of negligence combine to produce directly a single injury, each tort-feasor is
responsible for the entire result, even though his act alone might not have caused it.
Holding: Judgment for plaintiff new trial with the complaint against the truck driver reinstated.
Indivisibility: Indivisibility arose from the circumstance that the negligent conduct of either defendant would
not, without the other, have caused the accident in which plaintiff suffered harm.
*dont tell me when I ask you whether youre the but for cause of the accident, yea but somebody else
is, there may be two but for causes- there can be 3 but for causes, the lighting may be bad, and 4, etc, etc,
as long as you ask the question but for your negligence would this accident have happened- if its no, youre
the but for cause of the accident. You dont get off the hook by pointing to somebody else as the but for cause
of the accident.
Under common law, they are both 100% responsible for the Ps harm since both but for cause in this
case.
Now we have to figure out how to divvy up the damages dont get off the hook by having other but for
causes.
2. Joint and Several Liability
Defendants who are jointly liable can be joined in a single action, although a plaintiff is not required to join
them.
Defendants who are severally liable are each liable in full for the plaintiffs damages, although the plaintiff is
entitled to only one recovery.
Two circumstances in which two or more actors acted tortiously toward their victim gave rise to what is now
referred to as joint and several liability
o When they acted in concert to cause the harm.
Vicarious liability, in which all the negligent actors will be responsible for the harm actually caused by
only one of them.
Example: when two persons engage in an automobile race on a public street and one of them
negligently runs over the plaintiff. The one who did not run over the plaintiff is just as liable as the
one who did.
o When they acted independently but caused indivisible harm.
Where it is impossible to allocate the harm to either defendants conduct.
3. Kingston v. Chicago & N.W. Ry. (fancy name- overdetermined causation- unlike intersection accident we talked
about before about speeding/negligent lookout each but for cause, in this case if you play but for causation
issue by the book neither Is the but for cause of harm)
4. Substantial factor- if Ds fire was substantial factor cause of the harm fires were joined and were therefore
substantial causes of the harm. The court substituted for but for w/ substantial factor.
Facts: A fire to the northeast of s property was started by a spark from s locomotive; jury found this to be
proximate cause of the destruction of property; a northwest fire, whose cause is unknown, was also found to
be the proximate cause by the jury; the two fires united about 940 feet north of s property; therefore, we
have two separate agencies, each of which constituted the proximate cause of the damage.
Issue: Whether the railroad company, which is found to have been responsible for the origin of the northeast
fire, escapes liability, because the origin of the northwest fire is not identified, although there is no reason to
believe that it had any other than human origin.
Rule: Any one of two or more joint tortfeasors, or one of two or more wrongdoers whose concurring acts of
negligence result in injury, are each individually responsible for the entire damage resulting from their joint or
concurrent acts of negligence.
Holding: Judgment for plaintiff. Where one who has suffered damage by fire proves the origin of a fire and
the course of that fire up to the point of the destruction of his property, one has certainly established liability on
o
the part of the originator of the fire. It is impossible to apportion the damages or to say that either perpetrated
any distinct injury that can be separated from the whole.
o If the defendant claims that the uniting fire was made by nature or was of a larger proportion, the burden
is on him to prove that by reason of such union, the fire set by him was not the proximate cause of the
damage. does not do this in this case, and it appears that the northeast fire, for the origin of which is
responsible, was a proximate cause of s loss is responsible for the entire amount of that loss.
5. Restatements
431 An actors negligent conduct is a legal cause of harm to another if his conduct is a substantial factor in
bringing about the harm.
432(2) If two forces are actively operating, one because of the actors negligence, the other not because of
any misconduct on his part, and each of itself is sufficient to bring about harm to another, the actors
negligence may be found to be a substantial factor in bringing it about.
Restatement 2nd of Torts used substantial factor in bringing about the harm
Restatement 3rd used the but for test, but added 27 multiple sufficient cases: if multiple acts exist, each of which alone
would have been a factual cause under 26 of the physical harm at the same time, each act is regarded as a factual
cause of the harm. P232 dont get off the hook
the restatement has wisely rid itself of the substantial factor test except for the case where the two tortfeasors were
independently capable of causing the Ps harm, the substantial factor test caused confusion. P232
D. When One of Several Negligent Actors Clearly Harmed the Plaintiff, But We Cant Tell Which One
1. Summers v. Tice
Facts: was out with two s hunting quail in an open range; they were situated in a triangle and all shot at a
quail that was closest to ; was struck by both s once in the eye and once in the lip. Ds were not acting
in concert, if they were then they would be co-conspirators (dragracer example)
Issue: Whether s are liable for causing harm when there is no evidence that shows which of the two caused
the damage.
Rule: When two or more persons by their acts are possibly the sole cause of a harm, or when two or more
acts of the same person are possibly the sole cause, and the plaintiff has introduced evidence that the one of
the two persons, or the one of the same persons two acts, is culpable, then the defendant has the burden of
proving that the other person, or his other act, was the sole cause of the harm.
o Cant deny the injured person redress simply because he cannot prove how much damage each did,
when it is certain that between them they did all of it.
Holding: Judgment for plaintiff. It is apparent that s in cases like the present one may be treated as liable on
the same basis as joint tortfeasors.
o This theory condones imposing liability on actors who we know did not actually cause the plaintiffs harm.
We hold both hunters liable in Summers, even though we know one of them did not do it, because
otherwise we would let the one who did it off the hook. However, this is why the burden of proof is shifted
to the defendants. The plaintiff can prove with certainty that one of the two defendants was responsible,
so in order to avoid injustice, the defendants must figure it out on their own.
2. Sindell v. Abbott Laboratories
Facts: alleges that her mother took DES during pregnancy to prevent miscarriage, which caused to
develop a bladder tumor and runs the risk of having further malignancy; cannot provide proof as to which
produced the DES used by her mother, or even that the manufacturer of her mothers DES was a defendant
in this action; 5 companies are the defendants here, because together they comprise 90% of the total market
for DES; the trial court dismissed the complaint, and appealed.
Issue: Whether a claim can be brought against multiple defendants who comprise 90 percent of the market
that is responsible for the drug that led to tumors, and it is unknown which, if any, of the defendants are
responsible.
Plaintiffs Claims: The plaintiff makes three claims to introduce this cause of action on multiple defendants:
o #1: Alternative liability if a party cannot identify which of two or more defendants caused an injury, the
burden of proof may shift to defendants to show that they were not responsible for the harm. This theory
relies on Summers.
Courts Response #1: The possibility that any of the five defendants supplied the DES to plaintiffs
mother is so remote that it would be unfair to require each defendant to exonerate itself.
This rule cannot apply to relieve the plaintiff of the burden of proof
Twerski isnt convinced. Says a case can be made here for an even stronger case for shifting
burden of proof to Ds failure to test here was they didnt do generational testing on mice that
theyd be able to tell cancer causing properties once you know the result, its pretty easy to go
back and find out what you shouldve done- be that as it may, in all of these cases all of the Ds are
negligent so have problem of showing causation.
In summers one guy hit one missed, in this case, everybody hit. If you sold DES you caused
cancer. I just dont know whose cancer you caused (Twerski) problem with using the
summers rationale here (court refused to do summers) if you treat them all as joint
tortfeasors, then any D is liable for 100% of damages and didnt want this to happen. This
shift burden of proof to D would mean if they couldnt prove it wasnt them were 100% liable.
Defendants Response #1: Important to understand that there are approximately 200 drug companies
in this case that made DES, but only 2 possible shooters in Summers. There was a 50 percent
chance there, while here one of 200 companies may have made the product that harmed the plaintiff.
There is no rational basis upon which to infer than any defendant in this action caused plaintiffs
injuries, nor even a reasonable possibility that they were responsible.
o #2: Concert of Actions s failed to adequately test the drug or to give sufficient warning of its dangers
and that they relied upon the tests performed by one another and took advantage of each others
promotional and marketing techniques.
Courts Response #2: These allegations do not amount to a charge that there was a tacit
understanding or a common plan among defendants to fail to conduct adequate tests or give
sufficient warnings, and that they substantially aided and encouraged one another in these omissions.
There is no allegation here that each defendant knew the other defendants conduct was tortious
toward plaintiff, and that they assisted and encouraged one another to inadequately test DES and to
provide adequate warnings.
o #3: Enterprise Liability the practice of the industry of omitting a warning on individual blasting caps and
of failing to take other safety measures created an unreasonable risk of harm, resulting in the plaintiffs
injuries. The complaint did not identify a particular manufacturer of a cap that caused a particular injury.
This theory relies on Hall, in which they will try to show that if the caps were manufactured by one of the
defendants, the burden will shift to the defendants to prove which one is responsible.
Courts Response #3: Cant apply the rationale to this case because there are 200 manufacturers.
Also, the FDA does exist and sets standards for these circumstances. Thus, the manufacturers must
adhere to these standards, and cannot be held at fault for complying with the FDA.
Holding/New Rule: Judgment for plaintiff, with the creation of a new standard:
o Measure the likelihood that any of the defendants supplied the product that allegedly injured plaintiff by
the percentage that the DES sold by each of them for the purpose of preventing miscarriage bears to the
entire production of the drug sold by all for that purpose. Market share theory- The 5 defendants account
for 90 percent of the manufacturers. [you can prove yourself out of the case if didnt sell to that region of
the country or that pharmacy in California; NY doesnt allow this but most states do Twerski agrees
w/NY, if you cant prove yourself out were using the National market] NY said- 1 national market 2
you cant prove yourself out 3 no joint/several liability
o Attempts to push market share past this have almost always failed. Asbestos example. Market share
on breaks, 100 diff types, some give off asbestos some dont, too difficult to figure out.
o How do you define the market? NYS, Brooklyn, Eastern US, US as a whole if you want to define it
accurately Twerski says youd define it as jones pharmacy.
o Other ?: lets assume I have 50% of the market, should we have joint or several liability? Should 5% D be
treated as joint tortfeasor and be liable for 100% b/c P has 100% loss and Ds caused harm and cant
figure out who did what. In Restatement, say no grounds for doubling percentage (joint/several liability)- in
this case, dont know that anybody is the but for cause all we know is they caused 5% cancer in the US.
If 50% of market isnt there then P tough luck. Nothing to support a 5% D paying 10%.
o If plaintiff joins in the action the manufacturers of a substantial share of the DES that her mother might
have taken, the injustice of shifting the burden of proof to defendants to demonstrate that they could not
have made the substance that injured plaintiff is significantly diminished.
o Each defendant would be held liable for the proportion of the judgment represented by its share of that
market unless it demonstrates that it could not have made the product that caused plaintiffs injuries.
o Under this approach, each manufacturers liability would approximate its responsibility for the injuries
caused by its own products.
Dissent: The effect of its holding is to guarantee that plaintiffs will prevail on the causation issue because
defendants are no more capable of disproving factual causation than plaintiffs are of proving it. A mere
possibility of such causation is not enough; and when the matter remains one of pure speculation or
conjecture, or the probabilities are at best evenly balances, it becomes the duty of the court to direct a verdict
for the defendant. Majoritys holding makes a defendant fair game if it happens to be engaged in a similar
business and causation is possible, even though remote.
Notes: This case was a stronger case for shifting burden of proof to defendant than Summers. Each of the
defendants hit somebody. Market should be defined more narrowly when possible. But, there will be huge
evidentiary problems in defining the market because you have to look retrospectively over time.
o
DES was easy for this because it was a standard drug, given for one purpose, over a short period of time.
It made pinpointing the boundaries of the market share easy to determine.
Example: asbestos in car brakes gives auto mechanics various types of cancers. Difficult to
determine because you are not certain of the types of brakes, what conditions the workers were in,
etc. You are flipping a coin to determine the market share here because the plaintiffs cant point a
finger at anyone in particular even though they are all negligent in using asbestos in their products.
DES was easy because of the above bullet, but this is much more complex (what percentage of
asbestos related diseases did you cause impossible to determine).
Lead paint is a similar problem, because it has been sold for over 70 years how can you possibly
determine market share with such a broad sample?! However, if you dont do it, plaintiff walks away
with no recovery even though it is a clear case of negligence.
2.
3.
4.
5.
What if Ds conduct was intentional- how wide is the scope of liability? Talmage doctrine of transferred
intent hit the kid who was unforeseeable but his conduct was intentional and we held him liable for an
unforeseeable plaintiff. The worse the conduct of D is, the less chance well get them off the hook w/ regard to
proximate cause.
Why be especially careful in negligence with how far we impose liability? Partially because
disproportionately burden, the risk created by your negligent conduct- if entire theory is based on risks
created by your negligence, if risk isnt created by negligence, ought not impose liability.
Negligence always refers back to actor. All I can do w/ negligence is hold you accountable for information you
had or didnt have. What kind of risks were created by negligent act?
Bundle of Risks: When the truck did what it did it created a bundle of risks that were possible outcomes of its
actions; this means that negligence exists. Does this particular risk belong in that bundle? We now have a
liability limiting doctrine that says go back and look at the negligent conduct and ask if this risk fits the bundle,
and not if this risk alone is sufficient.
@ end of the case- why isnt this the learned hand formula all over again? Foreseeability is the probability
you say the risk has to be foreseeable, when we deal with the BPL we are not finetuning the question.
Regarding motor vehicle accidents one should contemplate a variety of risks- bundle of risks when ask the
BPL question, asking a very blunt question, did you act reasonably? If posed that in this case the P is so low
you could say no, P isnt negligent for that one risk. The risk is so slight. We never ask the ? that way, we
ask were you negligent and its undifferentiated (all the terrible things that could happen) and when we get to
prox cause have to ask ? for the first time, whether or not this risk was in the bundle in other words, if I were
to create a worry list- tell you be careful, accidents are messy- is this part of the risk? Diff than asking are you
negligent for this risk alone. We dont ask that. There is no litmus test but ultimately you have to confront the ?
of what the scope of the risks are and that is the restatement test
Twerski Aside: if we find that something is not the prox cause of the injury, liability is choked off because PC is
part of PFC. If we find there is PC, I still may have an issue of plaintiff fault in terms of whether the P acted
reasonably or not. And then- I could give the issue to the jury on the issue of comparative fault. [how at fault was
P how at fault was D] as a practical matter, the influence of comparative fault has been to lessen the importance
of prox cause and amount of time courts are going to direct a verdict. Court says is it within scope of risk, its
close Ill let it go to the jury.
McCahill v. New York Transportation Co.
Facts: s intestate was struck by an automobile driven by an employee of while he was walking on
Broadway; the injuries precipitated, hastened, and developed delirium tremens, the these caused death;
was drunk at the time, which is said to be a factor in this condition occurring; a jury ruled in favor of .
Defendant claims that this would not have happened if plaintiff were not drunk.
Issue: Whether s negligence was the proximate cause of intestates death when his medical condition could
not have occurred without having been drunk.
Rule: A negligent person is responsible for the direct effects of his acts, even if more serious, in cases of the
sick and infirm as well as in those of healthy and robust people, and its application to present case is not
made less certain because the facts are somewhat unusual and the intestates prior disorder of a
discreditable character.
Holding: Judgment for plaintiff. One, who has negligently forwarded a diseased condition, and thereby
hastened and prematurely caused death, cannot escape responsibility, even though the disease probably
would have resulted in death at a later time without his agency. was an alcoholic and suffering from delirium
tremens. If this were the only reason to not drive carefully, then there is a problem who will you hit next?
That is why it doesnt matter that he has this illness as a result of his drinking habits. If the only people you hit
were healthy specimens we wouldnt need rules. A standard of care test takes care of everyone because of
this.
Thin Skulls and Fragile Psyches
The thin skull rule a tortfeasor must take his plaintiff as he finds him. You are not entitled to a healthy P.
Its not that I dont have to worry about idiosyncracies of possible P, dont have to think about them of any
individual person but do have to think about the fact that theres a world full of idiosyncracies Twerski argues
when we do pinpoint idiosyncracies we change standard of care (speed limit lowered around schools) able to
identify a particular group. We set our standard of care taking into account the whole range of human
conditions and once you hit somebody, you cant get off on the grounds you didnt foresee that particular P.
American courts unanimously recognize the thin skull rule when the unexpected consequence occurs
systemically, within the plaintiffs body, as the result of a physical injury to the plaintiffs person.
This does not bear on whether the defendant was negligent in the first place. A defendant need not take into
account the idiosyncrasies of possible plaintiffs when acting. She is entitled to assume that the people her
actions may harm are of ordinary capacity to resist injury.
Thin Skull and Property Interests
6.
7.
8.
9.
Should the rule apply when the plaintiffs harm takes the form of property damage rather than personal injury?
Polemis held that to determine whether an act is negligent, it is relevant to determine whether any reasonable
person would foresee that the act would cause damage; if he would not, the act is not negligent. But if the act
would or might probably cause damage, the fact that the damage it in fact causes is not the exact kind of
damage one would expect is immaterial, so long as the damage is in fact directly traceable to the negligent
act, and not due to the operation of independent causes having no connection with the negligent act, except
that they would not avoid its results. Once the act is negligent, the fact that is exact operation was not
foreseen is immaterial.
Polemis says that if you are liable for the direct cause of harm for one part of the harm, then you are liable for
all of the harm. It is like a thin skill rule for property damage.
Foreseeability
Is the risk a foreseeable risk within the bundle, or are you negligent for this risk alone?
o If it is for this risk alone, then you will not have a case for proximate cause.
Is it fair to take THIS result and reflect it back on THAT series of events?
Overseas Tankship (U.K.) Ltd. v. Morts Dock & Engineering Co. Ltd. (Wagon Mound No. 1)
Facts: The Wagon Mound, a vessel, spilled oil into the harbor; workers at the nearby wharf were using
acetylene torches for repairs; did not try to disperse the oil, and they also assured the workers that their
torches would not ignite it; wharf owners sued alleging that the spillage of oil was negligent because it was
foreseeable that the oil would cause the fire; argued that it was not foreseeable that this would happen; trial
court held that it was unforeseeable but because of the Polemis decision, was liable.
o Polemis stated that if is guilty of negligence, he is responsible for all the consequences whether
reasonably foreseeable or not. The decision of the lower courts in this case rested on the notion that the
outbreak of fire was the direct result of the negligent act.
o English version of Nugent- risk must be within range of foreseeability
Issue: Whether an actor should be held liable for unforeseen consequences of negligence.
Rule: A man should be responsible for the natural or necessary or probable consequences of his act when it
is judged by the standard of the reasonable man that he ought to have foreseen them.
Holding: Judgment for defendant. The oil is highly unlikely to be ignited by fire, and their personal experience
was enough to know that likelihood. They could have regarded it as a possibility, but one that would occur
only under exceptional circumstances. The occurrence of damage to the plaintiffs property as a result of the
spillage was not reasonably foreseeable by those for whose acts the defendant would be responsible.
Therefore, the claim for negligence fails.
Notes: In rejecting the Polemis holding, this court seems to be returning to the approach set in Marshall.
Unlike in Polemis, the thin skull rule does not come into play in this case because the loss of the docks was
not caused by any special susceptibility on the docks part. The property is not thin skull here. It is normal.
o The element of unforeseeability relates to whether spilling the oil would cause fire in the first instance.
Obviously they thought it wouldnt. However, once it did, it was obvious it would damage the wharf, but it
still wasnt foreseeable that the fire would begin in the first place.
o Plaintiffs in Wagon Mound 1 decide not to make an argument about foreseeability because it could come
back to hurt them (they could have been contributorily negligent because they were using the torches).
Making Close Foreseeability Calls
In cases where the unexpected (and typically most severe) harm to the plaintiffs person or property does not
flow systematically from the first tortious invasion of the plaintiffs interest, American courts generally employ
the Marshall foreseeability approach in working out proximate cause. Bundle or risks!
Courts treat as foreseeable any misfortunes that befall victims as a direct result of seeking medical care for
the injuries suffered in the accident caused by the defendants negligence.
o Example: Court held defendant liable for a helicopter crash in which the plaintiff died, in the course of
which she was getting airlifted to a hospital because of a car accident caused by defendant in which she
survived.
Negligence Per Se and Proximate Cause
Dangerous conduct that violates safety statutes constitutes negligence per se; courts recognize proximate
cause in negligence cases involving statutory violations.
o Example: Court held defendant not liable when the ship he owned did not have the proper pens to contain
the sheep he was transporting. When a storm hit, the sheep went overboard. However, Parliaments
intention for the statute that required the pens was to prevent the sheep from infecting each other with
disease. Thus, the lack of pens was not the proximate cause of their death, because the statute was
designed for a different purpose.
Rule: risk must be the sort of risk that statute was intended to protect against in order to have proximate
cause link between negligent conduct and plaintiffs harm (defining the bundle of risks based on intent of
legislature).
10. Overseas Tankship (U.K) Ltd. v. The Miller Steamship Co. (Wagon Mound No. 2)
In Wagon Mound 1 it was warf owner, in Wagon Mound 2 it was ships in the harbor- fire spread to those.
Facts: The same facts as in the above case, but here is suing for nuisance, not negligence.
Issue: Whether a reasonable man having the knowledge and experience expected of the chief engineer of
would have known that there was a real risk of the oil on the water catching fire in some way.
Rule: Case law: Bolton v. Stone The court held that the risk was so small that someone would get injured in
the way the plaintiff was injured that in the circumstances a reasonable man would have been justified in
disregarding it and taking no steps to eliminate it.
o It does not follow that, no matter what the circumstances may be, it is justifiable to neglect a risk of such a
small magnitude like in Bolton. A reasonable man would only neglect such a risk if he had some valid
reason for doing so (that it would involve considerable expense to eliminate the risk). He would weigh the
risk against the difficulty of eliminating it.
o What Bolton did was to recognize and give effect to the qualification that it is justifiable not to take steps
to eliminate a real risk if it is small and if the circumstances are such that a reasonable man, careful of the
safety of his neighbor, would think it right to neglect it.
Holding: Judgment for plaintiff. The evidence shows that the discharge of so much oil onto the water must
have taken a considerable time, and a vigilant ships engineer would have noticed the discharge at an early
stage. Findings show that he ought to have known that it is possible to ignite the kind of oil on water. The most
that can be said is that this only has occurred in exceptional circumstances. But that does not mean that a
reasonable man would dismiss such a risk from his mind and do nothing when it was so easy to prevent it. If it
is clear that the reasonable man would have realized or foreseen and prevented the risk, then it must follow
that he is liable for damages.
Once find negligent, then theres no good answer to question why did you do it? Wouldnt have done it
cause shouldnt have done it anyway.
Twerski doesnt understand reasonable man only neglect risk if reasonable reason for doing soin
present case no justification for charge in Sydney harbor.not only was it an offence to do so, but it involved
considerable loss financially. P269 to say why did you do it, there is no justification for having done so in any
negligence case- if not, you wouldnt be negligent wed never get to prox cause ?. thinks its a poorly written
decision, thinks were living with wagon mound 1 (not back to Polemis) have fire risk which is within the scope
and therefore impose liability.
Wagon Mound Comparison: In this case the original plaintiff is the ship in the harbor, and in Wagon Mound
1 it was the dock.
o Wagon Mound 1 fire risk is unforeseeable.
o Wagon Mound 2 they push the foreseeability, which they did not do the first time.
o Plaintiffs here now have the foreseeability issue because they were the ships, and had no risk of
contributory negligence.
Sum UP- negligence asks generally did you act reasonably under the circum and we dont break down the ? were you
negligent for particular person or particular harm and then ask but for. We get to prox cause we go back negligent for what
kind of risk? What kind of risk did you bring to this world? If the risk is within the scope of the risk of the risk you brought to
this world, then youre liable. If not, then its not product of your negligence.
11. Palsgraf v. Long Island R.R.
Facts: was a passenger standing on a platform of the railroad after buying a ticket; a train stopped at the
station, and a man ran forward to catch it; when he attempted to board the train in haste, he was assisted by
employees and was pushed onto the train, at which point he dropped a package containing fireworks; as a
result, the passenger woman, standing 15 feet away, was injured from the subsequent explosion when scales
fell on her and sought to hold the railroad liable for negligence.
Issue: Whether the LIRR was proximately negligent under the foreseeability test and should have known that
the package could cause harm.
o Whether the defendant had a duty to protect the plaintiff passenger from an explosion caused by another
passengers fireworks package being dropped onto the tracks.
Rule: The risk reasonably to be perceived defines the duty to be obeyed, and risk imports relation; it is risk to
another or to others within the range of apprehension. The range of reasonable apprehension is at times a
question for the court, and at times, if varying inferences are possible, a question for the jury.
o If no hazard is apparent to the eye of ordinary vigilance, an act innocent and harmless, at least to outward
seeming, with reference to a plaintiff, does not take to itself the quality of a tort because it happened to be
a wrong, though apparently not one involving the risk of bodily insecurity, with reference to someone else.
In every instance, before negligence can be predicated of a given act, back of the act must be sought and
found a duty to the individual complaining, the observance of which would have averted or avoided the
injury. The ideas of negligence and duty are strictly correlative.
Holding: Judgment for defendant. The passenger failed to prove that the railroad's alleged negligence
proximately caused her injuries. Essentially, the court held that under the foreseeability test, it was not
reasonable to hold that the railroad's alleged negligence was the cause of the passenger's injuries. Rather, it
was the explosion that was the proximate cause, and the railroad could not have reasonably expected such a
disaster.
Twerski discussion- did LIRR breach their duty to palsgraf? Unknown how can you answer prox cause ? if you dont
have negligence answer. Hes mystified as to what the heck cardozos doing here.. ends up saying theres no duty to ms
palsgraf. Does that mean we run a b<pl for her alone if we start running them for her alone well never have liability
cause shes such a low foreseeability. We said were not going to do that. Whatever/however the negligence might be, she
isnt within the scope of the risk in other words, I dont know what the negligence is, but I cant think of any in which shes
in the scope. He cant be saying you have to have a duty to palsgraf, because if he is then we have to run a b<pl for her
alone and were not going to do that.
Andrews Dissent: Andrews suggests a test that encompasses the entire series of events and then you
determine negligence by weighing all the factors. This is a contradiction to Cardozo who wants to draw a
distinct line.
Scope of risk or foreseeability isnt be all/end all of prox cause in this case. For Cardozo this is the be
all/end all if something isnt in the scope there is no liability. Andrews says its something to think about but
isnt be all/end all. But for test lets too many cases through the screen and we have to be able to limit liability,
matter of expediency and public policy has a mixed view. Says looks into foreseeabiliy and take into
account how far we should push liability, look @ policy factors, @ any other factors that I want to look at in
deciding if we should expand scope of liability. Clearly differs with Cardozo, thinks Cardozo charitably read
something has to be within scope of risk to impose liability, he calls it no duty and Twerski would call it no prox
cause.
o The test is not foreseeability, because if youre negligent toward one person, you are negligent toward the
immediate world. He recognizes the need for a liability/negligence-limiting doctrine. However, he suggests
that foreseeability should not be the sole test. It should be taken into account, but there is no automatic
cutoff after foreseeability; it is one of many factors to be considered. At some point, liability must be cut off
and other factors must be considered to determine liability.
Cardozos Strategy: This case is put into duty language because then it becomes clear that it is the role of
the court to direct a verdict, whereas the jury would be given the case if it were a proximate cause case. This
is easier to do when you are using the word duty it means that there was a violation and the judge can step
in to make a ruling.
o Can this risk be attributed to a result of the negligence?
If no, there is no duty breached between the plaintiff and the defendant.
If after risk analysis, if the negligence does not apply to the risk, you can say, therefore, there is
no
o Cardozo did an analysis of risk of foreseeability in this case.
o If the question is whether or not Palsgraf was within the scope of the risk when we do a B<PL, was she
part of the kinds of probabilities of harm that may have taken place. Answer may very well be that she
was. Cardozo thought, and Twerski thinks this is the most charitable way to read that case- that w/e
negligence there was in the case could not have reached Palsgraf and she had to be off the risk list so no
liability to her. Twerski thinks use of the word duty signals importance of legal question of prox cause and
encourage judges to direct verdicts when outside scope of liability.
o The crucial thing to understand is that Twerski doesnt think cardozo is saying ask the B<PL ? w/ regard
to particulate risk- you dont do that cause youd eviscerate risk and P would be very low, instead we ask
whether or not D was negligent and then was but for cause and then look back and can you say this was
reasonably within the risk list.
12. Buffalo River Fiasco
Facts: Two ships collide and together go traveling down the river. This was caused because one of the ships
was negligently moored to a dock, which caused it to drift to sea. They reach a drawbridge, but it is
unattended, causing the ships to collide with it. This results in a backup of water and a flooding of the stream.
Issue: Whether it was foreseeable that a ship would cause damage by knocking over a drawbridge and
subsequently flooding the stream, when the ship was poorly moored, causing it to drift into another boat.
Rule: Determine if the flooding of the stream was in the bundle of risks associated with negligently mooring a
boat to the dock.
Holding: Judgment for plaintiff. It could be foreseen that damage would have occurred because of the faulty
mooring. It could be attached back to the initial risk.
Who the plaintiff is a foreseeable plaintiff because the flooding damage is foreseeable. It could
have loosened and not tied to the other ship, and it still could have caused natural problems to
the environment (i.e., the flooding and backup).
o What flooding damage is a foreseeable risk.
o How the way the damage happened is not foreseeable. The manner of the harm was the only thing
not foreseeable. Standard rule is the manner of harm doesnt have to be foreseeable if Plaintiff and What
of the harm is, you have liability. Not a directed verdict but goes to the jury.
B. Superceding Causes
1. Derdiarian v. Felix Contracting Corp.
Facts: Trial court held for , and Appellate affirmed; employee was injured on an excavation job when he
was struck by a car driven by a man suffering an epileptic seizure; the man suffered the seizure because he
failed to take medication that would have prevented it; sued defendant employer, claiming that the
employer failed to maintain a safe work site by erecting a traffic barrier. Not suing his er because the defense
would use workers comp defense.
Defense here- the person who didnt take medication is superceding cause here. The court responds and
says dont need to know precise manner; if construction site is not properly blocked- drivers could lose control
for innumerable reasons so it was foreseeable that some driver would enter construction site and hit him.
Manner of the harm not foreseeable wont get you off the hook for prox cause, that will be a jury issue wont be
a directed verdict.
Issue: Whether the defendant employers inadequate safety precautions on the work site were the proximate
cause of the accident.
Rule: The plaintiff must generally show that the defendants negligence was a substantial cause of the events
that produced the injury. Plaintiff need not demonstrate, however, that the precise manner in which the
accident happened was foreseeable.
o Liability turns upon whether the intervening act is a normal or foreseeable consequence of the situation
created by the defendants negligence. If the intervening act is extraordinary under the circumstances, not
foreseeable in the normal course of events, or independent of or far removed from the defendants
conduct, it may well be a superseding act that breaks the causal nexus.
Holding: Judgment affirmed for plaintiff. The precise manner of the event need not be anticipated. The finder
of fact could have concluded that the foreseeable, normal, and natural result of the risk created by was the
injury of a worker by a car entering the improperly protected work area. An intervening act may not serve as a
superseding cause, and relieve an actor of responsibility, where the risk of the intervening act occurring is the
very same risk that renders the actor negligent.
o That defendant could not anticipate the precise manner of the accident or the exact extent of injuries,
however, does not preclude liability as a matter of law where the general risk and character of injuries are
foreseeable.
o The court says that it is the manner of the harm. You had an unguarded workspace it did not matter that
a guy who was having a seizure drove this particular car. That is unforeseeable. It is foreseeable that
someone would drive his or her car there though.
Restatement 442 Considerations Important in Determining Whether an Intervening Force Is a Superseding
Cause
o The following considerations are of importance in determining whether an intervening force is a
superseding cause of harm to another:
(a) the fact that its intervention brings about harm different in kind from that which would otherwise
have resulted from the actor's negligence;
(b) the fact that its operation or the consequences thereof appear after the event to be extraordinary
rather than normal in view of the circumstances existing at the time of its operation;
(c) the fact that the intervening force is operating independently of any situation created by the actor's
negligence, or, on the other hand, is or is not a normal result of such a situation;
(d) the fact that the operation of the intervening force is due to a third person's act or to his failure to
act;
(e) the fact that the intervening force is due to an act of a third person which is wrongful toward the
other and as such subjects the third person to liability to him;
(f) the degree of culpability of a wrongful act of a third person which sets the intervening force in
motion.
2. Sequence of Considering Liability
Duty
Negligence
But-for liability
o
Proximate Cause
o Risk Foreseeability
o Other Policy Grounds
Ways to stop from imposing liability
Holding: Judgment reversed for ; it was impermissible for the Appellate Division to dismiss. Suicide is not a
superseding cause in negligence law precluding liability.
Third Parties: An initial tortfeasor may be liable for the wrongful acts of a third party if foreseeable.
o Example: a tortfeasor may be liable for the ensuing malpractice of a physician treating the victim for the
tortiously caused injuries.
6. Wagner v. International Railway
Facts: and his cousin boarded a train that was very crowded; the train doors did not close, and when the
train was making a wide turn before a bridge, it jerked violently, and the cousin fell out; the train stopped and
went out to look for him with the conductor who was carrying a lamp; when was inspecting the bridge, he
found his cousins hat, then missing his own footing and fell, becoming injured in the process; trial judge held
for .
o Defendant argues that plaintiff should have gone below the trestle with the others looking for the body,
and not on the bridge; he should have known, in view of the overhang of the cars that the body would not
be found above; his conduct was not responsive to the call of the emergency; it was a wanton exposure
to danger that was useless.
Issue: Whether the plaintiffs fall was due to the negligence of the defendant for operating the train in such a
way that it allowed a passenger to fall out of it, causing the plaintiff to disembark and search for him.
Rule: Danger invites rescue. The cry of distress is the summons to relief. The law does not ignore these
reactions of the mind in tracing conduct to its consequences. It recognizes them as normal. It places their
effects within the range of the natural and probable. The risk of rescue, if only it be not wanton, is born of the
occasion. The emergency begets the man. The wrongdoer may not have foreseen the coming of a deliverer.
He is accountable if he had.
o Example: The state that leaves an opening in a bridge is liable to the child that falls into the stream, but is
also liable to the parent who plunges to its aid.
o In a rescue situation, the quality of the rescuer's acts is to be determined by the jury . The standard is the
reasonable man. "Errors of judgment" will not count against him, if they resulted from the excitement and
confusion of the moment.
Holding: Judgment for ; danger invites rescue- the law does not ignore reactions of the mind. May have
been foolish to look for cousin but wasnt viewing the landscape he was trying to rescue. remanded for a jury
to decide upon, not the judge. Whether the cousins fall was due to the defendants negligence, and whether
the plaintiff in going to the rescue was foolhardy or reasonable in the light of the emergency confronting him,
were questions for the jury.
o Continuity in such circumstances is not broken by the exercise of volition. The law does not discriminate
between the rescuer oblivious of peril and the one who counts the cost. It is enough that the act, whether
impulsive or deliberate, is the child of the occasion.
o It is within the scope of the negligence to assume that if someone goes overboard, someone else will go
look for the person, and that any harm done to that person should be included.
o Is it foreseeable on the part of the defendant that someone might not be careful in searching? Yes!
o Any time you are negligent you have to foresee that other people might act negligently as well; the train
company must foresee that the rescuer might not have the tact to rescue the fallen man in a careful way.
It would be one thing if the rescuer began doing acrobatics to find the man, but he was going about his
mission with a normal approach, and for that it is reasonable to assume that the train company is
negligent.
C. Playing the Duty Card
1. Hamilton v. Beretta U.S.A. Corp.
Facts: The relatives of persons killed by handguns sued the manufacturers; a jury held for s; suit was
brought for market share liability, asserting s were severally liable for failing to implement safe marketing and
distribution procedures, and that this failure sent a high volume of guns into the underground market. Heart of
the claim is that they did not control the distribution of the guns. Guns were coming from states where there
were less-rigid laws, which provided easier access to obtaining a gun. The guns would then be imported to
states where the laws were difficult to overcome. Plaintiffs sued the defendant gun manufacturers
because of their collective failure to market and distribute their product in a more careful, riskreducing way.
o Argument: s have a duty to exercise reasonable care in marketing and distribution of their guns
based on four factors: (1) defendants' ability to exercise control over the marketing and distribution of their
guns, (2) defendants' general knowledge that large numbers of their guns enter the illegal market and are
used in crime, (3) New York's policy of strict regulation of firearms and (4) the uniquely lethal nature of
defendants' products.
o Argument: They do not owe a duty to members of the public to protect them from the criminal
acquisition and misuse of their handguns. s assert that such a duty potentially exposing them to
limitless liability should not be imposed on them for acts and omissions of numerous and remote third
parties over which they have no control. Further, they contend that, in light of the comprehensive statutory
and regulatory scheme governing the distribution and sale of firearms, any fundamental changes in the
industry should be left to the appropriate legislative and regulatory bodies.
Issue: Whether the defendants owed plaintiffs a duty to exercise reasonable care in the marketing and
distribution of the handguns they manufacture, and if so, whether the duty could be imposed as market share.
Rule: This case must be evaluated in terms of a duty issue to the plaintiffs, and a market share possibility
o Duty Issue
Foreseeability alone does not define duty it merely determines the scope of the duty once it is
determined to exist. The injured party must show that owed not merely a general duty to society but
a specific duty to him or her, for without a duty running directly to the injured person there can be no
liability in damages.
A defendant generally has no duty to control the conduct of third persons so as to prevent them from
harming others, even where as a practical matter defendant can exercise such control.
o Market Share Issue
Common law Hymowitz DES case is the governing law
(1) The manufacturers acted in a parallel manner to produce an identical, generically marketed
product; (2) the manifestations of injury were far removed from the time of ingestion of the
product; and (3) the Legislature made a clear policy decision to revive these time-barred DES
claims.
Holding: Judgment for defendant on the duty issue, thus making the market-share issue irrelevant.
o Duty Issue
The pool of possible plaintiffs is very large potentially, any of the thousands of victims of gun
violence. Further, the connection between defendants, the criminal wrongdoers and plaintiffs is
remote, running through several links in a chain consisting of at least the manufacturer, the federally
licensed distributor or wholesaler, and the first retailer.
Such broad liability, potentially encompassing all gunshot crime victims, should not be
imposed without a more tangible showing that defendants were a direct link in the causal
chain that resulted in plaintiffs' injuries, and that defendants were realistically in a position to
prevent the wrongs.
Plaintiffs have not shown that the gun used to harm plaintiff Fox came from a source amenable to the
exercise of any duty of care that plaintiffs would impose upon defendant manufacturers.
o Market Share Issue
Unlike DES, guns are not identical, fungible products.
Even more importantly, given the negligent marketing theory on which plaintiffs tried this case,
plaintiffs have never asserted that the manufacturers marketing techniques were uniform.
In Hymowitz, each manufacturer engaged in tortious conduct parallel to that of all other
manufacturers, creating the same risk to the public at large by manufacturing the same defective
product. Market share was an accurate reflection of the risk they posed. Here, the distribution and
sale of every gun is not equally negligent, nor does it involve a defective product. Defendants
engaged in widely varied conduct creating varied risks. Thus, a manufacturer's share of the national
handgun market does not necessarily correspond to the amount of risk created by its alleged tortious
conduct. No case has applied the market share theory of liability to such varied conduct and wisely
so.
Discussion:
o Does defendant owe a duty to the plaintiff? Foreseeability alone does not create a duty. There might be
foreseeable risks, but that does not necessarily mean that a duty exists. To have to set up rules for
how a distribution scheme for gun manufacturers should work is absurd to expect. It is a very complex
task to establish such a scheme, and is certainly not a task for the courts to overtake.
o But-for causation is difficult to determine, because you cannot know which guns were illegal, and which
guns were trafficked in a wrongful way. Would criminals still be able to get guns even if there were a rigid
distribution scheme? You cannot, under traditional tort law, solve the issues that are being asked of the
courts in this case by the plaintiffs.
o Assuming that you do find them liable, you dont know which manufacturers gun killed the person. It
would be impossible to determine market share at this point. Guns vary in make, size, caliber, function,
etc., making it difficult to determine the negligence of a particular defendant.
o The court, in essence, is saying that they do not feel it is appropriate to impose a duty on the entire gun
manufacturing industry for their marketing practices.
o Twerski thinks the court is saying leave me alone with gun manufacturers, we are staying out of this
business its way beyond our ability of control- negligent distribution of guns is a no duty case, we arent
touching this. Had they dont it on prox cause grounds but took whole grade of cases and said dont try
and bring this before us again. Because all methods of controlling it cant do it, market share cant do that,
causation question cant do that and said way beyond capacity.
Chapter 6: Nonliability for Foreseeable Consequences (Limited Duty Rules)
A. Limitations on the Duty to Rescue
1. General
The duty to protect against wrong is, generally speaking and excepting certain intimate relations in the nature
of a trust, a moral obligation only, not recognized or enforced by law.
Courts have recognized a number of exceptional situations in which such a duty may arise. The most
common of these occur when a preexisting relationship between the party in peril and the potential rescuer
justifies recognition of a duty to act.
o These have included special relationships running between a common carrier and its passengers, a
school and its students, and an employer and its injured employee. Beyond these classic examples,
generalizations are difficult.
Twerski aside on page 310 is just no liability for serving drinks to social guests. Palsgraf speaks to ? of whether or not
Mrs Palsgraf was within the scope of the risks. If you use duty to speak about a broad category of cases, by and large
duty does speak to a fairly large category of behavior.
When you make a no duty determination, youre deciding on fairly broad social grounds you dont want to impose liability
for that class of cases. Want to look @ how difficult the causation ?s are.
One question p311 had to do with whether or not the doctor shouldve prescribed the drug in the first place [McKenzie v
Permanente Med Group] 2nd upped the dosage and 3rd claim was failure to warn about fainting etc. with regard to failure to
warn patient about ramifications of drug court was willing to address court wasnt ready to recognize shouldnt have
prescribed the drug/or the dosage issues. One of the reasons didnt give the better drug was because managed care
didnt want to pay for expensive drug just the cheaper. We will not recognize malpractice actions in terms of how they
affect conceivable effect on third parties.
CHAPTER 6
Dont recognize a duty to rescue. In many instances its easy, and if wed gotten past it then negligence, proximate cause
etc would/might be easy. If you want to stop it have to stop it @ the duty stage. Difficulty with if 10 people couldve helped
are they all joint tortfeasors.
Yania v. Bigan
Facts: Defendant was engaged in a coal strip-mining operation whereby trenches were dug in order to
remove coal deposits; one trench contained several feet of water, and defendant had placed a pump in the
trench to remove the water; defendant asked the widow's husband to assist him in starting the pump;
according to plaintiff, defendant taunted and urged the husband to jump into the water; he jumped into
the water and drowned; the widow initiated wrongful death and survival actions against defendant, alleging
that defendant taunted her husband, failed to warn him of the danger, and failed to rescue him.
Issue: Whether defendant owed the plaintiff the legal duty of rescuing him when the plaintiff drowned as a
result of complying the defendants request to do a task.
Rule: If it appeared that the deceased, by his own carelessness, contributed in any degree to the accident,
which caused the loss of his life, the defendants ought not to have been held to answer for the consequences
resulting from that accident. He voluntarily placed himself in the way of danger, and his death was the result
of his own act.
Holding: Judgment for defendant. , a reasonable and prudent adult in full possession of all his mental
faculties, undertook to perform an act that he knew or should have known was attended with more or less
peril and it was the performance of this act and not any conduct upon s part that caused the death.
o Claim 1 to contend that the conduct of taunting directed to an adult in full possession of all his mental
faculties constitutes actionable negligence is not only without precedent, but also is completely without
merit.
o Claim 2 , as possessor of the land on which died, breached no duty to warn of the risk presented
by the water-filled trench. ** Twerski has a problem with this because was on the property for
business purposes, so therefore it is not purely a rescue case. The added element of duty for a
pre-existing relationship could be argued for in this case. **
o Claim 3 the mere fact that saw in a position of peril in the water imposed upon him no legal,
although a moral, obligation or duty to go to his rescue unless was legally responsible, in whole or in
part, for placing in the perilous position.
o Court holds no duty of care so dont get to the question of whether reasonable steps to rescue are.
Duty Based on Undertaking 42 of Restatement third: an actor who undertakes to render services to another that the
actor knows or should know reduce the risk of physical harm to the other has a duty of reasonable care to the other in
conducting the undertaking if:
1. the failure to exercise such care increases the risk of harm beyond that which existed without the undertaking or
2. the person to whom the services are rendered or another relies on the actors exercising reasonable care in the
undertaking.
Baker v Fenneman & Brown Properties, 793 N.E. 2d 1203
L.S. Ayres precedent case- department store found liable for aggravation of customers injuries caused by its ers failure to
stop the escalator. The ees were liable not for injury that wouldve taken place anyway but for add-on injuries. Not liable
for arm stuck in the first place.
Court said two things couldve triggered the duty- 1. Rescue helpless person when one proceeded against is a master or
an invitor or 2. When the injury resulted from use of an instrumentality under the control of the D.
A. In Vincent the court cited Putnam. P101 if in that case the vessel had been permitted to remain and the dock had
suffered an injury, we believe the shipowner would have been held liable for the injury done.
B. Depue v Flateau- privileged trespasser you have a right to be there. If you vomit on rug, youre liable for damages but
have a right to be there.
PROCEDURAL POSTURE: Plaintiff customer sued defendant restaurant, alleging that the restaurant
breached its duty to render assistance to him until he could be cared for by others, and gross negligence, wanton
disregard, and wanton recklessness. The customer sought damages for medical bills, lost wages, pain and
suffering, and mental anguish. The restaurant moved for summary judgment. The Warrick Superior Court
(Indiana) granted the motion. The customer appealed.
OVERVIEW: The customer entered the restaurant when he became nauseous and fell backward, hitting his
head on the floor. The customer was knocked unconscious and began having convulsions. Moments after he
stood up, the customer fell again, was knocked unconscious, and suffered injuries.
When he regained consciousness, the customer stumbled out of the restaurant and was taken to the hospital by
his fiance.
A. Twerski thinks its interesting, second time he fell what should the ee have done?
B. On appeal, the customer claimed that the restaurant had a duty to assist him and that it breached that duty.
C. The restaurant argued that it had no duty to assist the customer because it was not responsible for the
instrumentality that cause the customer's initial injury; a doctor determined that the customer had
experienced a form of fainting that resulted from stress.
D. The court of appeals concluded that the restaurant had a duty to provide reasonable assistance to the
customer even though it was not responsible for the customer's illness. Consequently, the restaurant had a
duty to take reasonable action to give aid to the customer after he fell and to care for him until he could be
cared for by others. Whether that duty was breached was a question precluding summary judgment.
OUTCOME: The judgment of the trial court was reversed and the case was remanded for trial.6
Stockberger v. United States of Americadiabetic working in a federal prison who left work and
was in an accident arguing negligent for not providing transportation for ees who become ill @
work.
Breached duty of Indiana tort law claim by letting hypoglycemic drive- Twerski says IN tort law
doesnt provide for this duty.
Federal Claims Act- apply the law of state in which court is sitting.
No duty holding.
o You cant sue the federal government for things that are discretionary functions
o There could have been a procedure set up with other employees or with the employer for
situations like these that rise (let contract take the burden off of tort)
o Twerski says Posner seems to say a public invitee duty that there isnt for an er. As you thin rank
of potential rescuers it becomes easier to impose a duty to rescue. Twerski thinks hes saying
there might be greater rights for public invitee than employee who knows the routine at work and
if you dont like it you can bargain for it or union can and maybe rights there are less than they
should be for public invitee Twerski isnt so convinced but he makes the argument.
o Then Posner says even if I recognize duty to invitee, I still wouldnt in this case b/c: they let him
go home. If they didnt, that would be false imprisonment. What would you argue in defense?
Defense of others.
o The employer should have a duty to the employee (even though that is not the ruling here)
because if not, there could not be a duty to anyone else.
o What did co-ees do for him? They gave him Ensure to drink. Started the rescue and didnt
complete it and Twerski thinks theres an argument to be made in this case that by giving him the
Ensure made him half-well but once you start the rescue you have to ask whether or not youve
increased the risk. Would he been better off rather than ensure to call 911?
o Punishing for acting and stopping gives incentives not to act @ all.
J.S. and M.S. v. R.T.H.2 young girls (12& 15) raped by their neighbor but is his wife liable?
o NJ statute: Anyone who has a reasonable cause to believe that a child is being abused must report
it immediately to DYFS. This duty is imposed on everybody.
o Wife knew, or should have known, about her husbands proclivities but not that he was abusing
the children. [Twerski: whatever that means]
o Major ? of this case was: duty to rescue case
Court starts this discussion general tort law that there is no general duty to rescue. Court has
to build a case here for duty to rescue. How do they do that? Court started out with statutory
duty to report.. strong public policy in the state to protest children from sexual abuse and NJ
statute requires any person that has reason to believe a child has been subject to abuse to
report the abuse. Not limited to doctors/teachers but required of every citizen
Statute does not provide for civil liability, written for criminal liability and the court is using
this statute to say should we create a civil duty?
Reasonable cause to believe- says nothing more than did you have to act reasonably.
P171 Texas court asked to do a similar thing, impose a duty based on the statutory
requirement of having to report in Perry v SN, 3 reasons why they ought not to use
the statute: 1. Tort law in general doesnt impose a duty to protect another from crim
acts of a 3rd person, would be created a duty 2. Language of statute was vague and did
not clearly set forth the standard of conduct, 3. The court noted a sexual abuser can be
imprisoned from 5 to 95 years. P171 nonreporters are subject to fines. The legislature
clearly wished to treat nonreporters far less harsh than abusers.
Court imposes many ridiculous things the wife could have done. List: confronted husband and
warned him, Twerski thinks this is silly he wont say. She could have stopped the girls from
coming over, how should she insist this? She tells the parents dont let the kids come over
why? Because my husbands a sexual pervert. Twerski is mocking this. Husband might beat her.
Twerski says court doesnt know anything about that marriage, and even if it did what would you
do with it? What would happen if someone actually saw sexual abuse going on and didnt
report.. not the wife, but someone who happened to be coming by, would the court impose a
duty? Most of this case is built on wifes special knowledge and ability to control events and
Twerski questions how far the court intends to take this.
o Twerski has grave doubt that if this wasnt the wife the court wouldnt have imposed civil
liability, even for actual knowledge. Twerski thinks wife is most vulnerable person in this
picture.
o He also says we dont even know what proclivities mean.. literature, child porn, what does that
mean? Most of the authority goes toward the Perry case and the unwillingness to impose a duty.
o Once you get past the duty stage, then the question is did she act reasonably, was it a cause-infact was it a proximate cause then youre done. You either kill it at the duty stage or plaintiff is
going to win the case.
Tarasoff v. Regents of the University of CaliforniaPoddar told his psychologist of his plans to
kill Tatiana. Actually killed her after a brief detention & release by police when he seemed rational
and psychologist took no further effort to warn or detain, 2 months later. Parents of deceased suing
school psych. dept for negligence in not warning of their daughters imminent death
Court struggles with whether there should be liability. No relationship betw psychologist and
Tatiana, but there is a relationship betw the psychologist and Poddar and the court suggests thats a
special relationship that can create a duty to rescue.
What does majority hold? Why does concurring only go along w/ it on limited grounds?
Psychologist did in fact predict Poddar would kill Tatiana. Bottom of 341 Amicus contends
even when a therapist predicts a patient poses a serious danger of violence to others, the therapist
should be absolved of any responsibility for failing to act to protect the potential victim. In our view
however, once a therapist does in fact determine that a patient poses a serious danger of violence to
tohers he bears a duty to exercise reasonable care to protect foreseeable victim of that danger.
J Mosk concurs in part because: in this case the psychologist actually predicted, not should have
predicted part.
o has a special relationship to the killer because he is a patient and this relationship becomes the
grounds for imposing liability
o Even though the relationship is not with the decedent, it is still sufficient to hinge the duty
o Problem of Dr. being the only one aware of the situation (they can keep two sets of records)
o It would be very difficult to prove that the Dr. should have known
o Since you cannot get someone committed without proving they are an immediate threat to
themselves or others, it is best to at least warn others of the immediate harm or danger
o Courts do not deal well with mental health problems
o Twerski thinks this is a case where causation is impossible to figure out, and ought not to touch
this case.
Twerski thinks these cases are a holy mess. The tort court is a limited place where you can put on certain shows
and Twerski says you cant put on the lets deal with the human condition show. The best world we can
possibly make is so open-ended and thats why we have limited duty rules not stepping in when its well
beyond ability to do it and Twerski thinks rescue cases are extraordinarily difficult because they implicate hard
questions of causation, standard of care, and come out mush.
2. LIMITATIONS ON RECOVERY FOR PURE ECONOMIC LOSS
Twerski is a big fan of the way this case is written strong circuit.
State of Louisiana, Ex. Rel. Guste v. M/V Testbank
o
Issue- there was a large ship that was carrying PCPs (12 tons) and it sprung a leak through
negligence and caused havoc on the Mississippi river. At the time, probably the worst spillage of
toxics into the Mississippi river that had taken place. Lots of people affected shipping, marina
operators, wholesale/retail food enterprises, etc.
Is there a cause of action for negligent infliction of economic loss? Court relies on Robins case,
someone had a K to repair ship which wasnt repaired in time and resulted in economic loss
and said there is no cause of action for negligent infliction of economic loss. can water businesses
recover for damages when there is no injury to the property but to the business?
Majority says negligent infliction of economic loss is potentially boundless. The number of
people who can be affected is quite large.
o If there was no limitations on economic loss, it would run into millions of dollars for one small
negligent act.
o What limitation should be applied?: A bright line no duty rule? duty in terms of proximate cause?
Case by case basis? Line drawing difficulty nothing law-like in drawing that line. It would
have nothing to do with scope of risk, and just for sake of drawing a line.
o Since there is no principle way of deciding these losses, it is best to have a bright line no duty
rule.
o These situations could be better dealt with through first-party insurance
People Express Airlines, Inc. v. Consolidated Rail Corp.airline suffered loss of business
because of chemical escape from the railway
o This court ruled to limit it to a clearly identifiable (this is the minority rule)
o If people could contract with potential s, that might solve the problem (but insurance is already
a partial contract)
o The problem is that they are at risk for a whole host of accidents that might not be due to
anyones negligence.
General Rule: No recovery for defective products that cause economic loss
A. Asbestos exception, cant really distinguish
3. LIMITATIONS ON RECOVERY FOR EMOTIONAL DISTRESS
4. Prior to 46 in Restatement there was cause of action for this only if it causes physical harm. If it
didnt there was no case. Then, the court said lets not play that game lets look @ what is really going
on, intentional (or reckless), outrageous conduct, serious/severe emotional distress then dont need
physical harm. Thats a limited duty rule, means you must check these three things before well listen to
your claim and a court can say this conduct is not outrageous enough.
5. 46: an actor whose negligent conduct causes serious emotional disturbance to another is subject
to liability to the other if the conduct: a) places the other in immediate danger of bodily harm and the
emotional disturbance results from the danger; or b) occurs in the course of specified categories of
activities, undertakings, or relationships in which negligent conduct is especially likely to cause serious
emotional disturbance. P369
The first limitation on emotional distress: Physical Impact (this is mostly dead except in a few
jurisdictions)
Now, there is a cause of action for negligent infliction of emotional distress if there is a physical
manifestation of it (this is an attempt to make a duty rule)
o Problem: What constitutes a physical manifestation? Once a line is determined, everyone will
claim to have that physical manifestation. People will conform their claim to it.
Daley v. LaCroixcar drove into a utility pole causing electrical wire explosion in s home causing
emotional distress
o Court said that a physical manifestation was necessary but then took evidence that was not
related to a physical manifestation (eg-sleeplessness, loss of weight)
o The sensible rule would be the use the same requirements of intentional infliction of emotional
distress instead of a physical manifestation and then use that as a cut off to limit litigation
The problem is that there are some cases of serious emotional distress but it might not meet
those requirements
o For negligent infliction of emotional distress, you have to figure out what kind of injury you are
going to recognize: emotional distress, serious emotional distress, physical manifestation?
o One set of distinctions: negligent infliction of emotional distress that leads to physical
manifestation vs. negligent infliction of emotional distress that causes emotional distress only
What about recovery for emotional distress suffered for negligence to another (such as a child)?
o Rules: Zone of dangerif you are physically in the zone of danger (close enough that it could
have been you that the negligence occurred to) of the original negligence, then you can recover
not for your own fear but for watching what happened to an immediate family member
recovery for mental shock
Not in the zone of danger no recovery
NY is a zone of danger state: Can recover without physical manifestation and within the zone
of danger if the is within risk of bodily harm and is an immediately family member
o Tough problem because memory isnt always accurate and you are usually not thinking about
exactly where you were standing because you arent expecting something happen.
Thing v. La Chusamom coming upon accident moments after her sons death
o CA Law: in the absence of physical injury or impact to the himself, damages for emotional
distress should be recoverable only if the is closely related to the injured victim, is present at
the scene of the injury-producing event at the time it occurs and is then aware that it is causing
injury to the victim and as a result suffers emotional distress beyond that which would be
anticipated in a disinterested witness.
o It is not recovery for the death but for watching the accident that caused the death
How can you possible differentiate between the pain from the death and the pain from
viewing the death.
o All of these are offensive and insensible because they are assuming there is a way to compensate
for these horrible offenses. Maybe a better solution is to cap the amount of damages and not
make distinctions such as these.
6. HARM TO UNBORN CHILDREN
Werling v. Sandyare stillborns considered persons that could therefore recover in a wrongful
death case?
o This court says they are
o Courts are not generous with wrongful death damages
Procanik by Procanik v. Cillobaby born with rubella because Dr. negligently failed to diagnose
mom with German measles
o Wrongful birth/wrongful life cases
o In all of these cases, if the child is born, the child will be born with a birth defect. The
negligence is in the failure to communicate to the parents that there was a birth defect or the
failure to discover by the parents by tests that the child would be born with birth defects?
o The first question is, Is there a cause of action for the child? For the parents? If there is a cause
of action, is it for special damages that are involved because it is more costly to bring up a child
with serious birth defects? Or are there pain and suffering/emotional distress damages for the
child? For the parents?
o Scott v. Bradford: Materiality questionthey are entitled to that information that the dr. failed to
warn them about; Causation questionhas flaws because it is what would this person do if they
had the information but they didnt have the information and so it should be an automatic cause
of action. The violation of personal autonomy is complete the second the person does not
receive the information.
o Childs best argument is that they would have been better off never being bornso most courts
have said no to any cause of action on behalf of the child
This court says there is a cause of action for the special costs of raising the child but not for the
emotional distress of being alive (for the child). The parents can recover for the specific
damages of medical expenses or for emotional distress (NJ).
o
7. Erie R. Co. v. Stewart
Facts: Plaintiff truck passenger was injured when a train at a railroad crossing struck the truck; a watchman
voluntarily employed by defendant gave a warning that was too late; the watchman was not there by law, but
by the defendants own attempt to provide safety. Plaintiff recovered in district court, and defendant appeals.
Issue: Whether there was any positive duty owed to the plaintiff in respect to the maintenance of a watchman
when defendant, at the place of the accident, had maintained one over a long period of time to the knowledge
of the plaintiff.
o In other words, the plaintiff relied on this watchman, but it wasnt required by law it was just a nice thing
that the defendant decided to do so it is hard to determine if a duty was breached by his absence.
Rule: Where the traveler upon the highway knows the practice, and such traveler has been educated into
reliance upon it, some positive duty must rest upon the railway with reference thereto.
o Where the voluntary employment of a watchman was unknown to the traveler upon the highway, the mere
absence of such watchman could probably not be considered as negligence toward him as a matter of
law, for in such case there is neither an established duty positively owing to such traveler as a member of
the general public, nor had he been led into reliance upon the custom. The question would remain simply
whether the circumstances demanded such employment.
Holding: Judgment for plaintiff. The evidence conclusively establishes the voluntary employment of a
watchman, knowledge of this fact and reliance upon it by the plaintiff, a duty, therefore, that the company,
through the watchman, will exercise reasonable care in warning such travelers as plaintiff, the presence of the
watchman thereabouts, and no explanation of the failure to warn. Therefore, even though the duty is
considered as qualified, rather than absolute, plaintiff established a prima facie case.
o The company has established for itself a standard of due care while operating its trains across the
highway, and, having led the traveler into reliance upon such standard, it should not be permitted
thereafter to say that no duty required, arose from or attached to these precautions.
8. Abandoned Rescue
Compare Erie to the case of Lacey in which plaintiff sued defendant Coast Guard for not completing a rescue
mission following a plane crash
o It is true that, while the common law imposes no duty to rescue, it does impose on the Good Samaritan
the duty to act with due care once he has undertaken rescue operations. The rationale is that other
would-be rescuers will rest on their oars in the expectation that effective aid is being rendered.
9. L.S. Ayres & Co. v. Hicks
Facts: The injured visited the store and while descending on an escalator, he fell and his fingers were
caught in the moving parts of the escalator; jury found for ; is not being held liable for negligently
operating the escalator because one is not bound to guard against a happening that there is no reason to
anticipate; is claiming that did nothing to prevent the injury from worsening.
Issue: Whether the defendant may be held liable for an aggravation of an injury when the injury occurred on
an escalator in the defendants department store, and the defendant did not act quickly to halt the
occurrences, or have a guard near the escalator to prevent such injury.
Rule: It has been said that the mere happening of an accident of this kind creates a relation that gives rise to
a legal duty to render such aid to the injured party as may be reasonably necessary to save his life, or to
prevent a serious aggravation of his injuries, and that this subsequent duty does not depend upon the
negligence of the one party, or the freedom of the other party from contributory negligence, but that it exists
irrespective of any legal responsibility for the original injury.
Holding: Judgment for plaintiff regarding aggravation of injuries. was an invitee and he received his initial
injury in using an instrumentality provided by and under its control. This was a sufficient relationship to
impose a duty upon . cannot be charged with its anticipation or prevention, but only with failure to exercise
reasonable care to avoid aggravation.
o There may be a legal obligation to take positive or affirmative steps to effect the rescue of a person who is
helpless and in a situation of peril, when the one proceeded against is a master or an invitor or when the
injury resulted from use of an instrumentality under the control of the defendant, like the escalator here.
Such an obligation may exist although the accident or original injury was caused by the negligence of the
plaintiff or through that of a third person and without any fault on the part of the defendant.
Note: The negligence was failure to stop the escalator once the child was in peril. Damages only start at the
aggravation of injury because they are not responsible for injuries that took place prior to the time the
defendant could have stopped the escalator.
Restatement 322: If the actor knows or has reason to know that by his conduct, whether tortious or
innocent, he has caused such bodily harm to another as to make him helpless and in danger of future harm,
the actor is under a duty to exercise reasonable care to prevent such future harm.
10. I Have a Right to Kill You, but Not to Let You Die
Kuntz features a situation in which the defendant was charged with the negligent homicide for causing the
death of her boyfriend. Her boyfriend was beating her, and in self-defense she stabbed him with a knife, and
then fled from her apartment. She did not call for medical help for her boyfriend, and he died. The court held
against the defendant:
o A duty to rescue exists in both criminal and tort law if the defendant has full knowledge of the injuries, the
failure to rescue is the cause-in-fact of the injuries, and the defendant has already exercised her right to
seek and secure safety from physical harm.
11. J.S. and M.S. v. R.T.H.
Facts: children, ages 12 and 15, spent substantial periods of recreational time with their neighbor wife
and her husband at their horse barn, riding and caring for the horses; s husband, an older man, sexually
abused for a period of more than a year; brought an action against for damages arising from s
husband's actions on the theory that she had a duty to protect them. wife admits that she should have
known of the sexual abuse.
Issue: Whether a wife who suspects or should suspect her husband of actual or prospective sexual abuse of
their neighbors children has any duty of care to prevent such abuse.
Rule: New Jersey Statute: Any person having reasonable cause to believe that a child has been subject to
abuse to report the abuse immediately to the Division of Youth and Family Services. The duty to report is not
limited to professionals, but is required of every citizen.
o If the wife had not admitted that she should have known, this analysis would be needed: A number of
factors are relevant when determining whether or not it is foreseeable to a wife that her husband would
sexually abuse a child. These include whether the husband had previously committed sexual offenses
against children; the number, date, and nature of those prior offenses; the gender of prior victims; the age
of prior victims; where the prior offenses occurred; whether the prior offense was against a stranger or a
victim known to the husband; the husband's therapeutic history and regimen; the extent to which the wife
encouraged or facilitated her husband's unsupervised contact with the current victim; the presence of
physical evidence such as pornographic materials depicting children and the unexplained appearance of
children's apparel in the marital home; and the extent to which the victims made inappropriate sexual
comments or engaged in age-inappropriate behavior in the husband and wife's presence.
Holding: Judgment for plaintiffs. When a spouse has actual knowledge or special reason to know of the
likelihood of his or her spouse engaging in sexually abusive behavior against a particular person or persons, a
spouse has a duty of care to take reasonable steps to prevent or warn of the harm. Further, a breach of such
duty constitutes a proximate cause of the resultant injury, the sexual abuse of the victim.
o Mary could have discharged her duty by confronting her husband and warning him, by insisting or seeing
that the girls were not invited to ride or care for the horses, by keeping a watchful eye when she knew the
girls to be visiting with her husband, by asking the girls' parents to ensure that the children not visit when
she was not present, or by warning the girls or their parents of the risk she perceived.
12. Stockberger v. United States
Facts: The widow's husband was an employee at a federal prison and was an insulin-dependent diabetic. His
co-workers knew the husbands medical condition. The husband was killed in a car accident after he left work
because he did not feel good and was having a hypoglycemic episode. Even though his co-workers did not
believe the husband was in any condition to be driving, no one prevented him from leaving work and driving
home; the co-workers gave him crackers and juice, and then the decedent insisted on driving home; court of
appeals held for defendant, and now the plaintiff appeals.
Issue: Whether co-workers have a duty to stop someone who they know to be ill from driving.
Holding: Judgment for defendant. The court of appeals found that the Federal Tort Claims Act barred the
widow's claim that the federal prison system was negligent in failing to have a policy of providing
transportation for employees who were dangerously ill at work because this claim was clearly barred by the
discretionary-function exception under 28 U.S.C.S. 2680(a).
o The widow's state law tort claim that the prison's action, or inaction, when it allowed the husband to drive
in his hypoglycemic condition was a breach of the duty of care imposed by Indiana tort law was also
properly denied because Indiana had not changed the common law rule with regard to the duty to rescue.
Indiana had not taken the step of imposing Good Samaritan liability on invitors.
Discussion: This case was decided on the duty to rescue issue it was found that there was none. This was
not a case where the defendant did nothing. Instead, they gave him sugar to aid his condition. They began the
rescue attempt. They made him better enough to make him believe that he was okay and capable of driving
home.
What is a reasonable rescue in this case? Do you have to drive him home, or to a hospital?
If you make the no duty claim, it will not get to the negligence stage. B < PL will never be reached, and we
wont have to worry about balancing the probability that he will get injured because the co-workers did not
provide him the standard of care. If we got to that stage, the defendants would be in bad shape.
13. Doe v. XYC Corporation
Facts: The employee was the mother's husband and worked for the corporation as an accountant. It was
discovered that he was secretly videotaping the mother's 10-year-old daughter at their home and sending the
photos to child pornography sites using the corporation's computer at his workstation. The corporation had
become aware of the employee viewing pornography, including child pornography, but did nothing. The
employee was arrested on child pornography charges. The corporation knew that the employee had a wife
and child since they had attending company outings with him. In granting summary judgment, the trial court
determined that the corporation had no duty to investigate the private communications of the employee and
that it had no control over the employee's conduct at home.
Issue: Whether an employer is responsible for an employees actions when the employer is aware that the
employee is using company computers to distribute child pornography of his 10-year-old stepdaughter.
Rule: Restatement 317
o A master is under a duty to exercise reasonable care so to control his servant while acting outside the
scope of his employment as to prevent him from intentionally harming others or from so conducting
himself as to create an unreasonable risk of bodily harm to them, if the servant is upon the premises in
possession of the master or upon which the servant is privileged to enter only as his servant, or is using a
chattel of the master, and the master knows or has reason to know that he has the ability to control his
servant, and knows or should know of the necessity and opportunity for exercising such control.
Holding: Judgment for plaintiff. The court held that there was no expectation of privacy on the part of the
employee and that the corporation was on notice that the employee was viewing pornography on the work
computer. As such, the corporation had a duty to report the employee's activities to the proper authorities and
to take effective internal action to stop those activities.
o An employer who is on notice that one of its employees is using a workplace computer to access
pornography, possibly child pornography, has a duty to investigate the employee's activities and to take
prompt and effective action to stop the unauthorized activity, lest it result in harm to innocent third-parties.
No privacy interest of the employee stands in the way of that duty on the part of the employer.
Note: This case is a stretch, and the implications for liability are horrendous. Duty could not be stretched
more according to Twerski. There are employer/employee limits, but the concerns of the employer were for
determining what was going on with the business, and not what he was doing in his personal life.
14. Tarasoff v. Regents of University of California
Facts: Defendants, therapists and police, did not confine a patient who expressed intentions to kill victim, nor
warn victim of patient's intentions; patient killed victim; plaintiffs, victim's parents, filed suit against defendants,
alleging failure to warn of impending danger, and failure to confine patient.
o Defendant argues that imposition of a duty to exercise reasonable care to protect third persons is
unworkable because therapists cannot accurately predict whether a patient will resort to violence. Open
communication in therapy sessions is essential to the practice, and if that is tainted, a patient will be
reluctant to make the full disclosure upon which diagnosis and treatment depends.
Issue: Whether the defendant therapist should take any steps at all to protect the threatened victim, when
doing so could deprive the therapists patient of his liberty.
Rule: Whenever one person is by circumstances placed in such a position with regard to another that if he did
not use ordinary care and skill in his own conduct he would cause danger of injury to the person or property of
the other, a duty arises to use ordinary care and skill to avoid such danger.
o As a general rule, one person owed no duty to control the conduct of another, nor to warn those
endangered by such conduct, the courts have carved out an exception to this rule in cases in which the
defendant stands in some special relationship to either the person whose conduct needs to be controlled
or in a relationship to the foreseeable victim of that conduct. Applying this exception to the present case,
we note that a relationship of defendant therapists to either Tatiana or Poddar will suffice to establish a
duty of care.
o Restatement 315:
A duty of care may arise from either "(a) a special relation . . . between the actor and the third person
which imposes a duty upon the actor to control the third person's conduct, or (b) a special relation . . .
between the actor and the other which gives to the other a right of protection."
Holding: Judgment for plaintiff regarding failure to warn. Defendant therapists' special relationship to patient
was extended to victim, and a duty existed to use reasonable care where they had knowledge that patient
was going to harm victim. The therapists obligations to his patient require that he not disclose a confidence
unless such disclosure is necessary to avert danger to others. Court says that there is no duty for failing to
o
o
commit the murderer once he told the therapist of his intentions. Instead, he should have called the victim or
her parents.
B. Limitations on Recovery for Pure Economic Loss
1. State of Louisiana, Ex Rel. Guste v. M/V Testbank
Facts: Forty-one actions were brought against defendant ships by plaintiffs state, shipping interests, marina
operators, and others following a collision between defendants two ships that resulted in the spill of 12 tons of
PCP into the Mississippi River Gulf outlet; the outlet was closed temporarily to all navigation and to fishing,
shrimping, and related activities; the trial court granted defendants summary judgment as to all such claims
except those asserted by, inter alia, commercial fishermen. The plaintiffs in this case are the merchants that
service the area, who were affected by economic damages only (we care only about those who are not
fishermen for the scope of this section).
o Plaintiffs argue that the requirement of physical injury to a proprietary interest is arbitrary, unfair, and
illogical, as it denies recovery for foreseeable injury caused by negligent acts. Plaintiffs advocate not only
that the lines be drawn elsewhere but also that they be drawn on an ad hoc and discrete basis. The result
would be that no determinable measure of the limit of foreseeability would preclude the decision on
liability.
Issue: Whether physical damage as a prerequisite to recovery for economic loss should be abandoned, when
a boat collision causes a toxic spill that devastated the local economy surrounding the affected water.
Rule: As the Robins court decided, there could be no recovery for economic loss absent physical injury to a
proprietary interest.
Holding: Judgment for defendant. Denying recovery for pure economic losses is a pragmatic limitation on the
doctrine of foreseeability, a limitation we find to be both workable and useful. You cannot impose liability
because there is no property damage. This rule exists because you need a cut-off point for where you can
assign liability. There needs to be a finite stopping place, which is difficult to determine when you are dealing
with economic loss. With physical damages it is easy to determine who was injured; with economic loss, there
are no boundaries.
Dissent: The judges dissent because of the economic loss doctrine, arguing for and against the burden of
this on tort law. You can only provide so much deterrence, but at some point, the losses get so large, that
there is no deterrence when your economic loss is concerned. If the loss potential is $1 trillion, there is no
deterrence, because what is the difference between $1 and $2 trillion? Both amounts are insurmountable. A
line needs to be drawn, so it is drawn barring ALL economic loss, as opposed to allowing some, but with a
capped amount (majoritys response).
o Wisdom (dissenting judge) says that you can permit liability with public nuisances. Use a case-by-case
analysis. Use proximate cause as the cutoff point instead of barring all economic loss recoveries. He
agrees that a line needs to be drawn, but he wants to use foreseeable risks. Duty should not be the
screening device because it cuts it off too early. Are there particular damages specific to these plaintiffs
that fall within the scope of duty?
Response: Majority says that there is no possible way to do this in a fair manner. What do you do,
allow restaurants to recover, but tackle and bait shops not to? Where do you draw the distinction?!
They need a firm rule. Dissent says it is too harsh, and that even though a line needs to be drawn,
they want to evaluate by checking out the whole case.
Palsgraf: This debate is like the one in Palsgraf in which they wrestled with the scope of risk/foreseeability
issue and duty. Their differences are that Andrews in that case wants to hear everything and then determine
where to cut it off (foreseeability alone will not be the test). Cardozo, on the other hand, wants a firm rule that
provides clarity as to the scope of liability (the limit that he wants is foreseeability). The majorities in both
cases require a firm rule, as opposed to the dissents, which are looking to evaluate on a case-by-case basis
in terms of scope of risk/foreseeability.
2. Middle Ground Position
In JAire Corp. v. Gregory (Cal. 1979), a restaurant operator brought suit against a general contractor for
construction delays that interfered with the operation of the restaurant. In denying the defendants motion for
summary judgment, the court addressed the concerns that liability could be imposed for remote
consequences that the measure of recovery would be highly speculative.
o The court identified six criteria to employ when assessing liability: (1) the extent to which the transaction
was intended to affect the plaintiff, (2) the foreseeability of harm to the plaintiff, (3) the degree of certainty
that the plaintiff suffered injury, (4) the closeness of the connection between the defendant's conduct and
the injury suffered, (5) the moral blame attached to the defendant's conduct, and (6) the policy of
preventing future harm.
3. People Express Airlines, Inc., v. Consolidated Rail Corp.
Facts: Plaintiff's airline business was evacuated for 12 hours because of a fire allegedly resulting from
defendants' negligence; plaintiff claimed damages for economic losses resulting from the evacuation; trial
court granted summary judgment for defendant; appellate division reversed.
Issue: Whether a defendants negligent conduct that interferes with a plaintiffs business resulting in purely
economic losses, unaccompanied by property damage to personal injury, is compensable in tort.
Holding: Abandons the standard set in Guste, and adopts a new quasi-property interest rule:
o A defendant owes a duty of care to take reasonable measures to avoid the risk of causing economic
damages, aside from physical injury, to particular plaintiffs or plaintiffs comprising an identifiable class with
respect to whom defendant knows or has reason to know are likely to suffer such damages from its
conduct.
Note: This is a much shorter version of the rule that I have written out in my outline. However, Twerski hates
this holding. He mentioned that he didnt like it in class, and there is an Authors Dialogue in the casebook.
Twerski likes the rule in Guste/Palsgraf in which there is a distinct line drawn for economic loss recovery.
C. Limitations on Recovery for Emotional Distress
1. General
Deals with claims in which a defendants allegedly negligent conduct, while not harming the plaintiff physically,
has caused the plaintiff to become fearful or otherwise emotionally distressed.
The potential for limitless liability gives courts some pause; thus, limitations on recovery have been imposed.
Emotional distress requires outrageous conduct and severe distress. See Harris (Chapter 1(E)(1)). Now we
are viewing this in terms of negligent conduct as opposed to intentional conduct. In other words, what
negligent acts could someone do that would result in emotional distress to another?
2. The Impact Rule
Mitchell v. Rochester Ry. established the impact rule there could be no recovery for fright alone without
impact.
There could be no recovery for any resulting physical manifestation of the fright, such as nervous disease,
blindness, insanity, or even a miscarriage.
This was once the clear weight of authority in the country, but not so much any more, because the zone-ofdanger rule is now favored. Only three jurisdictions still uphold this.
3. The Zone-of-Danger Rule
Waube v. Warrington established the zone-of-danger rule the defendants duty and the plaintiffs right can
neither justly be extended to any recovery for physical injuries sustained by one out of the range of ordinary
physical peril as a result of the shock of witnessing anothers danger.
o In this case, the plaintiffs decedent was looking out the window of her house, watching her child cross the
highway, when she witnessed the defendant negligent run over and kill the child. She did from the result
of her shock.
o Court holdings
It is one thing to say that as to those who are put in peril of physical impact, impact is immaterial if
physical injury is caused by shock arising from peril. It is quite another thing to say that those who are
out of the field of physical danger through impact shall have a legally protected right to be free from
emotional distress occasioned by the peril of others, when that distress results in physical
impairment.
The liability imposed by such a doctrine is wholly out of proportion to the culpability of the negligent
tortfeasor, would put an unreasonable burden upon users of the highway, open the way to fraudulent
claims, and enter a field that has no sensible or just stopping point.
The class of plaintiffs is narrowed in the zone-of-danger. It is now limited to people who witness the accident,
experience emotional distress, and perceived a threat of danger upon him.
o Example: A parent whose child got hit by a car when they were holding hands crossing a street.
The zone of danger is restricted by the proximity to the actual accident.
This is the law in New York, but it is not the law nationally.
4. Daley v. LaCroix
Facts: s vehicle left the roadway and crashed on s property, snapping high voltage lines, and causing a
great electrical explosion resulting in property damage; claims to have suffered traumatic neurosis,
emotional disturbance and nervous upset, and nervousness as a result of the explosion; the trial judge and
appellate division both ruled in favor of , stating that Michigan law denies recovery for this absent a showing
of physical impact.
Issue: Whether the defendant should be held liable for the emotional distress suffered by plaintiffs, when
there was no accompanying physical impact from his negligent operation of his car.
Rule: Common law is debated between the Mitchell case, above, and a New York ruling that overruled the
Mitchell court. In essence, Impact versus Zone of Danger.
Holding: Judgment for ; case remanded for new trial in which the following standard will apply
o Where a definite and objective physical injury is produced as a result of emotional distress proximately
caused by defendant's negligent conduct, the plaintiff in a properly pleaded and proved action may
recover in damages for such physical consequences to himself notwithstanding the absence of any
physical impact upon plaintiff at the time of the mental shock.
o Limitations
Absent specific knowledge of plaintiffs unusual sensitivity, there should be no recovery for
hypersensitive mental disturbance where a normal individual would not be affected under the
circumstances.
Plaintiff has the burden of proof that the physical harm or illness is the natural result of the fright
proximately caused by the defendants conduct.
New Precedent Established: There is no physical impact to the plaintiffs here. This court is not ready to
create a rule for no physical impact. The impact rule here is ditched though (there would be no liability
because nobody was touched). What is necessary, according to this court, is negligent infliction of emotional
distress that results in subsequent physical injury.
o You must now prove physical harm, not impact. This is a new element of the duty rule!
o This is only for a tort that is directed at that person, not viewing someone else. In other words,
bystanders/zone of danger problems do not apply. This is a limited duty rule.
5. Foreseeability and the Zone of Danger A New Rule
Dillon v. Legg established a test that would avoid the arbitrariness of the zone-of-danger rule and the openended liability that would flow from a pure foreseeability-based test
o This is a new test that adds foreseeability to the zone-of-danger rule! This is the majority rule on the
subject!
o In determining, in such a case in which plaintiff suffered a shock which resulted in physical injury, whether
defendant should reasonably foresee the injury to plaintiff, or, in other terminology, whether defendant
owes plaintiff a duty of due care, the courts will take into account such factors as the following:
(1) Whether plaintiff was located near the scene of the accident as contrasted with one who was a
distance away from it.
(2) Whether the shock resulted from a direct emotional impact upon plaintiff from the sensory and
contemporaneous observance of the accident, as contrasted with learning of the accident from others
after its occurrence.
This makes it difficult to determine accurately. What is contemporaneous observance? Within one
minute, or two minutes, etc. It makes it impossible to distinguish between cases. Makes you
wonder if wed be better off without a rule, so that there is no issue of distinguishing between
seconds or minutes.
(3) Whether plaintiff and the victim were closely related, as contrasted with an absence of any
relationship or the presence of only a distant relationship. (This element is retarded)
6. Thing v. La Chusa
Facts: was near the scene where her young son was injured in an automobile accident, but did not witness
the accident; she came upon the scene moments later and saw her son in a badly injured condition, and
suffered emotional harm; trial court granted summary judgment; appellate reversed.
Issue: Whether a mother who did not witness an accident in which an automobile struck and injured her child
may recover damages from the negligent driver for emotional distress suffered.
o Whether the guidelines established in Dillon are adequate, or if they should be refined to create a greater
certainty in this area of law.
Rule: Dillon is the current standard in California at this time, and the court here is determining if the test set
forth in that case is appropriate, or if the limitations are improper.
Holding: Judgment for , applying a new rule
o A plaintiff may recover damages for emotional distress caused by observing the negligently inflicted injury
of a third person if, but only if, said plaintiff:
(1) is closely related to the injury victim;
(2) is present at the scene of the injury-producing event at the time it occurs and is then aware that it
is causing injury to the victim; and
(3) as a result suffers serious emotional distress a reaction beyond that which would be anticipated
in a disinterested witness and which is not an abnormal response to the circumstances.
o Defendant recovers here because plaintiff was not present at the scene of the accident in which her son
was injured (element 2). She did not observe defendant's conduct and was not aware that her son was
being injured. She could not, therefore, establish a right to recover for the emotional distress she suffered
when she subsequently learned of the accident and observed its consequences.
Rationale for New Rule: It is appropriate to restrict recovery to those persons who will suffer an emotional
impact beyond the impact that can be anticipated whenever one learns that a relative is injured, or dies, or the
emotion felt by a "disinterested" witness. The class of potential plaintiffs should be limited to those who
because of their relationship suffer the greatest emotional distress.
o The elements which justify and simultaneously limit an award of damages for emotional distress caused
by awareness of the negligent infliction of injury to a close relative are the traumatic emotional effect on
the plaintiff who contemporaneously observes both the event or conduct that causes serious injury to a
close relative and the injury itself.
ASK TWERSKI IF THIS IS THE CURRENT STANDARD, OR IF DILLON V. LEGG STANDS
7. Direct Victim Rule
This is the narrowest rule.
Johnson v. State established the direct victim rule for when plaintiffs are victims to whom defendants owe
duties of care directly.
In this case, a state hospital notified the sister of a patient that the patient had died. However, it turned out to
be another patient at the hospital who also happened to be named the same name. The sister of the patient,
and the patients daughter sued the hospital. It was held that the daughter were entitled to recovery
o She was the one to whom a duty was directly owed by the hospital, and the one who was directly injured
by the hospitals breach of that duty.
D. Harm to Unborn Children
1. General
Unborn children are persons capable of being physically harmed for purposes of bringing an action to recover
civil damages later on, after the child is born with physical injuries caused prenatally by a wrongdoers
conduct.
2. Werling v. Sandy
Facts: brought a wrongful death action against medical care providers, alleging that their negligence
caused s child to be stillborn; the trial court dismissed the complaint, ruling that there is no action for the
wrongful death of an unborn child.
Issue: Whether the beneficiaries of the unborn fetus are entitled to damages for the wrongful death of the
fetus where both the alleged negligently inflicted injury and death of the child occurred before birth.
Rule: R.C. 2125.01
o When the death of a person is caused by wrongful act, neglect, or default which would have entitled the
party injured to maintain an action and recover damages if death had not ensued, the person who would
have been liable if death had not ensued, or the administrator or executor of the estate of such person, as
such administrator or executor, shall be liable to an action for damages.
o The purpose of this statute is to provide a remedy whenever there would have been an action in damages
had death not ensued.
Holding: Judgment for plaintiff. The ruling is reversed and remanded for new trial in which R.C. applies. A
viable fetus that is negligently injured in the womb, and subsequently stillborn, may be the basis for a wrongful
death action pursuant to R.C.
o A cause of action may arise under the wrongful death statute when a viable fetus is stillborn since a life
capable of independent existence has expired. It is logically indefensible as well as unjust to deny an
action where the child is stillborn, and yet permit the action where the child survives birth but only for a
short period of time.
o To allow a cause of action where it is established that the fetus was viable certainly furthers the remedial
nature of the wrongful death statute. To hold otherwise would only serve to reward the tortfeasor by
allowing him to escape liability upon an increase in the severity of the harm, if such harm results in death
to the child. In other words, the greater the harm inflicted, the better the opportunity that a defendant will
be exonerated. This result is clearly not acceptable under the statute. This is along the lines of when
Twerski says that if you hit a person with your car, youre better off backing up over them to ensure that
they die, because the damages will be less severe. Sad.
3. Procanik by Procanik v. Cillo
Facts: Infant plaintiff alleged defendant doctors negligently failed to diagnose that infant plaintiff's mother had
contracted German measles in the first trimester of pregnancy. As a result, infant plaintiff was born with
congenital rubella syndrome. Infant plaintiff filed a wrongful life suit that was dismissed for failure to state a
cause of action upon which relief could be granted.
o Plaintiffs argue that the doctor negligently deprived the parents of the choice to terminate the pregnancy;
the injury plaintiff seeks
General damages for his pain and suffering and for his parents impaired capacity to cope with his
problems; and,
Special damages attributable to the extraordinary expenses he will occur for medical care throughout
the course of his life.
Issue: Whether a wrongful life claim can be brought against a defendant when the defendant failed to
diagnose a pregnant mothers disease while her child was still in utero, preventing her from opting to abort.
o Additionally, whether the defendant can be held liable for future medical expenses as a result of failing to
do the above, which led to the babys illness.
Rule:
o Common law
Gleitman in 1967 held that if a doctor negligently diagnosed or treated a pregnant woman who was
suffering from a condition that might cause her to give birth to a defective child, neither the parents
nor the child could maintain a cause of action against the negligent doctor.
More recently the court advanced the parents right to compensation by permitting recovery of the
extraordinary expenses of raising a child born with cystic fibrosis, including medical, hospital, and
pharmaceutical expenses.
o Wrongful life" refers to a cause of action brought by or on behalf of a defective child who claims that but
for the defendant doctor's negligent advice to or treatment of its parents, the child would not have been
born.
o Wrongful birth" applies to the cause of action of parents who claim that the negligent advice or treatment
deprived them of the choice of avoiding conception or, as here, of terminating the pregnancy.
Holding: Judgment for regarding medical damages; for regarding general damages.
o Medical Damages Childs cause of action
When a child requires extraordinary medical care, the financial impact is felt not just by the parents,
but also by the injured child. As a practical matter, the impact may extend beyond the injured child to
his brothers or sisters. Money that is spent for the health care of one child is not available for the
clothes, food, or college education of another child.
o General Damages Parents cause of action
The congenital rubella syndrome that plagues him was not caused by the negligence of the
defendants doctors; the only proximate result of their negligence was the childs birth.
It is simply too speculative to permit an infant plaintiff to recover for emotional distress attendant on
birth defects when that plaintiff claims he would be better off if he had not been born.
You would not be held liable if you dont know that a child would be there. This is different than should
have known, which is the normal standard. There must be obvious facts that support the knowledge of
children in the area trespassed upon.
o Negligence differs from this because you could have a very high P, with a low probability of harm, and
they can still be negligent. With the Restatement, it does not matter. It is a duty analysis.
A duty analysis is designed to screen out cases so that you cant view them all as a whole it
becomes a structured analysis that requires you to satisfy each element!
(2) Duties Owed to Licensees
1. General
Licensees are persons who are on the land with the consent of the owner but are there for their own purpose.
Social guests are considered by the courts as licensees, as are other entrants who come with permission to
use short cuts, to distribute advertising leaflets, or to solicit charitable contributions.
Common law treats licensees similar to trespassers. Thus, the possessor has a duty to conduct activities on
the land in a reasonable manner and to warn of hidden dangers known to him.
(3) Duties Owed to Invitees
1. General
This is the highest standard of care that is owed. There are two categories of invitees according to the
Restatement 332:
o Persons who are invited to come on the land for a purpose connected with the business dealings of the
possessor and;
o Persons who come on the land as a member of the public for a purpose for which the land is held open to
the public.
The duty owed to an invitee is essentially a full duty of care. A possessor may have a duty to inspect her
premises for dangers that create an unreasonable risk of harm and then to take reasonable steps to protect
the invitee against such dangers. Very often warning against the dangers can fulfill the duty of reasonable
care.
It is the duty of the court to determine the status of the entrant, not the jury.
339 Duty to child trespassers: Knows or has reason to know (has facts within knowledge with
o
trigger that you know) that children are likely to trespass (if using B<PL formula, this factor would
be a part of P) AND has reason to know that there is an unreasonable risk of death or serious bodily
harm to children (P and L, mostly L) AND children do not discover the danger (P) AND burden is
low compared to the risk AND the possessor fails to exercise reasonable care to eliminate the danger
or otherwise to protect the children (B). Not B<PL mix it all in a pot, if you dont make out every
one of these items, you lost. Basically separated out issues from B<PL. this is a limited duty rule
because have to get each of these to have limited recovery.
Example: Without any negligence on his part, A, standing on the platform of a subway station of
the X Company, slips and falls onto the tracks. While there he is run over by the train of X
Company, and injured. A is a trespasser.
Holding: Judgment for defendant. Remanded to lower court to be considered as a trespasser. Gladon was an
invitee when he purchased an RTA ticket, rode the rapid transit train, and waited at RTA's platform. However,
RTA's invitation to Gladon to use their premises did not extend to the area on or near the tracks. In fact,
Gladon acknowledged that RTA did not permit the public in the area on or near the tracks. When an entrant
upon anothers land exceeds the scope of the landowners invitation, the entrant will lose the status of an
invitee, and become either a licensee or trespasser. Because Gladon then became either a licensee or a
trespasser for purposes of determining the duty RTA owed to him, the trial court erred in instructing the jury
that he was an invitee as a matter of law. [Twerski has a problem with illustration on p403, without any
negligence on As part standing on platform slips and falls on tracks he is a trespasser he thinks
trespassers make the decision to go onto property, not accidentally slip]
o Willful/Reckless/wanton behavior the court finds that there is evidence that she was going 20mph over
the speed limit. It is possible that this is wanton/reckless behavior. They dont feel as though they should
determine what is wanton/reckless, so they remand the issue for a jury to decide.
o Reasonable behavior after discovery the court says that this is a fact question because she might not
have pulled the brake early enough. A jury must decide this.
o This is an emergency situation, so the reasonable behavior must be construed in this light. This is also an
issue for the jury. Also, was her conduct reckless before her discovery, did she discover, and was it
negligent after she discovered the trespasser? She does not have a duty of reasonable care until she
discovers the trespasser.
Test: Whenever you can use the screen, your first question should be, Does this case fall within the category
in which a screen can be used to determine a duty, as opposed to negligence?
o The trial court let the case go through the filter, and ruled that if he is an invitee, he must be granted the
standard of care. The appellate court did not like this, and said that he did not belong on the track, so he
should not be treated as an invitee, but as a trespasser.
The duty to a trespasser is to avoid reckless and wanton conduct before you see him. This applies
even if you do not see the trespasser. Once you see the trespasser, you owe him the duty of
reasonable care. However, there is no duty to discovery. After discovers or has facts that should
have reason to believe trespassor is there, that triggers the duty.
The issue becomes whether the conductor acted recklessly and wantonly. If she does not act
reckless and wanton, there is no liability. Once she discovers him on the tracks, she has to act
reasonably. This duty kicks in when she discovers him, and not a second before.
o This screening device brings two questions: did you act like a fool when you operated the train before you
saw the plaintiff; did you act reasonably once you discovered the plaintiff on the tracks?
3. Hypothetical
Y becomes an invitee once she is asked to help out; she was a licensee beforehand. She is now owed a full
duty of reasonable care. X is then liable because the simple warning was not enough once the person is not
an invitee.
This is confusing as opposed to Hypothetical 35 (in which warning signs are placed around a wet floor in a
school and a student slips and falls). Why is the warning to the friend different from the warning to the student
who slipped and fell in the hallway of the building? It seems like the two warnings are the same, for the same
type of situation a slippery floor. Answer right below.
The duty to the student is that of a landowner to an invitee, and the duty to the guest has a duty from the
homeowner. The duties are the same to both injured parties. There is no duty screen any more. Now the
question is what the reasonable care is for the owners. It might be a jury issue to see if there is a rope in Hypo
35. Or, you might decide that its too big of a deal to have to do that, so there should be no issue for the jury.
As for the host, the question is, is she negligent? Is the verbal warning enough to fulfill the duty if there is a
very slipper floor? In this Hypo, she should have to do more than simply saying that.
(4) Rejection of the Categories
1. Rowland v. Christian (got rid of all the categories & say now have a duty of reasonable care)
Facts: Summary judgment was granted for ; plaintiff alleged that defendant failed to warn him that her
bathroom fixtures were cracked and dangerous. The fixtures severed tendons and nerves in plaintiff's hand.
The trial court granted summary judgment for defendant, following the rule that a licensee or social guest was
obliged to take the premises as he or she found them, and the possessor of the premises owed a duty only to
refrain from wanton or willful injury.
Issue: Whether a homeowner owes the licensee social guest the duties of warning him that her bathroom
fixtures were cracked and dangerous.
Rule:
1. Carter v. Kinney
Facts: signed up at church for a home Bible study, which was hosted by s and sponsored by their church;
when arrived for the study at the homeowners' residence, he slipped on a patch of ice and broke his leg;
claims that he is an invitee, and urges the court to abolish the common law standard; s filed a motion for
summary judgment, claiming that was a licensee and that they did not have a duty with respect to a
dangerous ice condition of which they had no knowledge; trial court granted summary judgment on behalf of
s.
Issue: Whether plaintiff is an invitee of defendant, when he went to defendants home as part of a churchsponsored Bible study group.
Rule: The fact that an invitation underlies a visit does not render the visitor an invitee for purposes of
premises liability law. This is because "the invitation was not tendered with any material benefit motive" and
"the invitation was not extended to the public generally or to some undefined portion of the public from which
invitation entrants might reasonably expect precautions have been taken, in the exercise of ordinary care,
to protect them from danger."
Holding: Judgment for defendants. The record shows that Mr. Carter did not enter the Kinneys' land to afford
the Kinneys any material benefit. He is therefore not an invitee under the definition of invitee contained in
Restatement 332. The record also demonstrates that the Kinneys did not "throw open" their premises to the
public in such a way as would imply a warranty of safety. The Kinneys took no steps to encourage general
attendance by some undefined portion of the public; they invited only church members who signed up at
church. They did nothing more than give permission to a limited class of persons church members to
enter their property.
o There is discussion here as to whether it is a social gathering (licensee) or a business venture (invitee).
The difference would make the duty to the guest significant. If he were an invitee, they would have a duty
of reasonable care. As a licensee, there is no duty to discover that there is an icy patch, and there is no
duty to prevent the person from falling on it. If there were a duty, they would be responsible for going
outside, and perhaps salting the walkways.
B. Brief Review of Duties and the Categories
1. Duty of a landowner has two options
Categories (trespasser, licensee, invitee)
o Must warn of hidden dangers (for a licensee, like in Hypothetical 1 there would be no liability). There is
no hidden danger because the pool is very apparent.
No categories
o Full duty of reasonable care depends on
Was the defendant negligent?
Weigh the circumstances to determine the proper course of action. (B<PL)
You do not touch the affirmative defenses until you pass through the duty screen.
C. Special Rules Limiting Liability
1. The Firefighters Rule
Firefighters or police who entered on the premises of another to perform their functions were treated as
licensees. However, there are many exceptions that have skewed this generalization.
2. Recreational Use Statutes
To provide incentives to landowners to allow the public to use their property for recreation (and remove
demand on public parks), most states have enacted recreational use statutes that partially immunize the
owner for accidents that take place on property.
o If the public has free access to the land, the statutes provide an effective shield to the host. If you have to
pay, then you are not afforded this protection.
o A public user of land under these statutes may be offered less protection than the common law granted to
a trespasser.
D. Duties Owed to those Outside the Premises
1. General
A possessor of land is liable for harm done by activity on the premises or for artificial conditions created by
her on the premises to persons injured off the premises.
o Example: plaintiff recovered for being hit by a baseball while walking outside of the stadium.
The corollary to this rule is that the possessor is not responsible for natural conditions on the land that cause
injury off the land.
o Example: plaintiff cannot recover when snow melts and the water collects off the premises, freezes, and
someone slips on the ice and is injured.
2. Taylor v. Olsen
Facts: Plaintiff was injured when her car struck a tree that had fallen in the road during a storm. The tree had
been standing on defendant's land before it fell. Plaintiff brought an action for her injuries, and a directed
verdict was entered in favor of defendant. Plaintiff appealed.
Issue: Whether the defendant landowner owed a duty to plaintiff to clear a tree off of a road adjacent to his
land when the tree had splintered and fallen on a windy night.
o Whether the defendant should have known of the danger of the fallen tree, and under what conditions he
has a duty to inspect his trees to discover a latent danger.
Rule: There is a duty problem because the court used to have different rules for urban and rural trees. For
urban trees, there might have been a full duty to care, whereas in rural areas there was no duty to discover.
This was most likely because the risks that each posed were much different (greater risk in urban areas,
difficult to oversee entire property high B in rural areas).
o This court abolished this. They instead opted for a full duty of reasonable care they eliminated the
screen. The screen did make sense in this case. However, classifications of what was
urban/suburban/rural/etc. made it way too difficult. They went for negligence standard.
Holding: Judgment for defendant. In this case, the tree was rotting from the inside. So, even if he had a full
duty to discover it, there would be no way to inspect the tree and discover that the tree was rotting and posed
a risk of falling as a result of decay.
lighting or fencing, or trimming shrubbery. The plaintiff has the burden of establishing the duty the
defendant owed under the circumstances.
o The most important factor to be considered is the existence, frequency and similarity of prior incidents of
crime on the premises, but the location, nature and condition of the property should also be taken into
account.
o The balancing test addresses the interests of both business proprietors and their customers by balancing
the foreseeability of harm against the burden of imposing a duty to protect against the criminal acts of
third persons. In determining the duty that exists, the foreseeability of harm and the gravity of harm must
be balanced against the commensurate burden imposed on the business to protect against that harm . In
cases in which there is a high degree of foreseeability of harm and the probable harm is great, the burden
imposed upon defendant may be substantial. Alternatively, in cases in which a lesser degree of
foreseeability is present or potential harm is slight, less onerous burdens may be imposed. Under this
test, the high degree of foreseeability necessary to impose a duty to provide security, will rarely, if ever, be
proven in the absence of prior similar incidents of crime on the property.
Holding: Judgment for defendant. Sam's did not possess the requisite degree of foreseeability for the
imposition of a duty to provide security patrols in its parking lot (3 prior incidents but only one mugging which
doesnt do it). Nor was the degree of foreseeability sufficient to support a duty to implement lesser security
measures. Court is saying foreseeability better be pretty high in other words, Court is saying this is not
mush Learned Hand, this is hard economic stuff and wont send this to a jury on facts like this- theres a
difference, Twerski thinks this court is saying this isnt merely a directed verdict on standard of care but that
theyre very concerned about economic aspect of this and saying dont come to us with cases where
foreseeability is low cause were going to dismiss those cases. Accordingly, Sam's owed no duty to protect
Mrs. Posecai from the criminal acts of third parties under the facts and circumstances of this case. Given the
large number of customers that used Sams parking lot, the previous robbery of only one customer in all those
years indicates a very low crime risk. Although the neighborhood bordering Sams is considered a high crime
area by local law enforcement, the foreseeability and gravity of harm in Sams parking lot remained slight.
Screening Test: The test that they now rely on, which is a screening test, is very similar to the B<PL test,
which is the third test suggested and rejected in the opinion. It is rather stupid how the totality of
circumstances test is rejected, but this test is applied, when they are practically one in the same.
o The difference is that the court is saying that even though it is B<PL, they intend to take it very seriously.
Here, the court will play a big role, and not the jury. Unless the burden is very high, the case will stay with
the court, and not go to the jury.
o If 8/10 is the normal standard to give to the defendant, and 2/10 is normally what gets it to the plaintiff, the
court will now use a 6 to give it to the defendant. They are very concerned with these cases, and are
looking to get them right by taking into their hands.
Chapter 8: Affirmative Defenses
Both parties were a but-for cause. The plaintiffs negligence was the proximate cause of his harm. The
defendants negligence was the proximate cause of the harm.
2. Eliminating the Contributory Negligence Doctrine
Courts detest the contributory negligence doctrine, so they took steps to eliminate it. The idea that a
defendant who was negligent would get away scot-free when the plaintiff was guilty of even a little bit of
negligence was an abomination. Courts sought to mitigate the harshness of the rule barring plaintiffs
recovery.
o Many courts never directed a verdict against the plaintiff, hoping a jury would instead allow recovery. This
was a firm rule!
o The last clear chance doctrine was imposed.
If the plaintiff was negligent but the defendant had the last clear chance to avoid the injury, the
plaintiffs negligence would not count.
The reasoning was that a defendant who had the last opportunity to avoid harm was certainly more
negligent than the plaintiff who was frozen in a position of peril.
Beginning in the 1960s, state legislatures and courts abandoned the rule that contributory negligence
operated as a complete bar to recovery and replaced it with a kinder and gentler regime of comparative fault.
o Courts said that they will no longer bar a plaintiff from recovery simply because he contributed to his
injury. Instead, we will compare the fault of the plaintiff to the fault of the defendant.
Was plaintiff negligent: but-for cause, proximate cause?
Was defendant negligent: but-for cause, proximate cause?
Once this has been determined, you must assign the percentage of fault to each party.
Problem arose which form of comparative fault to adopt?
3. Forms of Comparative Fault
Pure Comparative Fault (New York version and 12 others follow)
o Take the plaintiffs fault, sue the defendant, and reduce the plaintiffs fault from the verdict.
If plaintiff is 10% at fault, plaintiff recovers 90% of damages.
50/50 Rule
o Plaintiff can recover as long as the plaintiffs negligence is not greater than that of the defendant. In other
words, recovery as long as plaintiffs fault is 49 percent or lower. If plaintiffs fault is 51 percent, he
recovers nothing.
49 Percent Rule
o Plaintiff can recover if plaintiffs negligence is less than that of the defendant. If plaintiffs negligence is 50
percent, the plaintiff recovers nothing.
Reasons for these rules
o When juries have the burden of determining the negligence, a lot of times they will come in with a split of
50/50. That is why the states are divided between the 50/50 rule and the 49 Percent Rule. Those states
that believe 50/50 entitles the plaintiff to recovery, they implement that rule, and vice versa. A jury will not
say, 47 percent. Thats obviously way too specific. Instead, now the courts use these rules so that when
its less than 50, thats all that matters, or if its simply 50, thats all that matters.
(2) Comparative Negligence
1. McIntyre v. Balentine
Facts: Plaintiff entered the highway from an entrance ramp, and shortly after was struck by defendants truck,
resulting in injuries. The defendant claims that plaintiff is contributorily negligent because he was intoxicated,
with a .17 blood alcohol level. Jury found for the defendant. Plaintiff appealed, alleging that the jury should
have been instructed of comparative negligence, which was not the law in Tennessee at the time.
Issue: Whether comparative fault should be adopted by Tennessee, when the state does not recognize such
a doctrine, but the situation involves a motor vehicle accident in which the injured plaintiff was intoxicated
while operating his motor vehicle.
Rule: 49 Percent rule. So long as the plaintiffs negligence remains less than the defendants negligence the
plaintiff may recover; in such a case, plaintiffs damages are to be reduced in proportion to the percentage of
the total negligence attributable to the plaintiff.
Holding: Judgment reversed and remanded, for plaintiff. The case will be tried with the new 49 Percent rule
applied. Because the jury, without the benefit of proper instructions by the trial court, made a gratuitous
apportionment of fault, we find that their "equal" apportionment is not sufficiently trustworthy to form the basis
of a final determination between these parties.
Elimination of Old Standards: The adoption of the comparative negligence system in Tennessee makes the
doctrines of remote contributory negligence and last clear chance obsolete. Also, the doctrine of joint and
several liability is rendered obsolete.
o
In cases of multiple tortfeasors, plaintiff will be entitled to recover so long as plaintiff's fault is less than the
combined fault of all tortfeasors.
o If there is a joint tort for $1mil., and 1 is responsible for 60% and 2 is responsible for 40%. In common
law, you could sue both for the $1mil., and the defendants would have to work it out (the burden is on
them to determine think of the three friends shooting their rifles in which one of them gets hit in the face
by the other two).
o Under common law, you could sue either defendant for the full $1 million. However, what McIntyre says is
that comparative fault allows you to assign percentages, and therefore each tortfeasor is now responsible
for only their percentage. Thus, 1 will pay $600K and 2 pay $400K. Sucks if one party cannot fulfill the
financial obligation. Lets say 1 is a regular guy and 2 is Wal-Mart. 2 would be sued under common
law, because you could recover all million. However, with comparative fault, the plaintiff will most likely
have to eat up that $600K because Joe Schmo cannot afford it.
The Scorecard
12 states follow the pure rule.
33 states favor the modified rule. Here, once a plaintiffs fault reaches either 50 or 51 percent, the plaintiff is
entirely barred from recovery.
4 states follow the rule that a plaintiff cannot recover if there is contributory negligence.
Allocating Fault Among Multiple Parties
Lets say the jury determines that the plaintiffs fault is 40 percent, and the three defendants are each
responsible for 20 percent. Each jurisdiction treats it differently.
o In New York, you would sue each defendant and reduce the negligence by 40 percent.
o Some jurisdictions will say that if the plaintiffs negligence is less than that of the defendants combined,
the plaintiff recovers. Others will say that it can only recover from defendants that have a higher
percentage.
The Interplay Between Comparative Fault and Proximate Cause
Many scholars have taken the position that, with the adoption of comparative negligence, cases that a
plaintiff might have lost under proximate cause rules because the act of the plaintiff or third parties were
regarded as supervening causes should go to a jury to apportion fault between the plaintiff, defendant, and
any third party.
o You now have a way of punishing a party who is at fault by applying a percentage to their liability.
o Comparative fault makes it easier.
Alami v. Volkswagen of America, Inc.
Facts: Plaintiffs deceased husband was involved in a one-car accident with a steel utility pole while getting
off at an exit ramp; decedents blood alcohol content was above the limit; the accident resulted in his death;
plaintiff claims that the drunk driving was not the cause of death, but that the car manufacturer was at fault for
defective design, which should have prevented death and caused only minimal injuries. The lower court
granted the defendants motion for summary judgment.
o Plaintiff argues that her husband's injuries were caused by design defects in the vehicle that rendered it
unsafe. Thus, plaintiff asserts that under these circumstances, her claim is not precluded on public policy
grounds because the injuries upon which the claim is based do not have the necessary causal link to the
decedent's serious violation of the law.
Issue: Whether defendant can be held liable when they did not cause the accident, but allegedly caused the
injuries sustained in the accident to be worse due to bad manufacturing of the car.
Rule: Barker/Manning common law rule
o A plaintiff cannot rely upon an illegal act or relationship to define the defendants duty.
o Where a plaintiff has engaged in unlawful conduct, the courts will not entertain suit if the plaintiffs conduct
constitutes a serious violation of the law and the injuries for which the plaintiff seeks recovery are the
direct result of that violation.
Holding: Judgment for plaintiff. Summary judgment should not have been granted. If Volkswagen did
defectively design the Jetta as asserted by plaintiffs expert, it breached a duty to any driver of a Jetta
involved in a crash regardless of the initial cause. Overturns common law rule because of comparative fault!
o Extension of the rule here would repeal legislatively mandated comparative fault analysis in a wide range
of tort claims. In essence, the dissent would have this Court extend the Barker/Manning rule to relieve
Volkswagen in this case of its duty to manufacture a safe vehicle. This we will not do.
o The court here said that if the defendants car is allegedly safe, it should protect someone when there is a
crash. Thus, there liability really starts after the crash takes place. In other words, once a crash occurs,
the defendant should put its drivers in a cocoon against harm. The drivers liability ends once he gets into
the accident, at which point the car manufacturers liability kicks in. I dont care why you got into the
crash; it simply matters if you were protected once the crash occurred.
o
2.
3.
4.
5.
It should be irrelevant how the accident happened if the car cannot prevent injuries. The only way around
it is if the car is not capable of being manufactured in a way that would prevent the injuries due to a
certain type of accident (like if youre driving 120mph).
Dissent: The majority's rationale therefore invites people injured as a result of their own seriously unlawful
acts to blame others and recover damages previously prohibited under Barker and Manning. That invitation
confounds this Court's preclusion jurisprudence, which courts had readily understood and followed. Under
today's analysis, unless a defendant was complicit in a plaintiff's criminal act, a court cannot preclude suit
without first concluding that the alleged duty arose out of that illegal act.
o This struggles with the new rule that allows a defendant to be held liable for injuries that resulted from an
illegal act. What this says is that if the person were abiding by the law, they would not have been in a
position to be injured more severely because of the defendants negligence.
(3) Assumption of Risk
1. General
Side by side with contributory negligence that operated as a complete bar, courts recognized another defense
that similarly barred a plaintiffs claim. When it could be said that plaintiff voluntarily assumed a known risk,
courts refused him a right to recover.
2. Express Assumption of Risk
Cases where the defendant agreed to allow the plaintiff to be exposed to her conduct only if the plaintiff
agreed to exculpate (waive) the defendant from liability for negligence.
The problem here is that it is not always so clear what consent you are giving, and what rights you are
waiving. An example is Jorst, below.
3. Jorst v. DAmbrosio Bros. Investment Co.
Facts: Plaintiff, mother and 9-year-old daughter, bring this action against a ranch owner who gave riding
lessons to the girl; plaintiff mother signed a document releasing the ranch from liability for any injuries that
could ensue from the riding lessons; during a lesson, the girls foot got caught in a pipe that was attached to
the wall of the ranch; the pipes had been left there from an event they were used for several weeks earlier;
plaintiffs claim negligence for failing to remove the pipe and failing to supervise the lesson.
Issue: Whether a signed release waives liability for all incidents that could occur during the course of the
waiver (does it include premises, riding, etc.?)
Rule: California courts have identified three prerequisites for a release to be enforceable:
o #1: The release must be clear, unambiguous and explicit in expressing the intent of the parties.
o #2: The act of negligence, which results in injury to the releasor, must be reasonably related to the object
or purpose for which the release is given.
o #3: The release must not be contrary to public policy (does not apply to this case).
Holding: Judgment for plaintiff. The Jorsts' injuries were not reasonably related to the purpose for which they
signed the Release. Christine was not injured as part of the inherent risk of horseback riding; she was injured
in an unforeseeable, unrelated way when the Ranch's allegedly negligent maintenance of the indoor arena
caused her to fall.
o Element 1
The release does not clearly express intent to exculpate the defendants for its negligent maintenance
of its premises. It does not sufficiently notify a customer that the ranch cannot be held responsible if it
increases the danger of riding by operating an unsafe facility.
A drafter of a release cannot be expected to include every type of risk imaginable in its release, but
from the Ranch's perspective, the negligent maintenance of the Ranch's premises is one of the more
likely sources of injury to the Ranch's customers. The Ranch's failure to include it in the Release
renders the document unclear and ambiguous with respect to whether the Ranch intended to disclaim
liability for negligent operation of its facilities. As a result, the Release is unenforceable.
o Element 2
Even if the release were clear and unambiguous regarding premises liability, the plaintiffs injuries
must have been reasonably related to the purpose for which the release was given.
A court must employ an objective standard whether a reasonable person in the plaintiffs position
could have foreseen the particular act of negligence that caused his injuries.
The risk that the ranch would negligently maintain its premises is not one that is reasonably related to
the purpose for which the release was given.
The customer does not acknowledge the additional risk created when the Ranch maintains its
premises negligently, and he or she cannot reasonably foresee injuries caused by the Ranch's
facilities.
In other words, the plaintiff could have foreseen being thrown from the horse and sustaining
injuries. The release would protect that. However, the injury in this case was not something you
would expect to happen when youre in a riding lesson. Thus, it was not within the release.
o
The injured person may not recover damages that do not result proximately from the defendants breach
of duty. Damages that might be avoided or mitigated are, therefore, not recoverable.
Holding: Judgment for defendant; case remanded for new jury instructions included avoidable
consequences.
B. Analysis of Fleming/Twerski Implied Assumption of Risk Debate
1. Fact Pattern
A is a drunk driver. B is sitting in the passenger seat of As car, fully aware of As intoxicated state of being. C
is asleep in the back seat, and has no knowledge of As intoxication.
2. Flemings Take
Argues that there is no need for an assumption of risk analysis for two reasons
o Duty: There is no duty to the passengers of the car. Because B knows that A is driving drunk, A has no
duty to B. Therefore, we dont even need assumption of risk because the case is then cut off.
o Comparative Fault: If it isnt a case of duty, why not decide on comparative fault? He would then say: if A
has a duty to B, you should then do a standard comparative fault, and thats how it should play out. His
assumption of the risk is a mitigating factor, and not a complete bar to liability.
3. Twerskis Take
Argues that both duty and comparative fault are problematic for a case like this
o Duty: A still has a duty to B. Heres why. Lets suppose C is in the car and is sleeping in the back seat. If A
has a duty to C, then he has a duty to everyone. There is still a duty!
o Comparative Fault: We think that B should not recover at all. However, by using comparative fault, they
will still recover, which is wrong. They should have known their risk because he knew A was drunk!
4. Solution
The only way to remedy the duty and the comparative fault issue is to say B assumed the risk! He still has a
duty to the person, even though he assumes the risk, so you cant cut it off here. You can go to comparative
fault, but that would be bad, because B would be able to get something. So, instead of using comparative
fault, you should go to implied assumption of risk, which would bar B from recovery.
o However, C would have not assumed the risk. He passes that element. From here, go to the duty analysis
for C: A owed him a duty; A was negligent; A was the but-for and proximate cause; thus, A is liable to C,
but not to B.
5. Analysis for Bs Case
Remember to make negligence case. He had a duty to B and he breached the duty to B by driving drunk. Butfor his drunk driving, he would not have gotten injured. It was a foreseeable risk. Thus, he is liable for injuries
to B. However, after negligence, you have to evaluate assumption of risk (as opposed to comparative fault). In
this case, defendant would get off liability because of assumption of risk. This happens at the stage of
affirmative defense, even though its not a standard affirmative defense.
6. Summary
This is a dispute of damages because the plaintiff assumed the risk. A plaintiff will argue Flemings theory.
They will argue for comparative fault. Stress all of things before the assumption. Defense is only using
assumption of risk as a way to wiggle out of the case. You are looking for negligence but no damages!
o
Issue: Whether guests on a barge are responsible for the injuries of someone on the barge that were
sustained when plaintiff was thrown overboard in a playful manner, but in a concerted effort amongst several
of the guests.
Rule: Concerted action liability rests upon the principle that all those who, in pursuance of a common plan or
design to commit a tortious act, actively take part in it, or further it by cooperation or request, or who lend aid
or encouragement to the wrongdoer, or ratify and adopt his acts done for their benefit, are equally liable with
him. An injured plaintiff may pursue any one joint tort-feasor on a concerted action theory. Such tort-feasor
may, in turn, seek contribution from others who acted in concert with him.
o An injured plaintiff may pursue any one joint tortfeasor or a concerted action theory.
Holding: Judgment for plaintiff; remanded for a jury trial. The conduct of the host and guests alleged to be
dangerous and tortious was the pushing or throwing of guests, against their will, from the barge into the water.
The court held that the liability of an individual guest did not depend upon whether he actually propelled the
injured guest into the water, but that participation in the concerted activity was equivalent to participation in
the accident resulting in the injury. The court then held that whether the guests acted in concert was a
question for the jury, and thus, summary judgment for two of the guests was not appropriate in this case.
o This has to be viewed in terms of a conspiracy. The defendants are egging on the people who are
throwing the plaintiff off the side. The owner is doing nothing, staying neutral and letting his guests have a
good time. Refer to 876.
o In this case, the court decides that they need to hear more facts to determine if the defendants
encouragement was within the guidelines set forth in 876.
Restatement 876. Persons Acting in Concert
o For harm resulting to a third person from the tortious conduct of another, one is subject to liability if he
(a) does a tortious act in concert with the other or pursuant to a common design with him, or
(b) knows that the other's conduct constitutes a breach of duty and gives substantial assistance or
encouragement to the other so to conduct himself, or
(c) gives substantial assistance to the other in accomplishing a tortious result and his own conduct,
separately considered, constitutes a breach of duty to the third person.
C. Liability by Operation of Law
1. When the doctrine of respondeat superior applies, the employer is a joint tortfeasor with her employee. Each
bears entire responsibility for the plaintiffs injuries.
In general, one is not liable for the torts of an independent contractor who is hired to do work when the
employer does not supervise the work of the person hired and has no right to control the manner in which the
work is performed.
o You can get 100% from the employer, or 100% from the employee, or you can split it up. Most likely you
will target the wealthier defendant.
D. Indivisible Injury
1. General
When they are not acting in concert, but are independently negligent. Both have cause-in-fact and proximate
liability toward the same injury. You cannot divide a concussion between two people. It is a single indivisible
injury. Therefore, both are 100% liable for the harm.
o Before comparative fault, 1 would have to pay something, and 2 would have to pay. When you do away
with joint and several liability, a defendant who cannot afford to pay screws the plaintiff from recovering
that portion of the recovery.
If you do away with it and adapt comparative fault, you lose the solvent defendant, and you are left to
collect 20% of $900,000 from Wal-Mart, but you are screwed on the 80% from Joe Schmo. If you do
away with it, you are giving up the big cash!
o If you have joint and several liability each defendant is potentially responsible, but the plaintiff chooses.
Justification, 20% of fault is still 100% of cause. Without you, it would not have happened. Therefore,
since you were involved, you can be held liable for all of it. If you have a problem with this, then you
and the other defendant can work it out amongst yourselves.
As long as your fault caused the harm, you are responsible for 100%. The plaintiff should receive his
full amount no matter what if it is available!
2. American Motorcycle Assn. v. Superior Court of Los Angeles County
Facts: Plaintiff suffered injuries during a novice motor-cross race; this claim is brought against defendant
organizations who sponsored and collected the entry fee for the race, based on their alleged negligent
management of the race; plaintiffs spine was crushed.
Issue: Whether comparative negligence eliminates joint and several liability amongst co-defendants when
they are indivisibly responsible for the crash of a biker in an event sponsored by them.
Rule: When independent negligent actions of a number of tortfeasors are each a proximate cause of a single
injury, each tortfeasor is thus personally liable for the damage sustained, and the injured person may sue one
or all of the tortfeasors to obtain a recovery for his injuries; the fact that one of the tortfeasors is impecunious
or otherwise immune from suit does not relieve another tortfeasor of his liability for damage which he himself
has proximately caused.
Holding: Judgment for plaintiff. The mere fact that it may be possible to assign some percentage figure to the
relative culpability of one negligent defendant as compared to another does not in any way suggest that each
defendant's negligence is not a proximate cause of the entire indivisible injury. Under the circumstances, a
concurrent tortfeasor whose negligence is a proximate cause of an indivisible injury remains liable for the total
amount of damages, diminished only "in proportion to the amount of negligence attributable to the person
recovering.
o They are discussing whether comparative fault does away with joint and several liabilities. Comparative
fault only decides percentages. Now, they have to decide if carries over to paying the plaintiff.
3. The Scorecard
Common law rule: joint and several 15 states
o 1 and 2 each 100% liable
Total abolition 16 states
o 1 20%, 2 80%, then that is the proportion they are liable for.
Threshold non-economic loss NY is included
o Economic loss: joint and several
o Non-economic loss: if you are 50% or below, you only pay your portion; above 50%, you are fully.
o Exception: statute does not apply to automobile cases.
4. Allocating Liability When an Intentional Tortfeasor cant be Had, But a Negligent One Can
One issue that comes up in discussions about joint and several liability is what courts should do when a party
who clearly caused intentional injury to a plaintiff is unidentified or insolvent, but another party who is accused
of breaching a duty to protect the plaintiff from such intentional conduct is available.
States are divided:
o Many states that have abolished or modified joint-tortfeasor liability will not limit plaintiffs recovery to the
available negligent tortfeasors percentage of fault.
o But other states limit plaintiffs recovery to the negligent defendants share of the fault. Since the
intentional tortfeasor bears the lions share of the fault, when one tortfeasors conduct was intentional,
plaintiff will recover only a small percentage of her total damages from the negligent party.
Example: a rapist breaks into a hotel and rapes a woman. The rapist will be much more liable,
percentage-wise, than the hotel, meaning that the victim will get peanuts since they cant afford what
the hotel would be able to.
E. Satisfaction of a Judgment and the Aftermath: Contribution and Indemnity
1. General
Where the joint and several liability rule governs either in whole or in part, a plaintiff may take a judgment
against several tortfeasors and is free to collect all of her damages from any of the tortfeasors.
Thus, a plaintiff may get a judgment of $100,000 against 1 (20 percent at fault) OR 2 (80 percent at
fault). She may recover the full $100,00 from either of the defendants.
However, once her judgment is satisfied, she cannot turn to collect again from the other defendant.
Only one satisfaction per customer is allowed.
o A defendant who has paid more than his fair share may seek contribution from the second defendant.
F. Settlement and Release
1. Legislatures have provided that a release of one tortfeasor does not release other joint tortfeasors unless the
release specifically says so.
2. Settling Tortfeasors and Contribution
One defendant settles with the plaintiff and the plaintiff reserves his right to sue the remaining tortfeasor. The
non-settling tortfeasor goes to trial and suffers a substantial judgment. May the litigating defendant who loses
the lawsuit turn to the settling defendant and seek contribution for the damages he has paid out as a result of
the judgment? The law is divided.
The Uniform Comparative Fault Act tries to resolve the problem.
o A release, covenant not to sue, or similar agreement entered into by a claimant and person liable
discharges that person from all liability for contribution, but it does not discharge any other persons liable
upon the same claim unless it so provides.
Example: 1 is 30% and 2 is 70% for a $1,000,000 claim. You go to 1 and want to settle. You tell
him youll settle for $100,000. He then goes to 2 and he decides that he wants to litigate. 2 loses,
and is responsible now for $900,000. 2 is clearly pissed about this, so he goes to 1 and says he
wants a contribution for $200,000.
The non-settling tortfeasor is going to the settling tortfeasor and says, who are you to make
deals for me? Essentially, by 1 settling they are putting the full burden on 2, simply because
2 was willing to try the case.
This is all under joint and several liability.
What are the implications of allowing contribution from the settling tortfeasor from the non-settling
tortfeasor? There will never be settlements! This will deter people from buying out early because they
dont want to see if there will be a large sum.
3. Release Handout 15108. Release or Covenant Not to Sue
If you settle with 1, you can go after 2. But, your settlement is not in the amount of dollars, it is the percent.
Thus, once you get 1 out of the way, you can only recover 70% from 2. This puts the gamble on the plaintiff
and not on the defendants!
o Plaintiff here loses $200,000 by settling with 1.
o This is a smart way out of this, except that there are times when insurance comes into play. Lets assume
1 only has $100,000 of insurance. You find this out before the jury gives you the amount. If plaintiff takes
the money, he may give up the percentage of fault that he could have been entitled to. For example, what
if 1 is eventually given a 70% portion on a $1,000,000 claim. Plaintiff would be losing $600,000 simply
by settling beforehand!
This problem has no good solution. Every way you try to work around it is bad.
Chapter 10: Strict Liability
A. Introduction
1. General
Strict liability is based merely on the fact that the defendants conduct has caused harm to the plaintiff. The
defendant actor who causes harm is liable even if the actor exercises reasonable care and does not intend to
interfere in any way with the plaintiff.
Contrasting strict liability with negligence
o Under negligence, when an actor exercises reasonable care the accident victim bears the residual
accident accosts caused by the actors conduct those costs that are better (cheaper) to incur than to
make efforts to avoid and the actor escapes liability.
Thus, when actors take adequate care under negligence, you might say that the accident victims are
strictly liable for the residual accident costs.
o In contrast, holding actors liable, strictly rather then merely for their negligence, shifts responsibility for the
residual accident costs from the victims to the actors.
B. Possession of Animals
1. One of the earliest forms of strict liability came from those who possess, confine, and manage animals that are
capable of causing harm both to persons and property when they escape confinement. There are three basic
categories:
(1) Livestock, including cattle, horses, sheep, goats, and the like;
o Courts impose strict liability.
o A majority of these cases involve damage to land, crops, and other property interests by the escaping
animals.
(2) Wild animals confined for a variety of reasons, both personal and professional;
o Courts impose strict liability.
o An exception to the general rule of strict liability concerns public zookeepers, who are liable only if shown
to have been negligent.
o Even if the plaintiff can prove that the animal that injured him was a wild animal, he still has to establish
that the defendant in fact owned or controlled the animal.
(3) Domesticated animals, such as dogs and cats, other than livestock.
o Possessors of category (3) animals will only be held liable if the plaintiff proves that the defendant pet
owner knew ahead of time that the animal was prone to violence.
o The most obvious way to establish prior knowledge is for the plaintiff to prove that the dog or cat in
question had, to the defendants knowledge, attacked someone else before.
This has led to the common observation that every dog is entitled to one bite, implying that the first
time a dog bites someone comes as a surprise to its owner.
Some legislatures have intervened to protect against dog bites by eliminating the common law
requirement of knowledge.
2. Sandy v. Bushey
Facts: Plaintiffs mare was grazing in his neighbors pasture, in which defendants colt was also occupied;
when plaintiff went to grain his mare, he was kicked by defendants colt and was seriously injured; plaintiff
sued, and trial court held for plaintiff.
Issue: Whether contributory negligence bars plaintiff from recovery under strict liability for injuries sustained
when defendants colt kicked him as he was tending to his mare.
Rule: If a person keeps a vicious or dangerous animal that he knows is accustomed to attack and injure
mankind, he assumes the obligation of an insurer against injury by such animal, and no measure of care in its
keeping will excuse him. His liability is founded upon the keeping of such an animal when he has knowledge
of its vicious propensities and his care or negligence is immaterial.
o In an action for an injury caused by such an animal, the plaintiff has only to allege and prove the keeping,
the vicious propensities, and the scienter. Negligence is not the ground of liability, and need not be
alleged or proved.
o Contributory Negligence: In order to relieve the keeper of a known vicious animal from his liability as an
insurer with which he is charged in this state, it must be established that the injury is attributable, not to
the keeping of the animal but to the injured party's unnecessarily and voluntarily putting himself in a way
to be hurt knowing the probable consequences of his act, so that he may fairly be deemed to have
brought the injury upon himself.
Holding: Judgment for plaintiff. The plaintiff led his mare away from the other horses in the pasture and
started to grain her when the defendant's horse approached in a threatening manner. The plaintiff drove him
away and turned to continue feeding the mare. The colt's return was silent and swift and his attack
unexpected. It cannot be said that the plaintiff voluntarily put himself in a way to be injured by the defendant's
horse, knowing the probable consequences of his act.
o Contributory negligence: Ordinary negligence does not matter; you have to ask for it. This means that
you have to act in a way that invites getting kicked by the horse. There needs to be gross negligence.
Negligence Comparison: Court holds that so long as the defendant has knowledge that his animal is
dangerous he can be held liable because he assumes the obligation and no manner of care will excuse him.
Strict liability applies when there is knowledge of danger!
o This differs from negligence, because then, if you know your dog is dangerous, you would have to
exercise reasonable care. With this holding, even if you do exercise reasonable care, nothing will save
you. You are strictly liable.
C. Abnormally Dangerous Activities
1. Fletcher v. Rylands
Facts: Plaintiff and defendant are neighbors; defendant commissioned a reservoir to be built, beneath which
there were coal shafts that had filled up with soil over time; the workers hired by the defendant knew this;
when they constructed the reservoir and water filled it, the water seeped into the shafts and flowed down
through them into the old workings, and thence into the plaintiffs mine, where it caused severe damage.
Plaintiff brings this appeal, after receiving a verdict and having it reversed at the next level.
Issue: Whether a defendant is strictly liable for the damage done to a neighbors property when he
precipitated it by performing non-natural acts.
Rule: The person who for his own purposes brings on his lands and collects and keeps there anything likely
to do mischief if it escapes, must keep it in at his peril, and, if he does not do so, is prima facie answerable for
all the damages that are the natural consequence of its escape.
Holding: Judgment for plaintiff. This was a non-natural action by the defendant.
o Two options
The defendants might lawfully have used that close for any purpose for which it might in the ordinary
course of the enjoyment of land be used; and if, in what is termed the natural user of that land, there
had been any accumulation of water, either on the surface or underground, and if, by the operation of
the laws of nature, that accumulation of water had passed off into the close occupied by the plaintiff,
the plaintiff could not have brought a claim.
Natural = no claim
If the defendants, not stopping at the natural use of their close, had desired to use it for any purpose
that I may term a non-natural use, and if in consequence of doing so, the water came to escape and
to pass off into the close of the plaintiff, then it appears to me that that which the defendants were
doing they were doing at their own peril; and, if in the course of their doing it, the evil arose, they
would be liable.
Non-natural = claim.
2. Turner v. Big Lake Oil Co.
Facts: The oil company constructed large, artificial earthen ponds to store polluted water that was generated
by its oil wells. On one occasion, water escaped from the ponds. It flowed onto the landowners' property and
into the water sources the landowners used to water their livestock. The landowners filed suit to recover
damages for the harm caused to their property. A jury held for the defendants, finding them in error, but not
negligent. Plaintiff appeals.
o Plaintiff argues that the defendants, in permitting the levees and dams of their artificial ponds to break
and overflow the land of the plaintiffs and thereby pollute the waters and injure their turf, are liable.
Issue: Whether the construction of artificial ponds for water storage in Texas is natural, and when those
ponds overflow, if the liability should be placed on the builders in a strict liability way.
Rule: Where water is collected in reservoirs, behind dams, in canals or in ditches, in the ordinary manner for
the purpose of being used as a motive power, in navigation, in irrigation, in mining, or for any other convenient
and lawful end, the person so collecting it can only be held liable on the ground of something unlawful in the
manner in which he has built or maintained his structure, -- that is, on the principle of negligence.
Holding: Judgment for defendant. Rylands does not apply in this case because there is a distinct difference
as to the reasons behind the construction of the reservoir and in this case. In Teas there are conditions very
different from those that obtain in England. A large portion of Texas is an arid or semiarid region. Given this,
water is stored in thousands of ponds, tanks, and lakes on the surface of the ground. The storage of water is
a natural or necessary and common use of the land, necessarily within the contemplation of the state and its
grantees when grants were made, and obviously the rule announce in Rylands, predicated upon different
conditions, can have no application here.
3. Indiana Harbor Belt R.R. v. American Cyanamid Co.
Facts: Defendant manufacturer of chemicals was sued by plaintiff switching line for cost of decontamination
measures that resulted from railroad tank car leak. Plaintiff based its counts on theories of negligence and
strict liability arising from an abnormally dangerous activity. The court below dismissed the negligence claim
with prejudice and granted summary judgment on plaintiff's strict liability count.
Issue: Whether the shipper of a hazardous chemical by rail should be strictly liable for the consequences of a
spill or other accident to the shipment en route.
Rule:
o Restatement 520
o A paradigmatic case of strict liability is found where (a) the risk (probability) of harm is great; (b) the harm
that would ensue if the risk materialized is great; (c) yet, such an accident can not be prevented by the
exercise of due care; (d) the activity is not a matter of common usage, so there is no presumption that it
was a highly valuable activity despite its unavoidable riskiness; (e) the activity is inappropriate to the place
in which it takes place; and (f) the value to the community of the activity does not appear to be great
enough to offset its unavoidable risks.
Holding: Judgment for defendant regarding strict liability; remanded to lower court for negligence claim. The
court found that the strict liability regime did not apply, as the leak was not caused by the inherent properties
of the chemical; rather, the leak was found to arise from carelessness in transportation. Also, plaintiff failed to
establish hazardous nature of activity. The court reasoned that taking care can prevent accidents that are due
to a lack of care, and when a lack of care can be shown in court, such accidents are adequately deterred by
the threat of liability for negligence.
o Judge Posner says there is no sense to put strict liability on the defendant because they were not
responsible for the route in which the train traveled. Negligence should be enough of a case to prove if
they want to find the defendant liable for something here.
4. Foster v. Preston Mill Co.
Facts: Blasting operations conducted by defendant frightened mother mink owned by plaintiff and caused the
mink to kill their kittens. After the plaintiff complained to the defendant, they reduced the strength of their
blasts, but continued blasting. The trial court, without a jury, rendered judgment for plaintiff. The theory
adopted by the court was that, after the defendant was notified of the effect that its blasting operations were
having upon the mink, it was absolutely liable for all damages of that nature thereafter sustained. The trial
court decided in defendant's favor on the question of nuisance.
Issue: Whether a defendant should be held strictly liable for damages resulting from its blasting activities
when the damages only occurred because the injured animals were unusually sensitive in nature.
Rule: One who carries on an ultrahazardous activity is liable to another whose person, land, or chattels the
actor should recognize as likely to be harmed by the unpreventable miscarriage of the activity for harm
resulting thereto from that which makes the activity ultrahazardous, although the utmost care is exercised to
prevent the harm.
Holding: Judgment for defendant. Strict liability is still not absolute liability. If it is imposed with blasting, it is
because of the set of risks that come with blasting operations (things flying, vibrations, etc.). In this case, a
couple of miles from the blasting, minks were killing each other. The court imposed a proximate cause
analysis the risks that strict liability is used to defend against were not the risks that the plaintiffs claimed to
recover from. Minks killing one another are not the special risks that come with the territory of blasting.
The thing which makes blasting ultrahazardous is the risk that property or persons may be damaged or
injured by coming into direct contact with flying debris, or by being directly affected by vibrations of the
earth or concussions of the air. The relatively moderate vibration and noise that appellants blasting
produced at a distance of two and a quarter miles was no more than a usual incident of the ordinary life of
the community.
Chapter 11: Products Liability
Product Liability 1. Liability of Commercial Seller or Distributor for Harm Caused by Defective Products
o One engaged in the business of selling or otherwise distributing products who sells or distributes a
defective product is subject to liability for harm to persons or property caused by the defect.
Product Liability 2. Categories of Product Defect
o A product is defective when, at the time of sale or distribution, it contains a manufacturing defect, is
defective in design, or is defective because of inadequate instructions or warnings. A product:
(a) contains a manufacturing defect when the product departs from its intended design even though
all possible care was exercised in the preparation and marketing of the product;
(b) is defective in design when the foreseeable risks of harm posed by the product could have been
reduced or avoided by the adoption of a reasonable alternative design by the seller or other
distributor, or a predecessor in the commercial chain of distribution, and the omission of the
alternative design renders the product not reasonably safe;
(c) is defective because of inadequate instructions or warnings when the foreseeable risks of harm
posed by the product could have been reduced or avoided by the provision of reasonable instructions
or warnings by the seller or other distributor, or a predecessor in the commercial chain of distribution,
and the omission of the instructions or warnings renders the product not reasonably safe.
o Note on (b): You can sue up and down the distributive chain because they each were sellers of a
defective product. But, this does not define defect. That means that somebody screwed up in the
manufacturing. What Twerski says in this section is that you can get the retailer, but you dont have to
prove that he is negligent you just have to prove that someone above him screwed up in risk-utility.
Retailers are involved in this because they should have taken the appropriate steps to ensure that the
product was safe. Some states are limiting this option.
This is possible because but-for the negligence of the retailer making a defective product
available, the person would not have been injured in using it.
Defective product is defined in the first sentence of (b).
Easier way of saying this is negligence! A product is defective in design if it is negligently designed,
even though they took all reasonable precautions.
You better have one that works and is economically efficient in order to prove negligence this is
because the consequences are so bad. YOU MUST BE ABLE TO PROVE AN ALTERNATIVE IS
REASONABLE!
Courts are wedded to the term strict liability. They dont like negligence. These cases say, Use strict
liability, although they were actually employing negligence analysis.
(1) Manufacturing Defects
1. General
A product that departs from its intended design is defective.
o Compare the design of the allegedly defective product against the blueprint for the way the product is
designed, and if there is a difference, then you have a design defect.
o Meet your own standards!
Often expert testimony is available to prove that the defect was present in the product when it was first
manufactured. When such testimony is not available or is, at best, equivocal, plaintiffs may face problems
establishing defect.
Defect is easy to prove. But, proof that it was defective when it left the manufacturers hand is difficult.
o Rely on exclusive control to show that the defendant is responsible.
Quality control could be lowered in order to prevent design defect potential, but all that would lead to is faulty
quality control you would get sued for something different.
o Defendant can no longer rely on quality control as a defense in strict liability.
You must attack that the bottles should have been safer than they were all models!
2. Welge v. Planters Lifesavers Co.
Facts: Plaintiff consumer cut his hand and was left with permanent impairment when a jar of peanuts
shattered as he was replacing its cap. He filed a products liability action against defendant corporations, the
seller, the manufacturer of the product, and the manufacturer of the glass jar. Defendants filed a motion for
summary judgment, which the trial court granted on the ground that the consumer had failed to exclude
possible causes of the accident other than a defect introduced during the manufacturing process. Plaintiff
appeals.
Issue: Whether a plaintiff can recover for product liability when there is not enough evidence to absolutely
exclude other possible causes of the accident.
Rule: The plaintiff in a products liability suit is not required to exclude every possibility, however fantastic or
remote, that someone other than one of the defendants caused the defect, which led to the accident. The
doctrine of res ipsa loquitur teaches that an accident that is unlikely to occur unless the defendant was
negligent is itself circumstantial evidence that the defendant was negligent. If it is the kind of accident that
would not have occurred but for a defect in the product, and if it is reasonably plain that the defect was not
introduced after the product was sold, the accident is evidence of the defect.
Holding: Judgment reversed for plaintiff, remanded for trial. A seller who was subject to strict products liability
was responsible for the consequences of selling a defective product even if the defect was introduced without
any fault on his part by his supplier or by his supplier's supplier. The court held that misuse was no defense,
the accident was not due to mishandling after purchase, and the probability that the defect was introduced by
one of the corporations was very high.
Res Ipsa Loquitur: Defendants argue that there is strict liability, not negligence, and because of this, they
should get off because there was another cause. They claim the product was not defective when it left their
hand. You still have to pin the tail on the donkey! Plaintiff must prove that it didnt happen after it left the
manufacturer, and that it was faulty when it left the factory. You rely on res ipsa loquitur for this.
o That is the essence of 3 of the Product Liability Restatement.
3. Product Liability 3. Circumstantial Evidence Supporting Inference of Product Defect
It may be inferred that the harm sustained by the plaintiff was caused by a product defect existing at the time
of sale or distribution, without proof of a specific defect, when the incident that harmed the plaintiff:
o (a) was of a kind that ordinarily occurs as a result of product defect; and
o (b) was not, in the particular case, solely the result of causes other than product defect existing at the time
of sale or distribution.
Note: defect is substituted for negligence, and they are the same doctrine in this sense. The only difference is
that in negligence, you have to draw an inference that they were negligent, and more probably negligent than
not. Here you have to draw the inference that the defect was there when it was in the hands of the
manufacturer.
This is the same as res ipsa loquitur!
4. Causation
If a bottle drops from 5 feet, and the defense argues that the design defect was that it could not resist a drop
of more than 2 feet, you need to ask the expert, What design could have been made in order to resist a drop
of 5 feet? You need to overcome the fact that the design was the but-for cause of the accident.
o But-for the design defect, it would have sustained a drop from X feet.
It is not sufficient that the defect existed you need to have a standard.
o Then you need to prove that had the product been better, it would have adhered to the standard.
o You need to show that there was causation!
How do you decide how good the product should have been?
o The expert will tell you how it could have been done, but the defendant will then reply that the burden is
too high. B<PL (negligence).
Frequency of people dropping bottles from 5 feet (P).
Cost of making the bottle better to sustain a drop from a greater height (B).
o The B in products design cases is generally higher than what already exists.
You must come into court as the plaintiff saying that there is a reasonable alternative design!
This will hopefully get you past a motion for summary judgment.
A reasonable alternative design will show that the B is possible to adjust, and possible to do so in a
reasonable manner that is not excessively high.
(2) Design Defect
1. Risk-Utility Balancing: Reasonable Alternative Design
The simplest and most straightforward solution to the problem would be to utilize the Learned Hand/risk-utility
balancing test to decide whether a product design is reasonably safe.
o This is adopted in Product Liability 2(b). In requiring that a plaintiff establish that the foreseeable risks of
harm could have been reduced or avoided by the adoption of a reasonable alternative design, the
Restatement advocates that courts examine the risk-utility tradeoffs in determining whether a product is
defective.
o Comment f sets forth factors to consider: the magnitude and probability of the foreseeable risks of harm,
the instructions and warnings accompanying the product, and the nature and strength of consumer
expectations regarding the product, including expectations arising from product portrayal and marketing.
The relative advantages and disadvantages of the product as designed and as it alternatively could have
been designed may also be considered.
When evaluating the reasonableness of a design alternative, the overall safety of the product must be
considered. It is not sufficient that the alternative design would have reduced or prevented the harm
suffered by the plaintiff if it would also have introduced into the product other dangers of equal or
greater magnitude.
Rule: Under a modified formulation, the consumer expectation test would establish the product's risks
and utility and the inquiry would then be whether a reasonable consumer would consider the product
design unreasonably dangerous.
The jury should engage in the risk-utility balancing required by the court's modified consumer
expectation test when the particular facts do not reasonably permit the inference that the product did
not meet the safety expectations of the ordinary consumer.
o Holding: Judgment for plaintiff. Because there was sufficient evidence as a matter of law to support the
determination that the tools were unreasonably dangerous based on the ordinary consumer expectation
test, we conclude that this instruction was appropriately given to the jury.
This court attacks the Restatement. They say you do not have to prove RAD in order to make a prima
facie case.
5. Anderson v. Owens-Corning Fiberglas Corp. (Failure to Warn)
Facts: Plaintiff sued defendants, the manufacturers of asbestos-containing products, claiming strict liability in
tort for failing to warn plaintiff of design defects. A jury found that defendants' products were not defective;
however, the appeals court granted plaintiff a new trial because knowability was irrelevant in a failure to warn
case. Defendants argue that even those at the vanguard of scientific knowledge at the time the products were
sold could not have known that asbestos was dangerous to users in the concentrations associated with their
products.
Issue: Whether presenting evidence of lack of knowledge of a defective condition at the time of manufacture
is permissible as a defense against a design defect claim.
Rule: Knowledge, actual or constructive, is a requisite for strict liability for failure to warn.
o Failure to warn in strict liability differs markedly from failure to warn in the negligence context. In strict
liability, as opposed to negligence, the reasonableness of the defendant's failure to warn is immaterial.
o The fact that a manufacturer acted as a reasonably prudent manufacturer in deciding not to warn, while
perhaps absolving the manufacturer of liability under the negligence theory, will not preclude liability under
strict liability principles if the trier of fact concludes that, based on the information scientifically available to
the manufacturer, the manufacturer's failure to warn rendered the product unsafe to its users.
Holding: Judgment for defendant, remanded for knowledge to be considered. A defendant in a strict products
liability action based upon an alleged failure to warn of a risk of harm may present evidence of the state of the
art, i.e., evidence that the particular risk was neither known nor knowable by the application of scientific
knowledge available at the time of manufacture and/or distribution.
Negligence: Exactly the equivalent to negligence, and the majority of courts use the reasonable person
standard. Courts are quite explicit that it is a negligence test. You failed to warn, which means you failed to
do what a reasonable person should do. The problem with failure to warn is implementing it.
o Twerski doesnt understand why this isnt just stated as negligence. Why are they trying to make it into
strict liability when they are blatantly describing negligence?
Under negligence law, if your own testing does not show the risk but the knowledge is available to the
scientific community, you are not negligent. This is dumb, because if you did not know it, you should
have known it this is negligence!
o
Facts: Plaintiffs sue for damages in trespass and nuisance from the deposit on their property of microscopic,
airborne particles of heavy metals that came from defendants copper smelter some 4 miles away. These
cannot be detected by human senses and did no physical damage to the plaintiffs property.
Issue: Whether defendant had the requisite intent to commit intentional trespass, and if so, whether there is
proof of actual damages resulting therefrom.
Rule:
o Restatement 8A. Intent
The word "intent" is used to denote that the actor desires to cause consequences of his act, or that he
believes that the consequences are substantially certain to result from it.
o Borland v. Sanders Requirements for to recover in trespass
In order to recover in trespass by airborne pollutants, a plaintiff must show: 1) an invasion affecting
an interest in the exclusive possession of his property; 2) an intentional doing of the act which results
in the invasion; 3) reasonable foreseeability that the act done could result in an invasion of plaintiff's
possessory interest; and 4) substantial damages to the res.
Holding: Judgment for defendant. As for intent, the defendant acted on its own volition and had to appreciate
with substantial certainty that the law of gravity would visit the effluence upon someone, somewhere; thus,
plaintiff proves this element. But As for actual damages, no useful purpose would be served by sanctioning
actions in trespass by every landowner within a hundred miles of a manufacturing plant. This would bring
about limitless litigation. There are no actual damages to the plaintiffs property. Since this is an element of the
action, the plaintiff who cannot show that actual and substantial damages have been suffered should be
subject to dismissal of his cause.
o They are claiming that not only are they interfering with the use and enjoyment of the property, but they
are also trespassing onto the property with the microscopic particles.
Courts determine that you cannot establish a trespass this way (it has to be changed into something
more nuance-based). Ultimately, these trespass actions became nuisances how severe was the
interference with the property? This is because trespass is a black and white tort; either you did or did
not trespass. Nuisance has a grey area, which is very exciting to discuss.
Two kinds of trespassers
o Direct trespassers that are subject to the traditional rule that any interference with exclusive possession
constitutes a trespass.
Entitle you to nominal damages.
o Indirect trespassers arising from some form of pollution that require proof of actual harm.
Entitle you to actual and substantial damages.
C. Private Nuisance
1. Plaintiff Must Prove
(1) A basis for liability;
o The mere fact that a plaintiff has suffered a significant and unreasonable invasion of her property will not
make out a cause of action for nuisance absent conduct that is tortious.
(2) Significant harm, and;
o It would be sheer folly to protect a landowners right to use of enjoyment of her property from trivial harm
or annoyance. Nuisance will not deal with minor/insignificant harms.
(3) An unreasonable invasion of the plaintiffs land.
o This element has been subject to considerable debate. The defendant may be reasonable, but the
invasion of the land must be unreasonable.
2. Hughes v. Emerald Mines Corp.
Facts: Plaintiffs claim that defendants construction on adjacent land caused the drying of two of their wells,
causing them to not have fresh water from the time the construction began.
Issue: Whether non-trespassory invasion can be made out, and if so, whether damages should be awarded
for permanent damages or compensatory.
Rule:
o Restatement 822
One is subject to liability for a private nuisance if, but only if, his conduct is a legal cause of an
invasion of another's interest in the private use and enjoyment of land, and the invasion is either:
(a) intentional and unreasonable, or
(b) unintentional and otherwise actionable under the rules controlling liability for negligent or
reckless conduct, or for abnormally dangerous conditions or activities.
o Restatement 825. Intentional Invasion What Constitutes
An invasion of another's interest in the use and enjoyment of land or an interference with the public
right, is intentional if the actor:
Physical harm is worse than economic, repeated versus isolated incidents, intentional versus
unintentional harm, financial vulnerability of plaintiff, etc.).
Ratio between Punitive and Actual Damages should be single-digit.
Problem: Actual damages favor minimal awards and under-compensation for the plaintiff. To the
extent that the measure of punitive damages is based on a ratio, we must keep in mind that actual
damages might not be fully compensatory. This conclusion comes from the aforementioned
guidelines in (1).
Compare punitive damages to similar civil and criminal penalties from similar cases.
Evaluate how that jurisdictions criminal and civil penalties view the violation or crime, and use that as
a means to gauge a proper amount.